SlideShare une entreprise Scribd logo
1  sur  187
Respiratory diseases
Archer online USMLE reviews
www.ArcherReview.com
All Rights reserved
Archer Slides are intended for use with Archer USMLE
step 3 video lectures. Hence, most slides are very brief
summaries of the concepts which will be addressed in a
detailed way with focus on High-yield concepts in the
Video lectures.
URTIs
Allergic Rhinitis
 Hay fever
 Onset under age 30
 Peak incidence – childhood & adolescence
 Most common chronic disease in the USA and significantly
affects quality of life
 Pathophysiology : Type I hypersensitivity reaction to allergens
 Common allergens : Seasonal Allergens: Tree pollen (early
spring), Grass pollen (late spring) and Outdoor Molds (summer
and fall) ) , Perennial : Dust mites and Animal dander Irritant:
Cigarette Smoke
 Associated conditions :  Atopy : Eczematous Dermatitis ,
Allergic Rhinitis and Asthma
 Allergic Triad : Aspirin Allergy, Nasal
Polyp and Asthma
Allergic Rhinitis
Symptoms:
Specific : Sneezing, Rhinorrhea, Nasal congestion and Pruritus of the
nose, eyes, and throat , Eye Tearing and Conjunctival discharge
Symptoms due to Chronic Nasal Obstruction: Mouth Breathing,
Snoring, Anosmia, Cough, Headache and Halitosis
Signs : * Look for antihistamine induced Hypertension in these guys
*Nose exam : pale blue and boggy mucosa, clear discharge
*Face exam: “Allergic Shiners”  bluish purple rings arround both
eyes due to chronic mid face venos congestion
“ Dennie’s Lines:  Skin folds under the eyes
“ Allergic Salute:  transverse nasal crease from
chronic rubbing
*Sinuses: r/o sinusitis  purulent discharge, tenderness and impaired
transillumination
Allergic Rhinitis
 Diagnosis : * Skin testing  Gold Standard
* RAST  use this if unable to do a skin test or if its
contraindicated
* CBC  may show eosinophilia
* IgE levels are elevated
 D/D : 1) Nasal causes of Rhinitis : Nonallergic rhinitis ( eosinophila
synd), Nasal polyps, Vasomotor rhinitis, infectious rhinitis, Rhinitis
medicamentosa
2) Medications: Aspirin, Clonidine, Hydralazine, Labetalol,
propranolol, tearazosin, OC pills
 Management  Do Skin test / RAST and find the responsible
Allergen. Advise the pt to avoid the allergen. “Avoid pets in the bed if
its found to be animal dander”
 Intranasal Steroids ( are the drug of choice for pts with
chronic symptoms. Can be used prn but most effective when used as
maintainance therapy  fluticasone, beclomethasone) ,
Antihistamines ( cetrizine, loratidine) , Saline nasal drops ,
decongestants ( pseudoephedrine), nasal cromolyn
Allergic Rhinitis
Antihistamines vs. Nasal Corticosteroids. The majority of
studies favor the use of intranasal corticosteroids over
sedating or nonsedating antihistamines for relief of
symptoms of nasal allergy. These results are true for
seasonal and perennial allergic rhinitis. ( antihistamines are
used for immediate symptom relief)
Immunotherapy: Immunotherapy is indicated in patients who
present with any of the following characteristics:
 Insufficient control by pharmacotherapy;
 Insufficient control of symptoms;
 A desire not to take medication;
 Medication produces undesirable side effects; and
 A desire to avoid long-term pharmacotherapy (with
intranasal steroids)
Case Study
 You are treating an 18-year-old white male college
freshman for allergic rhinitis. It is September and he
tells you that he has severe symptoms every autumn,
which impair his academic performance. He has a
strongly positive family history of atopic dermatitis.
Which one of the following medication is
considered optimal treatment for this condition?
 Intranasal glucocorticoids
 Intranasal cromolym sodium
 Intranasal decongestants
 Intranasal antihistamine
Ans
 Topical intranasal glucocorticoids are currently believed to be the most efficacious
medications for the treatment of allergic rhinitis. They are far superior to oral
preparations in terms of safety.
 Cromolyn sodium is also an effective topical agent for allergic rhinitis; however, it
is more effective if started prior to the season of peak symptoms.
 Because of the high risk of rhinitis medicamentosa with chronic use of topical
decongestants, these agents have limited usefulness in the treatment of allergic
rhinitis.
 Some of the newer oral antihistamines have been found to be comparable in
efficacy to intranasal steroids, but their use slightly increases the incidence of
adverse effects and drug interactions. They are not as useful for congestion as they
are for sneezing, pruritis, and rhinorrhea. Newer agents are relatively free of
sedation. Overall, they are not as effective as topical glucocorticoids. Azelastine ,
an intranasal antihistamine, is effective in controlling symptoms but can cause
somnolence and has a very bitter taste.
Vasomotor Rhinitis
 Diagnosis of exclusion
 Symptoms similar as Allergic Rhinitis  has rhinorrhea, congestion, nasal
obstruction ( normal nasal exam, normal Ig E and Normal skin test/ RAST
)
 No specific test is available to diagnose vasomotor rhinitis  First
exclude allergic rhinitis as the cause of symptoms by using conventional
skin testing or by evaluation for specific IgE antibodies to known allergens.
 Rx: Stepwise Approach ( next slide )
 Pregnancy
Step 1: Nasal Saline
Step 2: Intranasal Atrovent (Pregnancy Category B)
Traditional oral antihistamines have no established beneficial effect in patients
with vasomotor rhinitis and may be associated with sedation.
Newer, less-sedating antihistamines also have no proven effectiveness for
vasomotor rhinitis, and their administration delays proper treatment while
incurring significant cost and burden to the health care system. Topical
antihistamines are used as first choice if symps are rhinorrhea, sneezing,
post nasal drip
Rhinitis Medicamentosa
Pathophysiology
 Associated with topical agent use >5-7 days
 Tachyphylaxis associated with medications
 Nasal Decongestants (Afrin, Neo-Synephrine)
 Other associated medications
 Reserpine
 Oral Contraceptive pills
 Inderal
 Aldomet
Symptoms
 Rebound nasal Congestion after nasal Decongestant
Signs
 Fiery red edema at nasal mucosa
Management
 Intranasal Steroid
 Withdrawal of nasal Decongestant
Acute & Chronic Sinusitis
Criteria for diagnosis:
 Maxillary toothache
 Purulent nasal secretion
 History of colored Nasal discharge
 Poor response to nasal Decongestants
 Abnormal Sinus Transillumination
If 4 or more criteria +  diagnosis is definite
If 2 or 3 crieria +  Diagnosis is intermediate  recommended initial study Sinus CT
If less than 2 criteria  negative for sinusitis
 Most common is maxillary sinusitis. Next common is Frontal. Ethmoids are most
commonly affected in children. Spenoids has highest risk of intracranial spread
 Symptoms may last as long as 4 weeks in acute sinusitis, Symptoms b/w 4-8 weeks is
subacute ans symptoms persisting > 8 weeks is chronic sinusitis.
In recurrent sinusitis, there are 3 or more episodes of acute sinusitis per year, and
different episodes may be caused by different organisms.
 Signs
 Diagnostic tests
 Step wise Treatment
 Complications
Acute & Chronic Sinusitis
Signs :
Nasal Mucosa erythema and boggy due to edema
 Contrast with Allergic Rhinitis (pale, boggy mucosa)
 Nasal exam to view pus discharge from lateral wall
Instruments Nasal speculum (minimal visualization) , Flexible
Nasolaryngoscopy
 Middle Meatus (hiatus semilunaris)  Drains Maxillary, Frontal, and
Anterior Ethmoid  Consider local Topical Decongestant application
 Superior Meatus (Rarely discharge is seen)  Drains posterior ethmoid
sinus
 Turbinates enlarged
 Sinus tenderness to percussion
 Sinus Transillumination in darkened room
 Frontal and maxillary sinus
Acute & Chronic Sinusitis
 Symptoms suggesting bacterial etiology
 Symptoms persist beyond 10 to 14 days,
Remember that under 10 days, viral sinusitis
predominates,  By day 10, 40% of sinusitis
resolves spontaneously  0.5% of viral URIs
develop into bacterial sinusitis
 Symptoms worsen after 5-7 days ( “double”
sickening)
 purulent nasal discharge
 “Unilateral” maxillary sinus tenderness
 Maxillary tooth or facial pain (esp. if unilateral)
Acute & Chronic Sinusitis
 Don’t culture nasal swabs  not cost effective
 Diagnosis is clinical in Acute Sinusitis
 Indications for Imaging
 Complicated sinusitis , Chronic or recurrent sinusitis , Sinusitis refractory to maximal medical
therapy
 Imaging is not needed in routine cases  Empiric therapy for 1-2 courses is appropriate
1. Sinus X-Ray (Sinus CT preferred)  Plain radiographic signs consistent with sinusitis
include greater than 6 mm of mucosal thickening in adults and 4 mm in children,
greater than 33% loss of air space volume in the maxillary sinuses, or opacification–air-
fluid levels.
 Single Waters' View X-Ray is sufficient
 Indication (rarely indicated unless CT not available)
 Complicated Acute Sinusitis & Suspected Chronic Sinusitis
 Sinus CT (gold standard) Indications
 Osteomeatal complex occlusion
 Complicated acute sinusitis  orbital cellulitis etc
 Chronic Sinusitis
 Recurrent Sinusitis
 Allergic Fungal Sinusitis
 Sinus MRI
 No advantage over Sinus CT (and more false positives)
 Indications : Suspected neoplasm and Fungal Sinusitis
Acute Sinusitis - Complications
 Unless severe symptoms of acute sinusitis develop, such as fever,
facial pain or tenderness, or periorbital swelling, antibiotics
should be withheld for 10 to 14 days.
 Complications : Orbital Cellulitis, Meningitis, Extradural
abscess , Subdural abscess , Brain abscess , Osteomyelitis and
Cavernous Sinus Thrombosis
 Symptoms: Red Flag (consider immediate ENT referral)
 High Fever over 102.2 F (39 C) or peristent fever
 Visual complaints (e.g. Diplopia)
 Periorbital edema or erythema ( check for EOMs  ?pain)
 Mental status changes
 Severe facial or dental pain
 Infraorbital hypesthesia
 consider referral in immunodeficiency or if persistent symptoms
despite treatment
Sinusitis - Treatment
General Measures
 Symptomatic relief : Warm, moist compresses over sinuses ,
Tylenol
 Nasal Saline spray (2% buffered saline)
 Effective Decongestant
 Also use as pretreatment prior to Intranasal Steroid
 Effective in recurrent Sinusitis when used daily
 Systemic Decongestant: Pseudoephedrine
 Mucolytic : Guaifenesin (e.g. Mucinex) 600 to 1200 mg PO bid 
there is no evidence that mucolytics are useful adjuncts
 Topical Decongestant (Maximum of 3 days of use)
 Oxymetazoline or Phenylephrine (Neo-Synephrine)
 Intranasal Steroid (treat for 3-6 weeks minimum)
 Chronic Sinusitis
 Nasal Polyp
 Avoid Antihistamines!!
 Dries secretions and Impedes osteomeatal complex drainage
Sinusitis - Treatment
ANTIBIOTICS:
 Indicated only in acute bacterial Sinusitis
 Protocol Antibiotic course  Minimum course: 10-14 days
Longer course for persistent symptoms: 28 days
 Change antibiotic if no improvement in 3 days REMEMBER
THAT Beta-lactamase resistance in acute cases: <30% , Beta-
lactamase resistance in chronic cases: 40-50%
 First-Line  Indications to start on first-line agents Mild to
moderate symptoms , No daycare exposure & No recent antibiotic
use
 Amoxiicillin  Disadvantages: Misses Beta-lactamase
producers : Haemophilus Influenzae , Moraxella catarrhalis &
Penicillin Resistant Pneumococcus (increasing)
 Trimethoprim Sulfamethoxazole (Bactrim) No longer
recommended as first-line agent , Higher resistance rate than
other agents
 Disadvantages : Misses Staphylococcus , Risk of Toxic
Epidermal Necrolysis& Risk of Steven's Johnson
Syndrome
Sinusitis - Treatment
 Second-Line  Indications to start on
second-line agents : Severe symptoms Daycare
exposure , Recent antibiotic use
 Amoxicillin-Clavulanate (Augmentin ) or
Cefuroxime (Zinacef) , Cefpodoxime
 Avoid Cefixime ( poor Gram + coverage )
 Third Line recommendation If no
improvement with above a) Consider adding
Flagyl to second-line agents b) Consider
second-line agent for longer course (4 week) c)
Switch to Fluoroquinolone (avoid under 16 yrs
of age ), Moxifloxacin or Gatifloxacin
(Tequin)
Sinusitis - Treatment
Management : Penicillin or Cephalosporin Allergy
 Macrolide antibiotics (High bacterial resistance rate) 
Erythromycin , Azithromycin (Zithromax) or
Clarithromycin (Biaxin)
 Trimethoprim-Sulfamethoxazole (Bactrim)  Increasing
bacterial resistance, So other agents are preferred for
Sinusitis
 Clindamycin  Consider in combination with Rifampin
if severe , Poor efficacy against Gram Negative Bacteria
 Fluoroquinolones ( avoid under age 16 years )
Sinusitis - Treatment
 “Unless severe symptoms of acute sinusitis develop, such as fever, facial pain or tenderness, or periorbital swelling, antibiotics should be
withheld for 10 to 14 days. Although the primary therapy for acute bacterial sinusitis is antibiotics, increasing resistance to penicillin may
necessitate the use of alternative antibiotics. The choice of antibiotics is based on predicted efficacy, cost, and adverse effects. A 10- to 14-
day course is generally adequate for acute disease, but shorter courses may be indicated for newer antibiotics. If there is no improvement
in 3 to 5 days, an alternative antibiotic should be considered” ( guidelines, journal of clinical immunology, 2006)
 Primary therapy for acute bacterial sinusitis is antibiotics with a 10-
to 14-day course considered adequate. Amoxicillin is a drug of
choice with trimethoprim-sulfamethoxazole an alternative.
 If no response occurs within 3 to 5 days, a change to high-dose
amoxicillin-clavulanate, cephalosporins, or macrolides may be
indicated.
 In areas of high antibiotic resistance or with failure to improve after
21 to 28 days, broad spectrum single agents should be considered,
such as amoxicillin-clavulanate, cefuroxime, or cefpodoxime, or use
of anaerobic coverage, such as clindamycin or metronidazole.
 Nasal corticosteroids are indicated in acute and chronic sinusitis
and short-term adjunct oral steroids may be used after failure of
response or when nasal polyps are present.
 Saline nasal sprays may help to reduce crusting!!
Acute Pharyngitis
 Symptoms: Sore throat , Dysphagia & Odynophagia (pain with swallowing)
 Generalized symptoms : Fever, Chills, Malaise, Headache , Abdominal Pain ,
Nausea or Vomiting
 Symptoms suggestive of viral illness: Coryza, Conjunctivitis & Hoarseness
 Signs
 Viral
 Non-exudative pharyngeal erythema
 Exception: Tonsillar exudate in Mononucleosis (EBV)
 Vesicular OR ulcerative oral lesions
 Conjunctivits in Adenovirus and Kawasaki Disease
 Group A Streptococcus and other bacteria  clues are Enlarged tonsils with or
without exudate , Petechiae on Soft Palate (pathognomonic) , Erythema , Tender
cervical Lymphadenopathy
 Strawberry Tongue (in Scarlet Fever)
 Peritonsillar Cellulitis or Peritonsillar Abscess  Suspect Unilateral erythema of
Soft Palate , Uvula deviated , Dysphagia, Odynophagia & Fever
 Diphtheria  Suspect when Gray membranous exudate covers tonsils and
pharynx or Exudate bleeds easily on removal
 Kawasaki Disease  Suspect when Pharyngitis with strawberry Tongue in age
<5 years , Non-purulent Conjunctivitis (also in Adenovirus) & Palmar erythema
and cracked red lips after 3 days
STREP THROAT
 Acute Pharyngitis caused by Group A beta
hemolytic streptococci.
 Most common in children 5-12 yr old
 Infectivity  Decreases 1-3 days after
antibiotic started
 Return to School and day care
recommendations  Child should receive
Antibiotics for minimum of 24 hours and
Afebrile
Strep Throat
Complications
 Non-suppurative
 Rheumatic Fever  we Rx Strep Throat to prevent this. ABX Rx does
not prevent PSGN
 Acute Post-Streptococcal Glomerulonephritis ( PSGN)
 Suppurative
 Peritonsillar Abscess
 Suppurative Otitis Media
 Cervical lymphadenitis
 Acute Sinusitis
 Mastoiditis
 Meningitis
 Bacteremia
 Endocarditis
 Pneumonia
Strep Throat – Strep Score
Original Criteria (interpretation
below based on these)  1
point for each
 Tonsillar exudate
 Tender, anterior cervical
adenopathy
 Cough absent
 Fever present
 Modifiers : Age younger
than 15 years: +1 point,
Age 15 to 45 years: 0 points
& Age over 45 years: -1
points
ER and OP probability:
 Score 0: Streptococcus
probability 1% (3% in ER)
 Score 1: Streptococcus
probability 4% (8% in ER)
 Score 2: Streptococcus
probability 9% (18% in ER)
 Score 3: Streptococcus
probability 21% (38% in
ER)
 Score 4: Streptococcus
probability 43% (63% in
ER)
Strep throat (?) - Approach
 Strep Score 4 (or Strep Score 2 if patient unreliable)
 Treat with antibiotics
 Strep Score 2 to 3: Perform rapid antigen test
 Antigen test positive: Treat with antibiotics
 Antigen test negative: Throat Culture (Requires 24 hour
minimum for adequate growth ) most specific (99%).
Sensitivity 90%. Not recommended as primary test due to
24 hour delay . Remember that –ve Rapid strep does not
rule out Strep throat
 Strep Score 0 to 1
 Provide Pharyngitis Symptomatic Treatment  salt water
gargles, sucking candies, ibuprofen
Strep throat - Antibiotics
 Penicillin is the first choice ( coz its strep) 
penicillin VK 500 mg  If using this standard course
duration is 10 days. Alternatively use Amoxicillin
500 bid in adults/ 10 days
 Alternative antibiotics : Five days of alternative
antibiotics effective Amoxicillin Clavulanate
(Augmentin) , Ceftibuten, Cefuroxime,
Clarithromycin or Erythromycin estolate ( for pen
allergic pts)
 Non-Compliant pts  single dose benzathine
penicllin IM
 Recurrent Strep Throat  Cephalosporins are choice
( Keflex  cephalexin 500 bid) or can use
Augmentin
Etiologies for recurrent Streptococcal Pharyngitis
 Poor Compliance with oral medications (most common)
 Day 3: 50% stopped antibiotics
 Day 6: 70% stopped antibiotics
 Day 9: 80% stopped antibiotics
 Families reporting taking all the medication: 80%
 Repeat exposure in crowded conditions
 School , Daycare & Home or workplace
 Eradicated protective throat flora by prior antibiotic
 a-hemolytic Streptococcus is protective normal flora
 Cephalosporins apparently do less harm
 Selected beta-lactam resistance by prior antibiotic
 Consider Augmentin for 10 day course
 Suppressed Immune response from prior antibiotics
 Antibiotic Resistance
 Penicillin resistance is infrequent in strep throat
 Macrolide (Erythromycin, Biaxin, Zithromax)
 Resistance 2-8% in U.S.
 Chronic Pharyngeal Carriage of Streptococcus pyogenes
 Consider Pharyngitis due to another cause
School Attendance
ADVISE TO PARENTS!!
High Yield!
Contraindications to school attendance
 Infectious
 Fever
 Vomiting or dehydration
Indications for school return in viral infection
 Viral infection examples : Influenza, Rhinovirus (Common Cold) , Fifth
Disease, Hand Foot and Mouth Disease
 Indications to return to school  No fever and Child must practice good
hygiene (i.e. hand washing)
Indications for school return in bacterial infection
 Bacterial infection examples: Impetigo, Bacterial Conjunctivitis,
Streptococcal Pharyngitis (Strep Throat)
 Indications to return to school  after Antibiotics for 24 hours
Indications for school return in specific conditions
 Chicken Pox  All lesions have crusted over
 Head Lice  After anti-lice shampoo and manual nit removal
 Pinworm  Day after Pyrantel, Mebendazole, or Albendazole
 Vomiting  24 hours after last Emesis
Conditions allowing immediate school return
 Viral Conjunctivitis (Pink Eye)
 Otitis Media (ear infection)
LRTIs
Acute Bronchitis
 Usually viral
 Treat with antibiotics if second sickening or if
associated with COPD exacerbation
Pneumonia
 Community Acquired Pneumonia
 Typical etiology : S.pneumoniae. Others:
H.influezae, M.catarrhalis
 Atypical pneumonia : Legionella,
mycoplasma, chlamydia
 Health care associated 
- Nursing home acquired,
- Hospl acquired , Ventilator associated
Severe Pneumonia
 CURB 65 predicts highly severe pneumonia
 RR>30
 DBP<60mmhg
 BUN>20
 CONFUSION
 Age>65 yrs
Whether to admit?
 Most Pneumonias are treated as Outpatient
 Admission is required if:
 Those with underlying immunosuppression (
chemotherapy, HIV)
 Elderly patients > 65 yrs
 Pts with altered mental status
 Those with hemodynamic ( shock) or respiratory
compromise ( tachypnea, respiratory failure)
 Pts with poor social support ( homeless) or
without ability to self supervise
Where to Admit?
 Admission to ICU is needed if:
LOOK AT VITALS!
 Hypotension (SBP<90)
 Hemodynamic Instability/ Shock (map<60)
 Hypoxemia<60
 Organ failure ( ARF etc)
 Impending respiratory failure that may require
mechanical ventilation ( persistent tachypnea,
desaturation etc)
 Deteriorating comorbid illness ( CHF, renal failure
etc)
 Heart failure, severe copd exacerbation, Diabetic
complications (?DKA)
Community acqd
 Outpatient  Rx with Macrolide (
azithromycin) or newer Quinolones
 Inpatient  Rx with Ceftriaxone + macrolide
or Fluoroquniolone alone
Health Care Associated Pneumonia
 Either NH associated or hospital acquired
 NH associated pneumonia may have MRSA and
Gram –ve bacteria as etiologies ( E.coli, proteus,
klebsiella)  so emperically Rx with Vanco +Zosyn
(pip/tazo) before sputum culture results are available.
Once Cx and sensitivity are obtained d/c the
antibiotic that’s not needed
 Hospital acquired pneumonia is the one that develops
48 hrs after hospitalization  has a different
spectrum of bacteria ( MRSA + resistant gram –ves)
 initially can start VANCO + Zosyn before cx
results are available. If severe, use imipenem instead
of Zosyn (pip/tazo)
VAP
 Ventilator Acquired Pneumonia  Pneumonia that develops 48
hrs after intubation  diagnosed by c/f like fever, leucocytosis,
newly developed CXR infiltrates and purulent ET tube
secretions  the spectrum of bacteria here is more resistant i.e;
MRSA+ Resistant gram –ves including P.aeruginosa  start
emperical VANCO+Imipenem ( do not take chance with
resistance here)
 Culture ET secretions, Get a CXR
 Bronchoscopy may be required in pts showing no response and
also to differentiate b/w colonization vs. Infection  Recovery of
bacteria in high concentrations from bronchoalveolar lavage
(BAL) >10,000 col/ml helps in differentiation of non infectious
from infectious causes of pulmonary infiltrates ( i.e; if the colonies
are this high think of infection other wise think of non infectious
cause like ARDS, CHF etc for explaining these pulmonary
infiltrates in vent patients)
PCP
 Pneumocystis Carinii pneumonia  Seen in
immunocompromised pts Pts who are HIV,{CD4< 200}
Immunocompromised and pts on high dose steroids (
prednisone>20mg/d),
 Symps: dry cough, fever, chills, sob, chestpain
 Needs high suspicion for diagnosis  LDH will help when in
doubt, Gallium scan will help too
 CXR  Interstitial infiltrates, LDH high, Ground glass
appearance on CT scan, Sputum for silver staining,  if
sputum –ve, bronchoscopy needed for diagnosis where you do
Bronchoalveolar lavage – silver staining
 Get an ABG
 Rx  Simple pcp  oral bactrim
• Severe pcp  iv bactrim + steroids ( make sure u give enough
i.e; prednisone 40mg bid or solumedrol 30mg iv bid  Po2 <
70mm hg/ increased A-a > 35are indication for steroid Rx)
• Sulfa allergy  aerosolized pentamidine
Case Study
 A 36-year-old woman is admitted to the medical
intensive care unit because of respiratory depression
resulting from a barbiturate overdose. She is intubated
and mechanical ventilation is begun. Physical
examination, except for her comatose condition, is
unremarkable. Chest radiography and arterial blood
gases are within normal limits. Which of the following
will minimize her risk of developing a nosocomial
infection?
 ( A ) Ventilator tubing changes every 12 hours
 ( B ) Elevation of the head of the bed to 45 degrees
 ( C ) Ceftriaxone, intravenously
 ( D ) Oropharynx polymyxin B spray every 8 hours
 ( E ) Enteral feedings by nasogastric tube
Ans.B
 Patients who are mechanically ventilated in the
supine position have an approximately six fold
increased risk of developing pneumonia compared
with patients maintained in a semirecumbent position.
Elevation of the patient's head to 45 degrees may
reduce aspiration and nosocomial pneumonia.
 Nosocomial pneumonia is a major cause of morbidity
and mortality in mechanically ventilated patients.
Case Study
 A 21-year-old woman with cystic fibrosis diagnosed at 6 months of age is
evaluated because of increased dyspnea, blood-streaked purulent sputum,
decreased energy, and a 1.8-kg (4-lb) weight loss of 4 weeks’ duration. She
was last treated with intravenous antibiotics 12 months ago. Her sputum
cultures repeatedly grow a mucoid strain of Pseudomonas aeruginosa. Her
forced expiratory volume in 1 second (FEV1) has decreased by 400 mL in
6 months and is now 47% of predicted. Chest radiography shows diffuse
bronchiectatic changes but no consolidation. She takes replacement
pancreatic enzymes, albuterol nebulization three times daily, inhaled
recombinant human Dnase once daily,and uses a flutter device to aid
expectoration. Which of the following is the best management option at
this time?
 ( A ) Tobramycin, inhaled, twice daily
 ( B ) Increase Dnase, albuterol nebulizations, and chest physiotherapy
 ( C ) Piperacillin and tobramycin, intravenously
 ( D ) Ciprofloxacin, orally, and tobramycin, inhaled, twice daily
 ( E ) Bronchoscopy
Ans.C
 Patients with cystic fibrosis and a bronchitic
exacerbation of chronic bronchiectasis with
Pseudomonas aeruginosa require intravenous
antibiotics with two antipseudomonal agents for 2 to3
weeks.
 The use of aerosolized tobramycin is indicated for
patients with chronic Pseudomonas colonization and
is associated with long-term improvement in forced
expiratory volume in 1 sec (FEV1) of about 10%, as
well as decreased need for hospitalization and
intravenous antibiotics, but it is not sufficient for an
exacerbation.
Pulmonary Embolism
 Causes
 Clinical features  chestpain, sob, cough, leg
swelling
 EKG – Sinus tachy, S1Q3T3
 ABGs – resp alkalosis
 Diagnosis  v/q, d-dimer, high resolution CT
(Spiral CT scan) ( Serum D-dimer < 500ng/ml
 Treatment – if shock or if no shock , if
anticoagulation is contraindicated
PE on EKG
 Pulmonary embolism (acute cor pulmonale):
Look for new signs of new signs of tachycardia;
complete or incomplete RBBB; the S1Q3T3
pattern; and/or right axis shift. There may be
inferior or RV injury patterns. The most
common cause of an S1Q3T3 pattern is a
completed inferior MI.  Get a Right sided
EKG.
PE on CXR
 Initial CxR may be
NORMAL. ( PIOPED study
showed that only 12% of
CXRs in pts with
angiographically proven PE
were interpreted as normal)
 May show – Collapse,
atelectasis, consolidation,
small pleural effusion,
elevated diaphragm.
 Pleural based opacities with
convex medial margins are
also known as a Hampton's
Hump
Hampton's Hump
 Pleural based opacities with convex medial
margins are also known as a Hampton's Hump.
This may be an indication of lung infarction.
However, that rate of resolution of these
densities is the best way to judge if lung tissue
has been infarcted. Areas of pulmonary
hemorrhage and edema resolve in a few days
to one week. The density caused by an area of
infarcted lung will decrease slowly over a few
weeks to months and may leave a linear scar
PE on CXR
 Westermark sign –
Dilatation of pulmonary
vessels proximal to
embolism along with
collapse of distal
vessels, often with a
sharp cut off.
Pulmonary Embolism with
Infarction
 Consolidation
 Cavitation
 Pleural effusion (bloody
in 65%)
 No air bronchograms
 “Melting” sign of
healing
 Heals with linear scar
Case Study
 A 56-year-old man is evaluated in the emergency department because of
progressive swelling of the right lower extremity during the previous 5
days and right-sided pleuritic chest pain and dyspnea beginning 1 to 2
hours ago.On physical examination, his temperature is 38.2 °C (100.8 °F),
pulse rate is 105/min, respiration rate is 28/min, and blood pressure is
160/80 mm Hg. Cardiac and pulmonary examinations are unremarkable.
Arterial blood gases with the patient breathing room air are PO2, 78 mm
Hg; PCO2, 30 mm Hg; and pH, 7.48.Electrocardiography shows sinus
tachycardia and nonspecific ST-T wave changes, and chest radiography is
normal.Ventilation-perfusion scanning shows two unmatched segmental
defects. The D-dimer value is three times the upper limit of normal.
 Which of the following is the most appropriate course of action?
 ( A ) Heparin
 ( B ) Helical computed tomography with contrast
 ( C ) Noninvasive studies of the lower extremities
 ( D ) Pulmonary angiography
Key Point
In patients with a high pretest probability of pulmonary
embolism
and high-probability ventilation-perfusion scanning,
additional
diagnostic testing is not necessary before initiating therapy.
Pneumothorax
 Causes – Trauma, bulla rupture, necrotizing
pneumonia
 Clinical features  chest pain, dyspnea, shock
 Ventilator associated Pneumothorax ? sudden
hypotension while on vent  look at peak and
plateau pressures
 Treatment  needle thoracentesis, needle
thoracostomy, tube thoracostomy
ARDS
 Diffuse pulmonary capillary damage leading to increased
permeability of alveolar capillaries  pulm edema
 Criteria  1) There should be a cause 2) PO2/Fio2 ( in
liter) Ratio, Po2/Fio2 < 300  ALI, <200 ARDS 3)
B/L CXR infiltrates 4) Should not be due to CHF; Clues:
2D ECHO EF Good/ no diastolic dysfunction. If in doubt
whether CXR infiltrates are due to CHF or ARDS 
measure PCWP ( Swan Ganz insertion )
 Ventilation strategies  Low Vt ( 6cc/kg) (  prevent
overdistension injury) and High PEEP strategy ( reduce
derecruitment injury)
 Causes  TTP, Sepsis, Shock, Aspiration pneumonia,
chemical pneumonitis, Drugs like Heroin, Pancreatitis,
Burns, Drowning
Case Study
 A 58-year-old man is admitted to the intensive care unit with increasing dyspnea after
developing influenza symptoms 3 days previously. On physical examination, his
temperature is 39.1 °C (102.3 °F), pulse rate is 110/min, and bloodpressure is 135/83 mm
Hg. He weighs 73 kg (161 lb). He is using accessory muscles of respiration, and he has
finecrackles throughout all lung fields. Cardiac examination is unremarkable, and no
edema is noted. Chest radiographyshows diffuse infiltrates throughout both lungs with
patchy areas of consolidation. The patient has a history of moderate obstructive lung
disease secondary to smoking. Several months before hospitalization his forced expiratory
volume in 1sec (FEV1) was 53% of predicted, and he had normal oxygen saturation and no
hypercapnia.Shortly after hospitalization, he is intubated because of increasing hypoxemia
and hypercapnia. Subsequent arterial blood gases with the patient breathing 100% oxygen
and 10 cm H2O of positive end-expiratory pressure are PO2, 68mm Hg; PCO2, 65 mm Hg;
pH, 7.23; and bicarbonate, 26 meq/L. Tidal volume is 450 mL, respiration rate is
25/min,inspiratory flow rate is 100 L/min, and inspiratory/expiratory ratio is 1:5. Peak
airway and plateau ventilatory pressures are 48 cm H2O and 32 cm H2O.
 Which of the following is the best option?
 ( A ) Increase the tidal volume
 ( B ) Increase the respiration rate
 ( C ) Increase the positive end-expiratory pressure
 ( D ) Decrease the positive end-expiratory pressure
 ( E ) Administer sodium bicarbonate, intravenously
Key Points
 Ans. E
 In patients with acute respiratory distress syndrome, mortality was significantly
improved by ventilating patients with tidalvolumes of 6 mL/kg of ideal body weight
and keeping plateau ventilatory pressure at =30 cm H2O.
 If changes in respirator settings required to prevent hypercapnia have associated
untoward effects, it is reasonable to allow arterial PCO2 to rise and, if necessary,
prevent acidemia by administration of buffer  as in this case!! ( don’t increase tidal
volume here  low Vt is good for this  remember Permissive Hypercapnia)
 Increasing PEEP is not good here. Raising PEEP is undesirable because this will
narrow the pressure difference between the plateau ventilatory pressure and the
PEEP, decreasing the pressure available to deliver the tidal volume. This will reduce
the tidal volume and exacerbate hypercapnia. PEEP should remain unchanged
because the patient has acceptable oxygenation with the present setting. The level of
PEEP cannot be reduced since reduction likely will lead to unacceptable hypoxemia.
The patient is barely at an acceptable level without any reduction.
 Increasing the respiration rate likely will increase auto-positive end-expiratory
pressure (PEEP) in this patient with chronic obstructive pulmonary disease ( they
have proloned expiration!) by “breath stacking,” that is, delivering the next breath
before the previous breath is completely expired.This will also raise the plateau
ventilatory pressure above a desirable range.
Case Study
 A 57-year-old man with severe chronic obstructive pulmonary
disease is hospitalized with respiratory distress of 12 hours’
duration. Arterial blood gases with the patient breathing 35%
oxygen through a face mask are PaO2, 50 mm Hg; PaCO2, 70
mm Hg; and pH, 7.24. When seen as an outpatient 1 month
previously, his arterial blood gases while breathing room air
were PaO2, 58 mm Hg; PaCO2, 50 mm Hg; and pH, 7.37.
Despite maximal therapy, mechanical ventilation is required.
During controlled breaths, his peak airway pressure is 25 cm
H2O, and plateau ventilatory pressure is 12 cm H2O. The
arterial blood gases are checked after 1 hour. Which of the
following is the most desirable set of arterial blood gas
values?
 ( A ) Pa O2, 50 mm Hg; PaCO2, 45 mm Hg; pH, 7.44; FIO2, 0.3
 ( B ) Pa O2, 65 mm Hg; PaCO2, 52 mm Hg; pH, 7.38; FIO2, 0.4
 ( C ) Pa O2, 65 mm Hg; PaCO2, 40 mm Hg; pH, 7.48; FIO2, 0.4
 ( D ) Pa O2, 90 mm Hg; PaCO2, 60 mm Hg; pH, 7.32; FIO2, 0.5
 ( E ) Pa O2, 133 mm Hg; PaCO2, 55 mm Hg; pH, 7.41; FIO2,
0.6
Ans.B
 When instituting mechanical ventilation in a patient with
chronic hypercapnia, it is critical to avoid the development of
respiratory alkalemia secondary to overventilation, and
ventilator settings should have pH as a target, rather than
PaCO2.
 When seen 1 month before hospitalization, the patient had
chronic carbon dioxide retention. When instituting mechanical
ventilation in a patient with hypercapnia, it is critical to avoid
the development of respiratory alkalemia secondary to
overventilation. Severe alkalosis in this setting may result in
cardiovascular instability, arrhythmias, andseizures. Ventilator
settings should have pH as a target, rather than PaCO2.
Acute Pulmonary Edema
 Treatment  morphine, loop diuretics in LVF,
Ventilation strategies in ARDS and
Hemodialysis when indicated
 Causes  ARDS, Acute LVF, Fluid Overload,
Missing Hemodialysis
 A 58-year-old man is admitted to the intensive care unit with increasing dyspnea after developing influenza
symptoms 3 days previously. On physical examination, his temperature is 39.1 °C (102.3 °F), pulse rate is
110/min, and blood pressure is 135/83 mm Hg. He weighs 73 kg (161 lb). He is using accessory muscles of
respiration, and he has fine crackles throughout all lung fields. Cardiac examination is unremarkable, and
no edema is noted. Chest radiography shows diffuse infiltrates throughout both lungs with patchy areas of
consolidation. The patient has a history of moderate obstructive lung disease secondary to smoking. Several
months before hospitalization his forced expiratory volume in 1 sec (FEV1) was 53% of predicted, and he
had normal oxygen saturation and no hypercapnia. Shortly after hospitalization, he is intubated because of
increasing hypoxemia and hypercapnia. Subsequent arterial blood gases with the patient breathing 100%
oxygen and 10 cm H2O of positive end-expiratory pressure are PO2, 68 mm Hg; PCO2, 65 mm Hg; pH,
7.23; and bicarbonate, 26 meq/L. Tidal volume is 450 mL, respiration rate is 25/min, inspiratory flow rate is
100 L/min, and inspiratory/expiratory ratio is 1:5. Peak airway and plateau ventilatory pressures are
 48 cm H2O and 32 cm H2O.
 Which of the following is the best option for improving this patient’s acid–base disorder?
 ( A ) Increase the tidal volume
 ( B ) Increase the respiration rate
 ( C ) Increase the positive end-expiratory pressure
 ( D ) Decrease the positive end-expiratory pressure
 ( E ) Administer sodium bicarbonate, intravenously
Ans.
 In patients with acute respiratory distress syndrome,
mortality was significantly improved by ventilating
patients with tidalvolumes of 6 mL/kg of ideal body
weight and keeping plateau ventilatory pressure at
=30 cm H2O.
 If changes in respirator settings required to prevent
hypercapnia have associated untoward effects, it is
reasonable to allowarterial PCO2 to rise and, if
necessary, prevent acidemia byadministration of
buffer.
COPD
COPD – Screening with Spirometry
 Consider screening smokers or former smokers with
certain clinical characteristics for COPD with pulmonary
function testing.
 In patients who smoke or have smoked, consider obtaining
screening spirometry readings to document obstruction if they
give a history of cough or sputum production or have findings
compatible with emphysema on chest x-ray.
 Obtain spirometry readings if the patient has limiting
symptoms such as dyspnea inappropriate to the level of
activity, frequent episodes of acute bronchitis related to upper
respiratory tract infections (i.e., a possible acute exacerbation),
difficulty sleeping due to cough and dyspnea, and general
diminished activity levels and energy from difficulty in
breathing.
 If the patient has no other clinical characteristics for COPD,
but has a significant history of smoking, consider obtaining
spirometry readings because significant pulmonary function
impairment may still be present.
COPD Exacerbations
 COPD – Chr.bronchitis, Emphysema – blue bloaters, Pink puffers
 COPD exacerbations  History, Clinical exam, get pulse ox,
 Mild, Moderate, Severe  classify depending on 3 criteria (Increase in
amount of sputum, Increased sputum purulence, worsening dyspnea)
 Mild exacerbation ( 1 of above criteria)  use simple antibiotics like Bactrim
or Doxycycline
 Moderate exacerbation ( 2 of above criteria) use 2nd line Antibiotics like
quinolones, b-lactam/clavulanate ( Augmentin)
 Severe Exacerbation ( 3 of above criteria)  Look at the ABGs, o2
inhalation, nebulizer with ipratropium + albuterol  caution with o2, o2
inhalation only as much as to maintain sao2>90%.  If no response , non
invasive ventilation ( positive pressure ventilation, BIPAP)  Pt must be
cooperative for this  if altered mental status, no response with non invasive
ventilation  Intubate and ventilate.
 Remember to get ABGs after u place a COPD guy on oxygen
 Beware of posthypercapnic alkalosis  if develops, acetazolomide
 COPD exacerbation  ? Ask urself secondary to what  Acute bronchitis,
pneumonia  use of antibiotics in COPD exacerbations
 Steroids is a MUST  methylprednisolone high doses 125mg q6hrs, then
tapering steroids
When To Admit?
Indications for hospitalization of patients with COPD:
 Patient has acute exacerbation plus one or more of the following:
 Inadequate response of symptoms to outpatient management
 Inability to walk between rooms (patient previously mobile)
 Inability to eat or sleep due to dyspnea
 Conclusion by family, physician, or both that patient cannot manage at home and
supplementary home care resources are not immediately available
 Presence of a high-risk comorbid condition, pulmonary (e.g., pneumonia) or
nonpulmonary
 Prolonged, progressive symptoms before emergency department visit
 Altered mentation
 Worsening hypoxemia
 New or worsening hypercarbia
 Patient has new or worsening cor pulmonale unresponsive to outpatient
management
 A planned invasive surgical or diagnostic procedure requires analgesics or
sedatives that may worsen pulmonary function
 Comorbid conditions (e.g., steroid myopathy or vertebral compression
fractures) have worsened pulmonary function
Where To Admit?
 Admit patients with COPD to an intensive care
unit if they meet specific criteria.
 Confusion, lethargy, or respiratory muscle fatigue
 Persistent or worsening hypoxemia despite supplemental
O2 or severe or worsening of respiratory acidosis (pH
7.30); use of supplemental oxygen should be at the lowest
flow rate to raise PaO 2 >60 or SaO 2 >90% to avoid
hyperoxic hypercapnia
 Need for assisted mechanical ventilation, whether through
means of tracheal intubation or noninvasive techniques
 Severe dyspnea that responds inadequately to initial
emergency room therapy
COPD – Home Oxygen Therapy
 At discharge, evaluate pt for home 02 therapy.
Especially at nights when pts may desaturate (
acidosis at nights shifts curve to right). Goal
maintain sao2 90 or po2 60
 Indications :
 Po2<55 or sao2 <85%
 Po2 b/w 56 to 59 if corpulmonale or
polycythemia ( erythrocytosis) (  these
suggest evidence of hypoxia)
Lung Volume Reduction Surgery
 Consider LVRS for patients whose initial clinical criteria
include:
 CT scan evidence of bilateral emphysema
 Prerehabilitation postbronchodilator TLC and residual volume >/= to
100% and 150% predicted, respectively
 Maximum FEV1 </= 45% predicted
 PaCO2 </= 60 mm Hg
 PaO2 >/= 45 mm Hg
 Completion of a pulmonary rehabilitation program
 Do not consider LVRS for patients whose clinical criteria
include:
 FEV1 less than or equal to 20% predicted ( very low for surgery) and
either homogenous emphysema or carbon monoxide diffusing capacity
less than or equal to 20% predicted (DLCO)
 Non-upper-lobe emphysema and high baseline exercise capacity
Interpretation of PFT’S
 Restrictive vs. Obstructive
 FEV1 to FVC Ratio (Normally over 75%)
 Not useful if both FEV1 and FVC are normal
 Obstructive lung: Moderately to severely decreased
 Restrictive lung: Normal or increased
Reversibility:
 Bronchodilator response (Significant values)
 Response suggests reversible component if
 FVC or FEV1 improves by 12 to 15% over baseline
 FVC or FEV1 increases by at least 200 ml
 FEF25-75 improves by 15 to 25% over baseline
COPD Outpatient Rx
 By MDIs  Ipratropium all the time ( q6hrs)
+ albuterol as needed. Can use tiotropium
because its long acting
 Evaluate for home o2 therapy
 Steroids/ antibiotics in acute exacerbations
only.  ( unlike in Asthma, steroids are not a
part of chronic therapy in COPD)
 MDIs deliver only fixed dose of drug.
Nebulizers deliver larger dose of drug so in
exacerbation u start with nebulizer if MDIs
don’t work
COPD with Asthma
 Asthma may be present in about 10% of cases
of COPD; however, reversibility of FEV1
alone should never be used to make a
diagnosis of asthma in the absence of other
supporting evidence such as a childhood
history of asthma, atopic symptoms, blood or
sputum eosinophilia, or onset of symptoms
before substantial history of cigarette smoking
COPD in the Young
 A 38-year-old man is evaluated because of a morning cough productive of clear sputum, chest
tightness, and shortness of breath when walking. He has smoked two packs of cigarettes per
day since his teenage years and says that previous chest radiography showed "early
emphysema." He is a baker but notes no improvement in symptoms when on vacation. His
wife has three indoor cats, and he has an outdoor dog. The patient has normal vital signs. The
chest is hyperresonant to percussion, breath sounds are decreased in intensity, and expiration
is prolonged. Pulmonary function tests show forced expiratory volume in 1 sec (FEV1) is
45% of predicted, forced vital capacity (FVC) is 65% of predicted, total lung capacity (TLC)
is slightly increased (120% of predicted), and diffusing lung capacity for carbon monoxide
(DLCO) is moderately reduced (60% of predicted). Chest radiography shows hyperinflation
with a suggestion of several small bullae in the lower lung fields.
 Which of the following tests is indicated?
 ( A ) Sputum Gram stain and culture
 ( B ) Methacholine inhalation challenge test
 ( C ) Skin tests for allergens and serum precipitins to wheat extract
 ( D ) Measurement of serum a 1-antitrypsin level
 ( E ) Esophageal pH monitoring for 24 hours
 Severe chronic obstructive pulmonary disease
in young persons is suggestive of a1-
antitrypsin deficiency, and an a1-antitrypsin
level should be measured.
 Smoking is an important precipitating factor
and also increases progression
Case Study
 A 67-year-old man with longstanding chronic obstructive pulmonary disease
(COPD) is hospitalized with a 1-week history of increasing cough productive
of large amounts of purulent sputum, low-grade fever, lethargy, and shortness
ofbreath.On physical examination, his vital signs are normal except for a
temperature of 38.2 °C (100.7 °F) and a pulse rate of 108/min. The neck veins
are not distended. The anterior–posterior chest dimension is increased and is
hyperresonant to percussion, breath sounds are reduced, and expiration is
prolonged.Arterial blood gases are normal except for a PO2 of 62 mm Hg with
the patient breathing 28% oxygen through a venturi mask. Chest radiography
shows changes compatible with COPD but no acute process.In the emergency
department, treatment with inhaled bronchodilators and antibiotics was begun.
 Which of the following options is the best choice?
 ( A ) Add inhaled fluticasone, every 12 hours
 ( B ) Add methylprednisolone, 500 mg intravenously once
 ( C ) Add methylprednisolone, 125 mg intravenously every 6 hours for 3 days,
then taper over 2 weeks
 (D) No need to add steroids in this patient
 E) Intubate the patient
Key Point
Patients with exacerbations of chronic obstructive
pulmonary
disease (COPD) who receive intravenous corticosteroids
and a
tapering dose of prednisone over 2 weeks experience
shorter
hospitalization and less treatment failures.
Two weeks of tapering prednisone is just as effective as 8
weeks
in treating exacerbations of COPD.
 A 57-year-old man with severe chronic obstructive pulmonary disease is
hospitalized with respiratory distress of 12 hours’ duration. Arterial blood gases
with the patient breathing 35% oxygen through a face mask are PaO2, 50 mm Hg;
 PaCO2, 70 mm Hg; and pH, 7.24. When seen as an outpatient 1 month previously,
his arterial blood gases while
 breathing room air were PaO2, 58 mm Hg; PaCO2, 50 mm Hg; and pH, 7.37.
Despite maximal therapy, mechanical
 ventilation is required. During controlled breaths, his peak airway pressure is 25 cm
H2O, and plateau ventilatory
 pressure is 12 cm H2O. The arterial blood gases are checked after 1 hour.
 Which of the following is the most desirable set of arterial blood gas values?
 ( A ) Pa O2, 50 mm Hg; PaCO2, 45 mm Hg; pH, 7.44; FIO2, 0.3
 ( B ) Pa O2, 65 mm Hg; PaCO2, 52 mm Hg; pH, 7.38; FIO2, 0.4
 ( C ) Pa O2, 65 mm Hg; PaCO2, 40 mm Hg; pH, 7.48; FIO2, 0.4
 ( D ) Pa O2, 90 mm Hg; PaCO2, 60 mm Hg; pH, 7.32; FIO2, 0.5
 ( E ) Pa O2, 133 mm Hg; PaCO2, 55 mm Hg; pH, 7.41; FIO2, 0.6
Q
 65 Y/O comes with cough and exertional sob of several month duration. He has smoked for
35 years. On physical examination, he is sweating, ruddy, and cyanotic. His pulse rate
is120/min and regular, respiration rate is 30/min and labored, and blood pressure is 150/90
mm Hg. The neck veins are distended to the angle of the jaw when sitting upright. The chest
shows hyperinflation, prolonged expiration, wheezing, and crackles at each posterior base.
The pulmonic sound is increased, and there is a summation gallop. An enlargedand tender
liver edge is felt 2 cm below the costal margin. He has marked dependent edema up to the
knees.The hematocrit is 55%, and leukocyte count is 8000/μL. Arterial blood gases with the
patient breathing room air arePaO2, 47 mm Hg; PaCO2, 50 mm Hg; and pH, 7.30. Spirometry
performed 2 years earlier showed a forced expiratoryvolume in 1 sec (FEV1) of 0.65 L and a
forced vital capacity (FVC) of 3.05 L. Chest radiography shows hyperinflation, clear lung
fields, and biventricular enlargement. Ventilation-perfusion lung scanning shows multiple
matched fillingdefects that are not segmental. Doppler studies of the legs are negative.After
treatment of the patient’s acute condition, which of the following is the best long-term
therapy for
 this patient?
 ( A ) Nifedipine
 ( B ) Warfarin
 ( C ) Bosentan
 ( D ) Oxygen
 ( E ) Phlebotomy
Case Study
 A 65-year old male hospital in-patient has smoked cigarettes since he was 18 years
old. He has a chronic cough and marked sputum production. When his doctor starts
to give him the usual talk about losing weight, he explains that since he has about
fifty pounds to lose, he has tried to exercise, but is unable to because of shortness of
breath with any activity. Upon further questioning, he comments that his symptoms
have been present for a very long time, but he was hospitalized due to a marked
exacerbation of his complaints. On auscultation, rhonchi and wheezes are heard.His
laboratory results are as follows:
 pCO2 60 mm Hg(35-45 mm Hg)
 pH 7.34( 7.35-7.45)
 bicarbonate 31 mEq/L( 24 mEq/L)
 Na+ 140 mEq/L( 135-145 mEq/L)
 K+ 4.0 mEq/L( 3.5-5.5 mEq/L)
 Cl-100 mEq/L(98-109 mEq/L)
 What is the primary disorder?
a) metabolic acidosis with a normal anion gap
b) metabolic acidosis with an elevated anion gap
c) metabolic alkalosis
d) respiratory acidosis
e) respiratory alkalosis
Ans.D
 This patient has symptoms and signs of chronic obstructive pulmonary
disease, specifically chronic bronchitis.
Symptoms and signs include cough, sputum production and dyspnea with
exertions. Patients tend to be stocky or overweight, as the case here.
Auscultation will reveal wheezes and rhonchi. This patient is retaining
CO2, since his pCO2 is elevated. CO2 is in equilibrium with carbonic acid.
An increase in CO2 will shift the Henderson Hasselbalch equation to the
left, resulting in acidosis. Since the cause of the primary problem is
respiratory, e.g. retention of CO2, this is a
respiratory acidosis.
This is reflected in the pH being reduced as well.Metabolic acidosis (choice
a, choice b) is incorrect because the primary problem is not due to a
administration of acid, excess metabolic acid formation, or loss of base.
Although the bicarbonate level is abnormal in this patient, that is due to
metabolic compensation for the respiratory acidosis.
Alkalosis (choice c, choice e) are incorrect because his pH is acidotic.
Although compensatory mechanisms can bring the pH towards the normal
range, compensatory mechanisms will never overshoot.
 A 54-year-old man is hospitalized because of severe shortness of breath, ankle
swelling, and confusion of 5 days’ duration. He has smoked for 35 years. On
physical examination, he is sweating, ruddy, and cyanotic. His pulse rate is 120/min
and regular, respiration rate is 30/min and labored, and blood pressure is 150/90
mm Hg. The neck veins are distended to the angle of the jaw when sitting upright.
The chest shows hyperinflation, prolonged expiration, wheezing, and crackles at
each posterior base. The pulmonic sound is increased, and there is a summation
gallop. An enlarged and tender liver edge is felt 2 cm below the costal margin. He
has marked dependent edema up to the knees. The hematocrit is 55%, and
leukocyte count is 8000/μL. Arterial blood gases with the patient breathing room air
are PaO2, 47 mm Hg; PaCO2, 50 mm Hg; and pH, 7.30. Spirometry performed 2
years earlier showed a forced expiratory volume in 1 sec (FEV1) of 0.65 L and a
forced vital capacity (FVC) of 3.05 L. Chest radiography shows hyperinflation,
clear lung fields, and biventricular enlargement. Ventilation-perfusion lung
scanning shows multiple matched filling defects that are not segmental. Doppler
studies of the legs are negative. After treatment of the patient’s acute condition,
which of the following is the best long-term therapy for this patient?
 ( A ) Nifedipine
 ( B ) Warfarin
 ( C ) Bosentan
 ( D ) Oxygen
 ( E ) Phlebotomy
Ans. D
 In patients with cor pulmonale caused by
chronic hypoxemia, oxygen therapy is the
treatment of choice; it may decrease the heart
failure and polycythemia seen in this
condition.
Asthma
 Classification: Management Grouping
 Mild Intermittent Asthma
 Occasional exacerbations (Less than twice per week)
 Mild Persistent Asthma
 Frequent exacerbations (>twice weekly, but not daily)
 Moderate Persistent Asthma
 Daily symptoms with daily Beta Agonist use
 Severe Persistent Asthma
 Continuous Symptoms and frequent exacerbations
 Treatment  short acting MDIs as needed, long acting bronchodilators (
once asthma becomes moderate to severe add these as adjuncts to inhaled
steroids), inhaled steroids ( first line agent in all persistent asthmas) ,
systemic steroids, monteleukast ( add this as adjunct in moderate to severe
asthma)
 Status asthmaticus
Asthma
Examples of different therapeutic approaches:
 Mild Intermittent: use only prn albuterol; if related to exercise,
use albuterol one-half hour prior to exercise; also used:
cromolyn one half-hour prior to exercise.
 Mild Persistent: daily: low dose inhaled steroids; and use
albuterol intermittently as needed. May use inhaled cromolyn.
 Moderate Persistent: use peak flow meter daily; use med dose
inhaled steroid or low dose steroids plus serevent or
singulair. Others switch to Advair. PO steroids prn.
 Severe Persistent: use peak flow meter daily; po steroids as
needed. Daily meds to include high dose inhaled steroids,
singulair, serevent or possibly Advair.
Classification:
Mild
Intermitt
ent
Mild
Persist
ent
Mod
Persist
ent
Severe Persistent
Sym
ptom
s
< 2 /
week
> 2 /
week
Daily Continual
Night
sx
< 2 /
month
> 2 /
month
> 1 /
week
Frequent
FEV
1
> 80%
predicte
d
>
80%pr
edicted
60-
80%
< 60%
Peak
flow
Varia
bility
< 20%
20-
30%
> 30 % > 30%
Management of different categories of
Asthma:Mild
Intermitte
nt
Mild
Persistent
Mod
Persistent
Severe Persistent
Peak Flow Meter X X
Preventive Meds (daily):
--Inhaled Cromolyn X (X)
--Inhaled Steroids
X (low
dose)
X (med
dose)
X (high dose)
--Po Singulair X X X
--Advair Diskus (
Salmetrol/
Fluticasone)
X X
--Serevent (
Salmetrol)
X X
Rescue Medications
--Albuterol X X X X
--Po Prednisone prn Prn Prn prn
Skin Testing:
Allergy Evaluation
Consider Consider Consider
Case Study
 A 68-year-old man with asthma is evaluated because he needs
to use his albuterol inhaler at night once or twice a week after
waking up with chest tightness. His forced expiratory volume
(FEV) is 2.18 L (65% of predicted) before and 2.62 L(82% of
predicted) after inhaled albuterol. Current medications include
inhaled fluticasone, 440 μg twice daily, and an albuterol
metered-dose inhaler as needed. Which of the following
should be done next to better control his symptoms?
 ( A ) Increase fluticasone to 880 μg twice daily
 ( B ) Add salmeterol
 ( C ) Add prednisone
 ( D ) Add allergen immunotherapy
 ( E ) Add a long-acting theophylline at bedtime
Key Point
In patients with moderate-to-severe asthma not responding to
adequate doses of a short-acting ß-agonist and inhaled
corticosteroids, the next step is addition of a long-acting
ß-agonist.
Case Study
 A 25-year-old woman is evaluated because of a 3-year history of a
nonproductive cough. The cough is aggravated by bicycle riding and
occasionally awakens her from sleep. During the past year, she experienced
two episodes of bronchitis followed by a dry cough persisting for 2 months.
The cough worsened when she visited her sister in Alaska. She has
seasonal symptoms of watery, runny nose and sneezing. There is no
postnasal discharge, nasal congestion, heartburn, weight loss, or night
sweats. She does not smoke. Her physical examination and chest
radiography are normal. Spirometry shows forced expiratory volume in 1
sec (FEV1) 3.29 L; forced vital capacity (FVC), 4.13 L; and FEV1/FVC
ratio of 79%. Which of the following is the best next management step?
 ( A ) Chest computed tomography
 ( B ) Bronchoscopy
 ( C ) Methacholine inhalation challenge testing
 ( D ) Observation and reassurance
 ( E ) Therapeutic trial of a proton pump inhibitor
Cough Variant Asthma
Cough-variant asthma is nonproductive, provoked by
exercise
and cold air, disturbs sleep, and worsens after a lower
respiratory tract infection.
The inhalation of methacholine produces airway obstruction
in
most patients with asthma; less than 10% of normal persons
have positive responses ( false +ves) .
Case Study
 A 45-year-old woman is evaluated because of dyspnea during exercise that
began when she started an aerobics class. She has dyspnea, chest tightness,
and a nonproductive cough after 15 minutes of vigorous step exercises. The
symptoms worsen slightly when she stops, then gradually abate. She has a
5-pack-year smoking history but quit 10 years ago. The physical
examination, chest radiography, and electrocardiography are all normal.
Spirometry shows forced expiratory volume in 1 sec (FEV1), 2.72 L (83%
of predicted); forced vital capacity (FVC), 3.2 L(86% of predicted); and
FEV1/FVC ratio of 85%. Postexercise spirometry shows FEV1, 2.04 L
(25% drop from baseline),and FVC, 3.00 L (2% drop from baseline).
Which of the following management options should be done next?
 ( A ) Reassure the patient
 ( B ) Prescribe an albuterol inhaler 15 minutes before exercise
 ( C ) Perform an exercise stress test
 ( D ) Measure lung volumes and diffusing capacity
 ( E ) Perform high-resolution computed tomography of the chest
Ans.b
 For patients with exercise-induced asthma, an
inhaled ß-agonist should be prescribed before
exercise.
Case Study
 WHEN CONSIDERING THE DRUG TREATMENT
OF ASTHMA WITH INHALER DEVICES
A. The incidence of oral candidiasis is increased by
the use of spacer devices.
B. Salmeterol is indicated for p.r.n. usage.
C. Intermittent terbutalin has been shown to lead to
long term worsening of asthma.
D. Steroid dosage of 600mg daily has been shown to
be associated with adrenal suppression in adults.
E. Sodium cromoglycate is of no proven value in
treating acute asthmatic attacks
Ans.E
 Spacer devices decrease the incidence of oral
candidiasis by preventing the deposition in the mouth.
Salmeterol is a long acting beta antagonist, its action
is slow in onset and therefore it should be given
regularly rather than p.r.n. The Committee on Safety
of Medicines has reported that salbutamol and
terbutaline have not been shown to lead to a
worsening of mild asthma. In adults an inhaled
dosage of steroid of 1,500 micrograms daily is
associated with adrenal suppression . Sodium
cromoglycate is of no value in and acute attack and is
only indicated for prophylaxsis.
OSA
 Check for symptoms of excessive daytime sleepiness
 Diagnose by sleep study.
 Obesity – neck circumference > 17cm important
predictor.
 Check local anatomy, Throat crowding, secondary
factors (thyroid, cushings) causing obesity.
 Obesity Hypoventilation syndrome
 Rx – c-pap at nights
Q
 A 43-year-old man is evaluated because of uncontrolled hypertension,
documented in and outside of the office, despite moderate doses of
hydrochlorothiazide and enalapril. For the past 6 months he has noted
increased fatigue and irritability that he attributes to personal problems at
work. He admits to difficulty concentrating at work. He has two beers
before bed to fall asleep. He is a salesman, smokes one pack of cigarettes
per day, and lives alone. His neck circumference is 17.5 in. On physical
examination, he has a ruddy complexion, body mass index is 32, and blood
pressure is 158/88 mm Hg. Jugular venous distention cannot be evaluated
because of obesity. An S4 is present. The remainder of the examination is
normal. A complete blood count, serum electrolytes, serum creatinine,
blood urea nitrogen, electrocardiography, and chest radiography are
normal.
 Which of the following is most likely to establish a diagnosis?
 ( A ) Ambulatory blood pressure monitoring
 ( B ) Pulmonary function studies
 ( C ) Polysomnography
 ( D ) Arterial blood gases and blood volume determination
Q2
 A 58-year-old man is evaluated because of daytime sleepiness. He is
requesting an evaluation at this time because last week he fell asleep while
driving and had a minor accident. He is a lifetime nonsmoker and is
otherwise healthy. On physical examination, his body mass index is 26.
There are no obvious abnormalities of his oropharynx. Chest and cardiac
examinations are normal. There is no peripheral edema. Chest radiography
and electrocardiography are normal. Overnight polysomnography for 6
hours of sleep shows 60 episodes of apnea (cessation of airflow for more
than 10 seconds) per hour accompanied by frequent oxygen desaturation
below 85%. There is evidence of rib cage and abdominal motion during the
apneic periods. Which of the following is the most appropriate form of
therapy for this patient?
 ( A ) Nasal continuous positive-airway pressure
 ( B ) Uvulopalatopharyngoplasty
 ( C ) Progesterone
 ( D ) Mandibular repositioning device
 ( E ) Nocturnal supplemental oxygen by nasal cannula
Ans. A
 Nasal continuous positive-airway pressure (CPAP) is the
standard initial treatment for patients with symptomatic
moderate-to-severe obstructive sleep apnea syndrome. It
works by splinting the upper airway in an open position.
 Surgical procedures such as uvulopalatopharyngoplasty
(UPPP) and tracheostomy are best considered in severely ill
patients for whom more conservative measures such as nasal
CPAP are ineffective.
 Although tracheostomy cures obstructive sleep apnea, it
carries associated complications and is poorly accepted by
patients. UPPP, whether performed with conventional or laser
surgery, has variable long-term results and also has associated
complications.
Restrictive Diseases - Sarcoidosis
Pathophysiology
 Noncaseating granuloma formation
 Idiopathic  underlying genetic predisposition
 Precipitated by trigger  Infection (e.g. Mycobacteria, Borrelia Burgdorferi),
Environmental exposure (e.g. Beryllium, Aluminum)
 Common involvement sites (affects all organ systems)
 Lungs (>90%): Interstitial Lung Disease
 Lymphadenopathy: Hilar adnenopathy (>95%)
 Liver (50-80%) : Hepatic Granulomas (86%), Increased Alkaline Phosphatase
 Spleen
 Skin lesions (25%)
 Eyes : Anterior Uveitis ( also in Spondyloarthropathy) , Posterior Uveitis,
 Heart (5%)
 Tachyarrhythmias
 Cardiomyopathy
 ERYTHEMA NODOSUM SUGGESTS BETTER PROGNOSIS
 Associated with acute arthritis (Lofgren's Syndrome)
- Not associated with chronic arthritis
 Most common associated nonspecific skin sign
Sarcoidosis
Diagnosis
 Pulmonary Function Testing
 Findings consistent with Interstitial Lung Disease
 Serum Angiotensin-converting enzyme (Serum ACE)
 Increased in 50-80% of Sarcoidosis patients
 Biopsy or Cytology (Gold standard)
 Finding
 Discrete noncaseating epithelioid granuloma
 Biopsy sites
 Transbronchial lung biopsy (preferred site)
 Bronchoalveolar lavage (CD4-CD8 ratio >3.5)
 Skin biopsy of lesion
 Palpable peripheral lymph node biopsy
 Salivary Gland biopsy
Sarcoid - lung
Radiology: Chest XRay (abnormal in 90% of cases)
 Type 0: No abnormality (<10% of cases)
 Type I: Lymphadenopathy alone (43% of cases)
 Bilateral hilar Lymphadenopathy
 Mediastinal Lymphadenopathy
 Right paratracheal Lymphadenopathy
 Type II: Adenopathy and Infiltrates (24% of cases)
 Lymphadenopathy as in Type I Chest XRay findings
 Parenchymal infiltrates
 Symptomatic respiratory disease presentation
 Type III: Infiltrates alone (13% of cases)
 Parenchymal infiltrates
Sarcoidosis - Treatment
 Cutaneous  for Erythema nodosum use
NSAIDS, For sarcoid lesions use intralesional
Corticosteroids
 For uveitis  topical CS, Systemic CS if
refractory
 For pulmonary sarcoidosis stage 2 or 3 
Indications : Dyspnea , Persistent cough and
Widespread debilitating disease  systemic CS/
AZA
Sarcoidosis - Prognosis
Prognosis
 Overall mortality (from respiratory failure): 1-5%
 Factors suggestive of worse prognosis
 Onset after age 40 years
 Black race
 Chronic Hypercalcemia
 Specific higher risk organ involvement
 Neurologic involvement
 Cardiac involvement
 Eye involvement (Chronic Uveitis)
 Renal involvement (Nephrocalcinosis)
 Cystic bone lesions
 Progressive pulmonary fibrosis
Idiopathic Pulmonary Fibrosis
 CXR – Picture
 Physical – chest bibasal creps
 HRCT confirmatory
 Not responsive to steroids
TB - Screening
 Screen for LTBI in persons at increased risk of recent infection, including
immigrants within the last 5 years from high prevalence countries; pre- and
postexposure in travelers visiting countries with a high prevalence of TB;
those in recent contact with a case of infectious TB; health care workers
with potential exposure to mycobacteria; and residents and employees of
high-risk congregate settings where local epidemiology indicates a high
rate of TB.
 Screen for LTBI in persons with conditions associated with an increased
risk of developing active TB, including HIV infection, diabetes, silicosis or
exposure to silica dust, low body weight, chronic renal failure or
hemodialysis, gastrectomy, jejunoileal bypass, cirrhosis of the liver, organ
transplantation, anticancer chemotherapy and other immunosuppression
(e.g., TNF-α antagonists), malignant head or neck carcinoma, or fibrotic
changes on CXR film compatible with previous TB.
 Screen children and adolescents for LTBI who have risk factors for
development of active disease (e.g., HIV), have been exposed to adults at
high risk for TB, or have been adopted from abroad, especially if they were
born in countries with endemic TB.
TB - Screening
 PPD test is used for screening and its sensitivity
approaches 100% in pts with normal immunity.
 As an alternative to the PPD, consider using a whole-
blood IFN-γ assay such as QuantiFERON-TB Gold,
recognizing its limitations in children and
immunocompromised patients and that all currently
available studies on the sensitivity and specificity of
this test are limited by the lack of an eternal “gold
standard” for the diagnosis of tuberculous infection,
but that there is good reason to believe that the
QuantiFERON-TB Gold test is superior to the TST in
BCG-vaccinated individuals because it employs the
ESAT-6 antigen that is lacking in BCG
TB and PPD
Interpretation: PPD under 5 mm
 Negative
 Observe Patient
Interpretation: PPD 5 mm or greater
 Positive if
 HIV Infection
 Tuberculosis contact
 Immunosuppressed (e.g. HIV, Prednisone >15 mg qd)
 Abnormal finding on Chest XRay
 Management
 Chest XRay and exam for disseminated disease
 If cxr –ve INH for 9 mos
TB and PPD
Interpretation: PPD 10 mm or greater
 Positive if
 Health care workers
 New immigrant within last 5 years
 Intravenous Drug Abuse
 Homeless
 Under 4 years old
 Malnutrition
 Diabetes Mellitus
 Silicosis
 Tuberculosis endemic to region
 Management
 Chest XRay and exam for disseminated disease
TB and PPD
Interpretation: PPD 15 mm or greater
 Positive in all persons
 Management
 Chest XRay and exam for disseminated disease
 INH 9 mos if no active disease ( i.e; you are treating
Latent TB)
 If active disease  First step Sputum for AFB smear x
3, Sputum for AFB cx and Sensitivity; isolate the
patient, isolate organism for susceptibility testing 
start emperic multi drug regimens HRZE
 For failure/ resistant TB  SHRZE
 Sputum –ve pts can be taken off Isolation.
Latent TB – Imp Points
 In case of patients who are TB contacts:
 Do PPD test and if –ve repeat another after 8-12 wks
 Begin latent TB therapy in contacts such as children
and patients with HIV even if the initial skin test is
negative.
 If the second test is also negative, stop medication in
immunocompetent individuals. If a known high-risk
TB exposure has occurred in a patient with HIV
infection, continue LTBI treatment for the full period,
regardless of TST results.
Tuberculosis & PPD – Imp Points
 PPD skin test
 Next step if ppd +ve ( Latent TB)
 INH rx
 When screening for LTBI in TB contacts, if the initial
PPD result is negative a second PPD should be done
8 to 12 weeks after the last known exposure ( In a
review of literature, several studies support a
maximum interval of 8 weeks from initial infection to
development of a delayed-type hypersensitivity
reaction. A Tuberculin Skin Test > 8 weeks from last
exposure is recommended. )
Tuberculosis & PPD – Imp Points
 Do not do a Tuberculin Skin Test on any patient
 with a history of severe blistering reactions
 with previously documented active TB, With a
history of treatment for TB, With a documented
previous positive TST result  because the TST
remains positive in most of these patients
 Do not use TST to see if it turns –ve to monitor
the success of your treatment  that wont happen
 TST remains +ve even after therapy.
Tuberculosis & PPD – Imp Points
 Delay the PPD Test for 4 to 6 weeks after a
major viral illness such as measles, mumps,
rubella, or influenza, because cutaneous anergy
can develop, leading to a false-negative TST
result.
 Do periodic serial PPD screening in persons with
ongoing exposure to TB ( Health Care Workers,
Residents)  q1year is good enough!
Chronic Cough
Common Causes of Chronic Cough
 Smoker’s cough
 Chronic bronchitis due to smoking
 Post-nasal drip
 Post-infectious Viral Bacterial—Bordetella
pertussis, Mycoplasma, Chlamydia (
tracheobronchitis)
 Gastroesophageal reflux disease
 Asthma
 Angiotensin converting enzyme inhibitors
Less Common Causes of Chronic
Cough
 Infectious causes
Tuberculosis—typical or atypical,
 Fungal
 Endobronchial lesions
 Benign — bronchial adenoma, carcinoid tumor
 Malignant — bronchogenic carcinoma, metastatic cancer
 Foreign body
 Interstitial lung diseases
 Hypersensitivity pneumonitis
 Bronchiolitis obliterans with organizing pneumonia, ( BOOP)
 Sarcoidosis
 Chronic interstitial pneumonia
 Chronic aspiration
 Masses in the neck/thyroid disorders
 Hair impinging on the tympanic membrane
 Bronchiectasis
 Occult congestive heart failure
 Disorders of the pleura, pericardium, diaphragm
 Psychogenic/habitual cough
 Occupational bronchitis
 Enlarged tonsils or uvula
History
 The cause(s) of chronic cough may become apparent after
taking a careful history.
 Is the symptom a cough or “hawking” or clearing the throat?
 It helps to have the patient act out the cough to distinguish
true cough from throat clearing.
 Is the cough dry or productive? If so, what is produced?
 Are systemic symptoms such as fever, night sweats or weight
loss present?
 A detailed history of the work and home environment should
be taken with emphasis on possible exposure to noxious
inhalants or allergens. The history should include the time and
circumstances of onset, frequency, and aggravating and
relieving factors.
 Patients with asthma may note worsening of cough on
exposure to cold air, irritants or allergens. Is there an allergic
history? Does the patient wheeze with cough?
HISTORY
 Is the cough accompanied by dyspnea? If so, congestive heart
failure or interstitial lung disease may be suspected.
 Is the cough related to time of day, eating or position?  A
nocturnal cough may be associated with asthma, post-nasal
drip, congestive heart failure or gastroesophageal reflux
disease (GERD). Half of the patients with GERD have none of
the classic symptoms.
 Does the patient cough while eating? Chronic aspiration is
common in the elderly patient, especially following stroke.
 Is the patient on angiotensin converting enzyme inhibitors or
other drugs that may predispose to cough or asthma?
 Do not overlook ophthalmic preparations. Beta blocker eye
drops may precipitate asthma.
Physical
 The physical examination may provide clues to the
causes of cough.
 Examination of the upper airways may show nasal
mucous membrane swelling, post-nasal drip or nasal
polyps.
 The finding of wheezes, rhonchi or crackles may
indicate asthma, bronchitis, COPD, interstitial lung
disease or congestive heart failure.
 The finding of unilateral wheezing may be due to an
endobronchial lesion or foreign body.
 Masses in the neck, including thyroid enlargement,
can compress the trachea and cause cough.
Diagnostic Tests
 The work-up for chronic cough should begin with standard posterior- anterior and
lateral chest x-rays  these often reveal the presence of underlying infectious or
neoplastic causes of chronic cough.
 Spirometric studies before and after bronchodilator administration may reveal
reversible airways obstruction (asthma).
 In patients with normal base-line spirometry  methacholine inhalation challenge
(MIC) is indicated to rule out asthma that presents primarily with cough. ( COUGH
VARIANT ASTHMA)
 Computerized tomograms (CT) of the sinuses are superior to plain x-rays in
identifying sinusitis.
 High-resolution or spiral CT scans of the thorax may reveal subtle changes
consistent with cough due to chronic interstitial pneumonia or bronchiectasis.
 The finding of a reduced single breath diffusing capacity (DLCO) may suggest
interstitial lung disease.
 Barium esophagograms and upper gastrointestinal endoscopy have a low sensitivity
(48%) and specificity (76% ) for identifying GERD as the culprit in chronic cough
 monitoring the esophageal pH for 24 hours is the gold standard. ( If cough is the
only symptom of GERD it gets difficult to diagnose  so, 24hr Ph monitoring)
 In patients suspected of having chronic aspiration, a video swallowing study with a
speech therapist in attendance should be performed. ( SWALLOW
EVALUATION)
 A systematic approach to the work-up of a patient with nondrug-related chronic
cough is presented in THE NEXT SLIDE. If you suspect Drug related cough 
stop the drug and observe
Chronic Cough
Post-nasal Drip Syndrome
 Post-nasal drip syndrome is said to be one of the most
common causes of chronic cough and is caused by a variety of
conditions including vasomotor rhinitis, allergic rhinitis, nasal
polyps and chronic sinusitis.
 The diagnosis is made on clinical grounds.
 Patients may complain of a tickle or drainage of liquid in the
back of the throat.
 On examination, cobblestoning of the nasal or oropharygeal
mucosa may be observed. In many patients cough may be the
only symptom of post-nasal drip syndrome.
 Confirmation of the diagnosis may depend on the resolution of
symptoms after treatment with antihistamines and intranasal or
systemic corticosteroids.
Chronic Cough
Asthma & Cough-variant Asthma
 Typically, asthma patients complain of episodic wheezing, cough,
chest tightness and dyspnea and demonstrate reversible obstructive
air flow.
 In so called cough-variant asthma  a dry cough, particularly at
night, is the only symptom and routine spirometry is normal.
 Diagnosis : Spirometry is normal in cough variant type  The
diagnosis is often made on the basis of a favorable clinical
response to empirically administered beta2-agonist bronchodilators
and inhaled corticosteroids, and a positive bronchoprovocation test
using methacholine inhalation challenge (MIC)  A positive MIC
test, defined as a 20% or greater decrease in the FEV1 after MIC,
indicates bronchial hyperreactivity but not necessarily asthma. For
example, bronchial hyperreactivity may follow viral respiratory
tract infections and persist for as long as 6 weeks. Because MIC
has a positive predictive value of from only 60% to 80%, Irwin and
colleagues advise that a positive test must be correlated with
favorable response to therapy before concluding that a patient has
cough-variant asthma.
Chronic Cough
Gastroesophageal Reflux-related Chronic Cough
 GERD is a very common problem. Surveys of the general population have
led to estimates that 10% of the adult population of the United States have
daily heartburn and a third have intermittent symptoms; moreover, GERD
has been shown to cause 10% to 40% of cases of chronic cough  Cough
in GERD is triggered by reflux of acid into the distal esophagus and
stimulation of an esophageal-tracheobronchial reflex. Cough is not
dependent on aspiration into the larynx or tracheobronchial tree.
 Proving the relationship of chronic cough to GERD can be difficult. The
lack of typical symptoms of reflux and negative endoscopic and
radiographic studies do not rule it out.
 The 24-hour esophageal pH monitoring test has become the gold standard
for diagnosis and has both a sensitivity and specificity approaching 90%.
 Correlation of the results of pH monitoring with response to therapy adds to
the reliability of the test.
 If GERD is the sole cause of chronic cough, aggressive anti-reflux therapy
should eliminate the cough in nearly all cases. One study reported 100%
success. Treatment involves the use of dietary, mechanical and drug
therapy. Drug therapy should be initiated with proton pump inhibitors for
GERD.
Chronic Cough
Post-infectious Cough
 Patients who have had recent viral respiratory tract
infections may have prolonged cough that is
refractory to treatment. Airway hyperresponsiveness
can be demonstrated by MIC testing in some cases.
Treatment with bronchodilators and inhaled or
systemic corticosteroids in moderate to high doses
may help relieve symptoms. The cough can be self-
perpetuating and cause continuing trauma to the
airways, and in these cases, prolonged suppression
with narcotics may eventually allow resolution.
 Bordetella pertussis (the cause of whooping cough)
infection in adults should be included in the
differential diagnosis of chronic cough. In one series
of 75 patients with chronic cough lasting longer than
2 weeks, 21% had pertussis.
Chronic Cough
Angiotensin Converting Enzyme Inhibitor Cough
 Angiotensin converting enzyme inhibitor (ACEI) drugs are frequently used
in the treatment of hypertension, congestive heart failure and myocardial
infarction. Ten to 20% of patents taking ACEI drugs develop cough. There
is no evidence at this time that any one ACEI drug is less likely to cause
cough than another. In spite of this well-documented side effect, referrals to
a specialist for evaluation of chronic cough still occur frequently. Many of
these patients have had extensive and costly work-ups and treatment with a
variety of medications, including antihistamines, antibiotics, cough
suppressants and corticosteroids, without relief.
 Clinically, the cough may begin from as early as 3 weeks to as long as a
year after starting treatment. The severity of the cough can vary from a
mild tickle in the throat to a severe hacking, debilitating cough that
interferes with sleep, work and social function. It is frequently worse at
night and in the supine position.
 When the ACEI drug is discontinued, the cough usually abates in 2 weeks
but may persist for months.
 Angiotensin ll receptor antagonists have not been associated with an
increased incidence of cough.
Chronic Cough
Less Common Causes of Cough
 Chronic cough may be the presenting
complaint in patients who ultimately prove to
have tumors, both benign and malignant,
sarcoidosis or other infiltrating lung diseases;
all these conditions require special
investigations to make the diagnosis.
 Psychogenic or habitual cough does exist but
patients should not be put in this category
without an exhaustive work-up, failure of
empirical therapy and prolonged follow up.
Chronic Cough
Symptomatic Treatment
 The treatment of cough is effective only if directed at the cause, but patients should
be offered symptomatic relief while awaiting the results of specific therapy.
Expectorants such as iodides and guaifenesin, hydration, inhaled steam, cough
lozenges and hard candies are helpful. Dextromethorphan and codeine are effective
cough suppressants.
When to Refer
 When the patient with chronic cough remains symptomatic despite evaluation and
treatment for 6 to 8 weeks, the primary care physician should consider referral to a
specialist. In difficult cases referral to a pulmonologist for evaluation, therapy and
for specific testing such as fiberoptic bronchoscopy and MIC is recommended.
Referral for upper gastrointestinal endoscopy and 24-hour pH monitoring may be
indicated to rule out cough due to GERD. Referral to an allergist may be indicated
for allergy testing and subsequently for immunotherapy if the patient is sensitive to
an unavoidable antigen.
Medicolegal Issues
 One of the most common reasons patients file suit is for failure to diagnose cancer.
 Even though bronchogenic carcinoma is an uncommon cause of chronic cough
in the context of a normal chest x-ray, it must not be overlooked .
 Failure to diagnose tuberculosis is another cause of litigation but again would be an
unlikely cause of chronic cough with normal chest roentgenograms .
Solitary Pulmonary
Nodule
Characteristics of Solitary
Pulmonary Nodules
Variable Benign Malignant
Age < 30 years > 50 years
Calcificati
on
Popcorn, dense,
concentric
None or minimal
Nodule
edge
Smooth, round Irregular, spiculated
Smoking
history
Never smoked > 20 pack-years
Size of
diameter
< 1.5 cm > 1.5 cm
X-ray Characteristics of SPN
 Benign nodule charecterestics : the presence of calcification, which can be
a diffuse speckled, or “popcorn,” pattern, typical of a hamartoma, or a large
central nidus or concentric calcification typical of a granuloma.
 The second important factor distinguishing a malignant from a benign
nodule is the growth rate. Since the “doubling time” of a lung cancer
ranges from 15 to 450 days, the nodule that does not increase in diameter
over a two year period can be considered benign. Any lesion that increases
in size over a two year period of observation, or less, must be considered
malignant until proven otherwise. One exception is a nodule doubling in
less than 20 days, which usually suggests an acute inflammatory process.
 The third important characteristic is the appearance of the nodule’s edge.
Benign lesions have smooth rounded edges, whereas the incidence of
neoplasm increases dramatically in lesions with irregular, spiculated
borders. An increasing incidence of malignancy occurs, ranging from 20-
93%, depending on the degree of border irregularity
Diagnosis
 The first step in evaluating a SPN is to try to obtain old chest x-rays for
comparison  If this is not possible, and the nodule does not have a classic,
calcified appearance typical of a granuloma or hamartoma, then further testing or
a period of careful observation must be undertaken.
 A CT scan can help distinguish the pattern of calcification, and classify lesions as
“indeterminate” based on the presence of stippled or eccentric calcification and
medium density, or “benign” based on the presence of fat density typical of a
hamartoma.
 The most common CT finding in early stage adenocarcinoma and squamous cell
carcinoma of the lung is that of a solitary pulmonary nodule which enhances after
administration of IV contrast. In small cell carcinoma, however, hilar and
mediastinal adenopathy secondary to metastases is the most common CT
presentation. The presence of irregular margins, associated air bronchogram,
convergence of the surrounding structure, or the involvement of three or more
blood vessels is more likely in malignant lesions.
 If a period of observation is chosen, chest x-rays, and possibly serial CT scans,
should be done at 3-month intervals over at least a two year period to
determine if any change in the size of the nodule has occurred. An increase in
the diameter of the nodule by 25% indicates a doubling of the mass volume
a sign of malignancy.
Diagnosis - PET
 Because of the difficulty with noninvasive diagnosis of the
SPN, new radiologic techniques are being studied, including
positron emission tomography imaging (PET), which is able to
distinguish benign from malignant pulmonary nodules by
measuring 18-fluorodeoxyglucose (FDG), and by showing
increased FDG uptake and retention in malignant cells. PET
scanning is a valuable, noninvasive tool with a 95% sensitivity
for identifying malignancy and a specificity of 85% or greater.
However, false positive results may be obtained in lesions
containing an active inflammatory process (for example a
reactive lymphadenopathy), and this diagnostic modality is not
generally available.
SPN
When to Refer
 Once the decision has been made that the patient’s SPN may represent a
malignancy, a histologic diagnosis is needed. If the patient’s SPN has
characteristics strongly suggesting malignancy, and there are no
contraindications to surgery, refer to a thoracic surgeon.
 In most other circumstances refer to a pulmonologist for further workup.
Diagnostic procedures may include: fiberoptic bronchoscopy aided by
fluoroscopy, or CT-guided transthoracic fine needle aspiration. The yield of
these procedures in the diagnosis of the small solitary pulmonary nodule (<
1.5 cm in diameter) is about 40% for fiberoptic bronchoscopy, and 50% for
fine needle aspiration. The incidence of pneumothorax requiring chest tube
insertion from bronchoscopic transbronchial biopsy is about 5% and from
needle aspiration about 25%, depending on patient characteristics and
variation of local physician technique.
 Thoracoscopic resection or thoracotomy is needed for diagnosis in about
20% of patients, in whom the less invasive techniques were not successful.
 A 43-year-old woman is evaluated because of an abnormal chest
radiograph taken before an elective hysterectomy for fibroids. She has no
previous history of pulmonary disease. Her cardiac and pulmonary review
of systems is unremarkable. The patient smoked one pack of cigarettes per
day from age 16 to 33 years, but has not smoked since then. On physical
examination, her blood pressure is 120/60 mm Hg and the lung fields are
clear. The remainder of the examination is unremarkable. Her laboratory
evaluation, including a complete blood count and chemistry profile, is
normal. Spirometry shows forced expiratory volume in 1 sec (FEV1), 2.72
L (84% of predicted); forced vital capacity (FVC), 3.68 L (98% of
predicted); and FEV1/FVC ratio of 74%. Chest radiography shows an
approximately 1-cm nodule in the left lower lobe periphery. There are no
previous radiographs. High-resolution helical computed tomography (CT)
of the 1.2-cm lesion in the left lower lobe is shown. The full chest CT
shows no evidence of mediastinal adenopathy. Which of the following is
the best management option?
 ( A ) Bronchoscopy with transbronchial biopsies
 ( B ) No further studies are needed
 ( C ) Repeat high-resolution CT in 3 months
 ( D ) Resection of the lesion with video-assisted thoracoscopic surgery
 ( E ) Percutaneous fine needle aspiration of the lesion
 The likelihood of a solitary pulmonary nodule
being malignant substantially decreases if the
lesion is small (approximately 1cm), has
smooth borders, is located in a lower lobe, and,
most importantly, has central calcification.
 Observation at 3 month intervals for 2 years to
ensure stability of the finding is sufficient!!
Pleural Effusions
EFFUSIONS
 EXUDATES: pleural fluid [protein] / plasma
[protein] > 0.5 and/or pleural fluid
[LDH] / plasma [LDH] > 0.6 and/or pleural fluid
LDH >200 or >2/3 of serum LDH
 TRANSUDATES – Nephrotic syndrome, CHF,
Atelectasis, Cirrhosis (Hydrothorax)
 EMPYEMA
 Diagnosis  By thoracentesis. Except in known
CHF, must be done in all pleural effusions.
 A 56-year-old man with chronic alcoholism is evaluated because of fatigue, decreased appetite, and
episodes of sweating. He has a cough productive of purulent, foul-smelling grayish-green sputum and
describes heaviness in the right lower chest. The patient has reduced his activity, but only missed 2 days of
work as a painter because of his illness. On physical examination, his temperature is 37.4 °C (99.3 °F),
pulse rate is 84/min, respiration rate is 14/min, and blood pressure is 132/85 mm Hg. There is dullness to
percussion at the right lateral and posterior lung base associated with decreased breath sounds. Chest
radiography shows a large right-sided pleural effusion and a small parenchymal infiltrate. The effusion did
not layer along the chest wall in a lateral decubitus film. A thoracentesis is performed, and the leukocyte
count is 32,500/μL with83% neutrophils, the pH is 7.12, and glucose is 25 mg/dL. The fluid is sent for
culture, and antibiotics are begun.
 Which of the following should be done next to manage the pleural fluid?
 ( A ) Insertion of a pigtail catheter into the pleural space
 ( B ) Insertion of a large-bore chest tube
 ( C ) Repeat thoracentesis if the pleural effusion does not improve in 48 hours
 ( D ) Open surgical decortication
 ( E ) Reevaluation of the pleural effusion in 7 days
Ans.
 Frankly purulent pleural fluid, pleural fluid pH
<7.20, and,possibly, loculated collections of
fluid are indications for chest tube drainage
Critical Care Aspects
1. Shock
2. Respiratory Failure
3. Acute coronary Syndrome
4. Acid-Base problems
5. Neurological Emergencies
6. GI Bleeding
Shock - Objectives
 Recognize the clinical manifestations of shock -
the major classifications of shock
 Identify the IV solution type that is best suited
for water and electrolyte loss
 Describe conditions that may indicate successful
volume resuscitation
Shock
 Hypovolemic
 Distributive – Septic, Drug overdose ( opiod, BZDs,
Ethanol), Hepatic failure
 Obstructive – PE, Cardiac tamponade,
Pneumothorax, Air Embolism
 Cardiogenic – Acute LHF, RHF
 Miscellaneous – think about Adrenal insufficiency,
hypocalcemia, Autonomic dysfunction in a shock
that is refractory to pressor support and in a shock
that tends to persist even after correction of the
initial etiology
Distributive Shock
 1. Distributive (maldistribution of blood flow)
- Warm Shock – Warm extremities
 a. Septic
 b. Neurogenic – Drug overdose, vasovagal
 c. Anaphylactic
 d. Toxic shock syndrome
 e. Metabolic
Hypovolemic Shock
 Hypovolemic (decreased cardiac output)
Secondary to loss of intravascular volume
 a. Hemorrhagic
 b. Nonhemorrhagic
1. GI fluid loss (e.g., diarrhea, vomiting)
2. Renal loss (e.g., diabetes insipidus, osmotic
diuresis)
3. Evaporative loss (e.g., hyperthermia, severe
burns)
4. Third space loss (e.g., peritonitis, pancreatitis)
Cardiogenic Shock
 Cardiogenic (decreased cardiac output due to
cardiac factors)
a. Impaired left ventricular contractility (e.g.,
ischemia, infarct, congestive myopathy)
b. Impaired right ventricular contractility (e.g.,
right ventricular infarction)
c. Acute regurgitant lesions (e.g., mitral or aortic
regurgitation, intraventricular septal rupture)
d. Obstructive cardiac lesions (e.g., aortic stenosis,
subaortic stenosis)
e. Bradyarrhythmias or tachyarrhythmias
Obstructive Shock
 Obstructive (decreased cardiac output due to
factors extrinsic to the heart)
 a. Pericardial tamponade
 b. Tension pneumothorax
 c. Pulmonary embolism
 d. Severe pulmonary hypertension
Shock
 Pulmonary Embolism
Etiology
Clinical Features
Investigations – BEDSIDE ECHO THE BEST INITIAL
SCREENING TEST IF U SUSPECT SHOCK DUE
TO PE IN A CRITICALLY ILL PATIENT – Can
give you information on other causes – Acute MI,
Cardiac tamponade, Acute corpulmonale ( ARDS,
PE)
Treatment
Shock
 Cardiac Tamponade
Etiology – Trauma,Rapid Effusions - tumor
Clinical features
Investigations – X-Ray, CT chest, 2D echo
Treatment
Shock
 Pneumothorax ( obstructive phenomena)
Etiology – usual scenario- an icu patient on ventilator –
sudden hypotension
Others – trauma, rupture emphysemal bullae
Clinical features – auscultate – SILENT!!
- look at the ventilator parameters -----
high peak pressures as well as plateau pressures
Investigations – CXR, Needle thoracentesis
Treatment – tube thoracostomy, high fio2
Shock
 Acute Corpulmonale
- ARDS
- Pulmonary Embolism
Rx – IV fluids, correct the cause
Shock
 Cardiogenic - Right Heart Failure
 Symptoms, EKG, Signs, Enzymes
 Treatment – fluids, fluids, fluids
cardiac cath, fix the lesion
Shock
 Cardiogenic – Left Heart Failure
 Causes - Acute MI
 Rx – IABC – To buy time
- Dopamine, Dobutamine
- then cardiac cath, fix the lesion -
ptca
Shock
 Distributive
SEPSIS ---- distributes to periphery  WARM shock, systemic
vasodilation - reduced SVR ( Swan Ganz )
Investigations – pan cultures, cxr, --- consider random
cortisol…if requiring pressor support for a long time  rule
out relative adrenal insufficiency.
Rx – Fluids,fluids, fluids  antibiotics  hydrocortisone 100
q8hrs if relative/ absolute adrenal insufficiency. Remove the
source - ? IV Catheters, hemodialysis access, foley
Pressor support – norepinephrine, dopamine, vasopressin
 Distributive/ Anaphylactic – Epinephrine, corticosteroids,
beta agonists, Antihistamines
Shock
 Hypovolemic –
Hypovolemia Vs. Dehydration
- GI blood loss, vomiting, excessive diarrhea,Other trauma/
blood loss, Diabetes Insipidus, Diuretics ( overdiuresis),
Diuretic abuse, Laxative abuse, Bulimia nervosa, Surreptious
vomiting
- Rx: Fluids, blood if required, Correct hypernatremia slowly <
0.5 meq/hr if present
- Rx the underlying cause – fix GI bleeds, any ? coagulopathy,
consults if required
- ? Renal failure – prerenal – indicators Urine Na+, FeNa, urine
sp gravity,
Shock – Case studies
1) A 38-year old female on birth control pills, has suddenly become extremely
short of breath. Someone has seen her collapse and called 911. She was
diaphoretic and complained of severe chest pain before she collapsed. She is
now in the ER/ED and you have been asked to evaluate her. Her old records
show that she is a cocaine abuser and was admitted for subarachnoid
hemorrhage 6 weeks ago from which she completely recovered. Clinical
findings. Vitals : B.P 65/ palpable, R.R 45. Pulse 140, Tm: 99.2 F. Chest
showed decreased breath sounds in right lower lobe and distant heart
sounds. Pulse oximetry revealed 88%. EKG showed sinus tachycardia with s
wave in lead I, q wave in lead III and T wave inversion in lead III. 2D echo
showed hypokinetic right ventricle with pulmonary hypertension. You
started her on Intravenos fluids and her blood pressure has slightly
improved to 66/30. Your next step in management ?
A) Transfer to cath lab and notify the interventional cardiologist stat
B) Intra aortic balloon counterpulsation
C) Thrombolytic therapy
D) Surgical Embolectomy and Inferior vena cava filter
E) Obtain cardiothoracic surgery consult for subxiphoid window
Pulmonology  critical care- archer step3 lectures
Pulmonology  critical care- archer step3 lectures
Pulmonology  critical care- archer step3 lectures
Pulmonology  critical care- archer step3 lectures
Pulmonology  critical care- archer step3 lectures
Pulmonology  critical care- archer step3 lectures
Pulmonology  critical care- archer step3 lectures
Pulmonology  critical care- archer step3 lectures
Pulmonology  critical care- archer step3 lectures
Pulmonology  critical care- archer step3 lectures
Pulmonology  critical care- archer step3 lectures
Pulmonology  critical care- archer step3 lectures
Pulmonology  critical care- archer step3 lectures
Pulmonology  critical care- archer step3 lectures
Pulmonology  critical care- archer step3 lectures
Pulmonology  critical care- archer step3 lectures
Pulmonology  critical care- archer step3 lectures
Pulmonology  critical care- archer step3 lectures
Pulmonology  critical care- archer step3 lectures
Pulmonology  critical care- archer step3 lectures
Pulmonology  critical care- archer step3 lectures
Pulmonology  critical care- archer step3 lectures
Pulmonology  critical care- archer step3 lectures
Pulmonology  critical care- archer step3 lectures
Pulmonology  critical care- archer step3 lectures
Pulmonology  critical care- archer step3 lectures
Pulmonology  critical care- archer step3 lectures
Pulmonology  critical care- archer step3 lectures
Pulmonology  critical care- archer step3 lectures
Pulmonology  critical care- archer step3 lectures
Pulmonology  critical care- archer step3 lectures

Contenu connexe

Tendances

Hypereosinophilia overview - pediatric resident
Hypereosinophilia overview - pediatric resident Hypereosinophilia overview - pediatric resident
Hypereosinophilia overview - pediatric resident DrSheika Bawazir
 
Hypertensive emergencies treatment
Hypertensive  emergencies treatmentHypertensive  emergencies treatment
Hypertensive emergencies treatmentabhishek144
 
Snake Bite Icu Management
Snake Bite Icu ManagementSnake Bite Icu Management
Snake Bite Icu Managementchandra talur
 
Hypertensive crisis
Hypertensive crisisHypertensive crisis
Hypertensive crisisSMSRAZA
 
Fluids and electrolytes Archer NCLEX content review webinar
Fluids and electrolytes  Archer NCLEX content review webinarFluids and electrolytes  Archer NCLEX content review webinar
Fluids and electrolytes Archer NCLEX content review webinarArcher Review USMLE and NCLEX
 
Pals 2017 part 2
Pals 2017  part 2Pals 2017  part 2
Pals 2017 part 2Sayed Ahmed
 
Drowning by dr mahmoud zalam kfmc riyadh ksa
Drowning by dr mahmoud zalam kfmc riyadh ksaDrowning by dr mahmoud zalam kfmc riyadh ksa
Drowning by dr mahmoud zalam kfmc riyadh ksamahmoud mohamed
 
Scorpion sting
Scorpion stingScorpion sting
Scorpion stingikramdr01
 
Management of venomous snake bite in Bangladesh
Management of venomous snake bite in BangladeshManagement of venomous snake bite in Bangladesh
Management of venomous snake bite in BangladeshForhad Uddin Maruf
 

Tendances (20)

Archer step 3 ccs workshop 2019
Archer step 3 ccs workshop 2019Archer step 3 ccs workshop 2019
Archer step 3 ccs workshop 2019
 
Hypertensive emergency
Hypertensive emergencyHypertensive emergency
Hypertensive emergency
 
Hypereosinophilia overview - pediatric resident
Hypereosinophilia overview - pediatric resident Hypereosinophilia overview - pediatric resident
Hypereosinophilia overview - pediatric resident
 
Hypertensive emergencies treatment
Hypertensive  emergencies treatmentHypertensive  emergencies treatment
Hypertensive emergencies treatment
 
Snake Bite Icu Management
Snake Bite Icu ManagementSnake Bite Icu Management
Snake Bite Icu Management
 
Hypertensive crisis
Hypertensive crisisHypertensive crisis
Hypertensive crisis
 
Hp Crisis
Hp CrisisHp Crisis
Hp Crisis
 
Fluids and electrolytes Archer NCLEX content review webinar
Fluids and electrolytes  Archer NCLEX content review webinarFluids and electrolytes  Archer NCLEX content review webinar
Fluids and electrolytes Archer NCLEX content review webinar
 
Drowning
DrowningDrowning
Drowning
 
Acetaminophen poisoning
Acetaminophen poisoningAcetaminophen poisoning
Acetaminophen poisoning
 
Pals 2017 part 2
Pals 2017  part 2Pals 2017  part 2
Pals 2017 part 2
 
Hypertensive crisis
Hypertensive crisisHypertensive crisis
Hypertensive crisis
 
Toxicology
ToxicologyToxicology
Toxicology
 
Dr. Red Sample C C S Casesfor U S M L E Step3.Doc
Dr. Red  Sample C C S Casesfor U S M L E Step3.DocDr. Red  Sample C C S Casesfor U S M L E Step3.Doc
Dr. Red Sample C C S Casesfor U S M L E Step3.Doc
 
Anaphylaxis
Anaphylaxis Anaphylaxis
Anaphylaxis
 
Drowning by dr mahmoud zalam kfmc riyadh ksa
Drowning by dr mahmoud zalam kfmc riyadh ksaDrowning by dr mahmoud zalam kfmc riyadh ksa
Drowning by dr mahmoud zalam kfmc riyadh ksa
 
Scorpion sting
Scorpion stingScorpion sting
Scorpion sting
 
Endocrine Archer NCLEX course Webinar
Endocrine Archer NCLEX course WebinarEndocrine Archer NCLEX course Webinar
Endocrine Archer NCLEX course Webinar
 
Management of venomous snake bite in Bangladesh
Management of venomous snake bite in BangladeshManagement of venomous snake bite in Bangladesh
Management of venomous snake bite in Bangladesh
 
ECG: Hyperkalemia
ECG: HyperkalemiaECG: Hyperkalemia
ECG: Hyperkalemia
 

Similaire à Pulmonology critical care- archer step3 lectures

Ent By Prof. Dr.Yasser Nour.
Ent By Prof. Dr.Yasser Nour.Ent By Prof. Dr.Yasser Nour.
Ent By Prof. Dr.Yasser Nour.guest1fcaba5
 
Geekymedics.com allergic rhinitis
Geekymedics.com allergic rhinitisGeekymedics.com allergic rhinitis
Geekymedics.com allergic rhinitisMahir Mohamet
 
11. RHINITIS.pptx
11. RHINITIS.pptx11. RHINITIS.pptx
11. RHINITIS.pptxGalagaPius
 
Acute sinusitis
Acute sinusitisAcute sinusitis
Acute sinusitisElias Pour
 
Rhinitis presentation
Rhinitis presentationRhinitis presentation
Rhinitis presentationSasha Bondi
 
Allergic Rhinitis
Allergic RhinitisAllergic Rhinitis
Allergic RhinitisNeil Kao
 
Allergic Rhinitis.pptx
Allergic Rhinitis.pptxAllergic Rhinitis.pptx
Allergic Rhinitis.pptxDoinaChiaburu1
 
Allergic Rhinitis.pptx
Allergic Rhinitis.pptxAllergic Rhinitis.pptx
Allergic Rhinitis.pptxh8wds7pgpv
 
Allergic Rhinitis.pptx
Allergic Rhinitis.pptxAllergic Rhinitis.pptx
Allergic Rhinitis.pptxfaheem411362
 
Asthma & allergic rhinitis
Asthma & allergic rhinitisAsthma & allergic rhinitis
Asthma & allergic rhinitisMohibullah salih
 
Allergic rhinitis &amp; vasomotor rhinitis theory modified dr.bakshi-17.05.06
Allergic rhinitis &amp; vasomotor rhinitis theory modified dr.bakshi-17.05.06Allergic rhinitis &amp; vasomotor rhinitis theory modified dr.bakshi-17.05.06
Allergic rhinitis &amp; vasomotor rhinitis theory modified dr.bakshi-17.05.06ophthalmgmcri
 
Allergic rhinitis lecture 100829
Allergic rhinitis lecture 100829Allergic rhinitis lecture 100829
Allergic rhinitis lecture 100829Neil Kao
 
Upper Respiratory Tract Infection by Dr. Sookun Rajeev Kumar
Upper Respiratory Tract Infection by Dr. Sookun Rajeev KumarUpper Respiratory Tract Infection by Dr. Sookun Rajeev Kumar
Upper Respiratory Tract Infection by Dr. Sookun Rajeev KumarDr. Sookun Rajeev Kumar
 
Allergic rhinitis
Allergic rhinitisAllergic rhinitis
Allergic rhinitisReynel Dan
 

Similaire à Pulmonology critical care- archer step3 lectures (20)

Pulmonology
PulmonologyPulmonology
Pulmonology
 
Allergy rhinitis
Allergy rhinitisAllergy rhinitis
Allergy rhinitis
 
Ent By Prof. Dr.Yasser Nour.
Ent By Prof. Dr.Yasser Nour.Ent By Prof. Dr.Yasser Nour.
Ent By Prof. Dr.Yasser Nour.
 
Geekymedics.com allergic rhinitis
Geekymedics.com allergic rhinitisGeekymedics.com allergic rhinitis
Geekymedics.com allergic rhinitis
 
RESPIRATORY DISORDERS
RESPIRATORY DISORDERSRESPIRATORY DISORDERS
RESPIRATORY DISORDERS
 
11. RHINITIS.pptx
11. RHINITIS.pptx11. RHINITIS.pptx
11. RHINITIS.pptx
 
Acute sinusitis
Acute sinusitisAcute sinusitis
Acute sinusitis
 
Rhinitis presentation
Rhinitis presentationRhinitis presentation
Rhinitis presentation
 
Allergic rhinitis
Allergic rhinitisAllergic rhinitis
Allergic rhinitis
 
Allergic Rhinitis
Allergic RhinitisAllergic Rhinitis
Allergic Rhinitis
 
Allergic Rhinitis.pptx
Allergic Rhinitis.pptxAllergic Rhinitis.pptx
Allergic Rhinitis.pptx
 
Allergic Rhinitis.pptx
Allergic Rhinitis.pptxAllergic Rhinitis.pptx
Allergic Rhinitis.pptx
 
Allergic Rhinitis.pptx
Allergic Rhinitis.pptxAllergic Rhinitis.pptx
Allergic Rhinitis.pptx
 
Asthma & allergic rhinitis
Asthma & allergic rhinitisAsthma & allergic rhinitis
Asthma & allergic rhinitis
 
Allergic rhinitis &amp; vasomotor rhinitis theory modified dr.bakshi-17.05.06
Allergic rhinitis &amp; vasomotor rhinitis theory modified dr.bakshi-17.05.06Allergic rhinitis &amp; vasomotor rhinitis theory modified dr.bakshi-17.05.06
Allergic rhinitis &amp; vasomotor rhinitis theory modified dr.bakshi-17.05.06
 
Allergic rhinitis lecture 100829
Allergic rhinitis lecture 100829Allergic rhinitis lecture 100829
Allergic rhinitis lecture 100829
 
Upper Respiratory Tract Infection by Dr. Sookun Rajeev Kumar
Upper Respiratory Tract Infection by Dr. Sookun Rajeev KumarUpper Respiratory Tract Infection by Dr. Sookun Rajeev Kumar
Upper Respiratory Tract Infection by Dr. Sookun Rajeev Kumar
 
Allergic rhinitis
Allergic rhinitisAllergic rhinitis
Allergic rhinitis
 
Allergic rhinitis.
Allergic rhinitis.Allergic rhinitis.
Allergic rhinitis.
 
RhinoSinusitis
RhinoSinusitisRhinoSinusitis
RhinoSinusitis
 

Plus de Archer Review USMLE and NCLEX

Safety and infection control - Archer NCLEX webinars
Safety and infection control - Archer NCLEX webinarsSafety and infection control - Archer NCLEX webinars
Safety and infection control - Archer NCLEX webinarsArcher Review USMLE and NCLEX
 
Archer NCLEX Webinars - Diets and Nutrition crash course
Archer NCLEX Webinars - Diets and Nutrition crash courseArcher NCLEX Webinars - Diets and Nutrition crash course
Archer NCLEX Webinars - Diets and Nutrition crash courseArcher Review USMLE and NCLEX
 
Archer NCLEX Rapid Review - Full Content Syllabus - Two days
Archer NCLEX Rapid Review - Full Content Syllabus - Two daysArcher NCLEX Rapid Review - Full Content Syllabus - Two days
Archer NCLEX Rapid Review - Full Content Syllabus - Two daysArcher Review USMLE and NCLEX
 
Cardiology and EKGs - Archer NCLEX crash course/ webinar
Cardiology and EKGs - Archer NCLEX crash course/ webinarCardiology and EKGs - Archer NCLEX crash course/ webinar
Cardiology and EKGs - Archer NCLEX crash course/ webinarArcher Review USMLE and NCLEX
 

Plus de Archer Review USMLE and NCLEX (20)

Safety and infection control - Archer NCLEX webinars
Safety and infection control - Archer NCLEX webinarsSafety and infection control - Archer NCLEX webinars
Safety and infection control - Archer NCLEX webinars
 
Archer NCLEX Webinars - Diets and Nutrition crash course
Archer NCLEX Webinars - Diets and Nutrition crash courseArcher NCLEX Webinars - Diets and Nutrition crash course
Archer NCLEX Webinars - Diets and Nutrition crash course
 
Rapid Content Prep - NCLEX- Archer Review
Rapid Content Prep - NCLEX- Archer ReviewRapid Content Prep - NCLEX- Archer Review
Rapid Content Prep - NCLEX- Archer Review
 
Archer NCLEX Rapid Review - Full Content Syllabus - Two days
Archer NCLEX Rapid Review - Full Content Syllabus - Two daysArcher NCLEX Rapid Review - Full Content Syllabus - Two days
Archer NCLEX Rapid Review - Full Content Syllabus - Two days
 
Cardiology and EKGs - Archer NCLEX crash course/ webinar
Cardiology and EKGs - Archer NCLEX crash course/ webinarCardiology and EKGs - Archer NCLEX crash course/ webinar
Cardiology and EKGs - Archer NCLEX crash course/ webinar
 
Archer NCLEX Review - Electrolyte imbalances
Archer NCLEX Review - Electrolyte imbalancesArcher NCLEX Review - Electrolyte imbalances
Archer NCLEX Review - Electrolyte imbalances
 
Endocrinology archer step3 lectures
Endocrinology   archer step3 lecturesEndocrinology   archer step3 lectures
Endocrinology archer step3 lectures
 
Archer USMLE Step 3 CCS workshop 2018
Archer USMLE Step 3 CCS workshop 2018Archer USMLE Step 3 CCS workshop 2018
Archer USMLE Step 3 CCS workshop 2018
 
Cutaneous lymphomas
Cutaneous lymphomasCutaneous lymphomas
Cutaneous lymphomas
 
Acute Lymphoblastic Lymphoma
Acute Lymphoblastic LymphomaAcute Lymphoblastic Lymphoma
Acute Lymphoblastic Lymphoma
 
Gastroenterology
Gastroenterology Gastroenterology
Gastroenterology
 
Carcinoma of unknown primary
Carcinoma of unknown primaryCarcinoma of unknown primary
Carcinoma of unknown primary
 
Bladder cancer
Bladder cancerBladder cancer
Bladder cancer
 
Plasma cell disorders ppt
Plasma cell disorders pptPlasma cell disorders ppt
Plasma cell disorders ppt
 
Thrombophilias
ThrombophiliasThrombophilias
Thrombophilias
 
Thrombocytopenia
ThrombocytopeniaThrombocytopenia
Thrombocytopenia
 
Several Samples - Personal Statements
Several Samples - Personal StatementsSeveral Samples - Personal Statements
Several Samples - Personal Statements
 
Internal medicine Personal Statement
Internal medicine Personal StatementInternal medicine Personal Statement
Internal medicine Personal Statement
 
Acing the residency interview
Acing the residency interviewAcing the residency interview
Acing the residency interview
 
How to nail that Residency interview?
How to nail that Residency interview?How to nail that Residency interview?
How to nail that Residency interview?
 

Dernier

Software Engineering Methodologies (overview)
Software Engineering Methodologies (overview)Software Engineering Methodologies (overview)
Software Engineering Methodologies (overview)eniolaolutunde
 
Q4-W6-Restating Informational Text Grade 3
Q4-W6-Restating Informational Text Grade 3Q4-W6-Restating Informational Text Grade 3
Q4-W6-Restating Informational Text Grade 3JemimahLaneBuaron
 
Activity 01 - Artificial Culture (1).pdf
Activity 01 - Artificial Culture (1).pdfActivity 01 - Artificial Culture (1).pdf
Activity 01 - Artificial Culture (1).pdfciinovamais
 
APM Welcome, APM North West Network Conference, Synergies Across Sectors
APM Welcome, APM North West Network Conference, Synergies Across SectorsAPM Welcome, APM North West Network Conference, Synergies Across Sectors
APM Welcome, APM North West Network Conference, Synergies Across SectorsAssociation for Project Management
 
fourth grading exam for kindergarten in writing
fourth grading exam for kindergarten in writingfourth grading exam for kindergarten in writing
fourth grading exam for kindergarten in writingTeacherCyreneCayanan
 
Grant Readiness 101 TechSoup and Remy Consulting
Grant Readiness 101 TechSoup and Remy ConsultingGrant Readiness 101 TechSoup and Remy Consulting
Grant Readiness 101 TechSoup and Remy ConsultingTechSoup
 
IGNOU MSCCFT and PGDCFT Exam Question Pattern: MCFT003 Counselling and Family...
IGNOU MSCCFT and PGDCFT Exam Question Pattern: MCFT003 Counselling and Family...IGNOU MSCCFT and PGDCFT Exam Question Pattern: MCFT003 Counselling and Family...
IGNOU MSCCFT and PGDCFT Exam Question Pattern: MCFT003 Counselling and Family...PsychoTech Services
 
General AI for Medical Educators April 2024
General AI for Medical Educators April 2024General AI for Medical Educators April 2024
General AI for Medical Educators April 2024Janet Corral
 
Call Girls in Dwarka Mor Delhi Contact Us 9654467111
Call Girls in Dwarka Mor Delhi Contact Us 9654467111Call Girls in Dwarka Mor Delhi Contact Us 9654467111
Call Girls in Dwarka Mor Delhi Contact Us 9654467111Sapana Sha
 
Disha NEET Physics Guide for classes 11 and 12.pdf
Disha NEET Physics Guide for classes 11 and 12.pdfDisha NEET Physics Guide for classes 11 and 12.pdf
Disha NEET Physics Guide for classes 11 and 12.pdfchloefrazer622
 
Sports & Fitness Value Added Course FY..
Sports & Fitness Value Added Course FY..Sports & Fitness Value Added Course FY..
Sports & Fitness Value Added Course FY..Disha Kariya
 
BAG TECHNIQUE Bag technique-a tool making use of public health bag through wh...
BAG TECHNIQUE Bag technique-a tool making use of public health bag through wh...BAG TECHNIQUE Bag technique-a tool making use of public health bag through wh...
BAG TECHNIQUE Bag technique-a tool making use of public health bag through wh...Sapna Thakur
 
1029 - Danh muc Sach Giao Khoa 10 . pdf
1029 -  Danh muc Sach Giao Khoa 10 . pdf1029 -  Danh muc Sach Giao Khoa 10 . pdf
1029 - Danh muc Sach Giao Khoa 10 . pdfQucHHunhnh
 
Measures of Central Tendency: Mean, Median and Mode
Measures of Central Tendency: Mean, Median and ModeMeasures of Central Tendency: Mean, Median and Mode
Measures of Central Tendency: Mean, Median and ModeThiyagu K
 
Kisan Call Centre - To harness potential of ICT in Agriculture by answer farm...
Kisan Call Centre - To harness potential of ICT in Agriculture by answer farm...Kisan Call Centre - To harness potential of ICT in Agriculture by answer farm...
Kisan Call Centre - To harness potential of ICT in Agriculture by answer farm...Krashi Coaching
 
Russian Escort Service in Delhi 11k Hotel Foreigner Russian Call Girls in Delhi
Russian Escort Service in Delhi 11k Hotel Foreigner Russian Call Girls in DelhiRussian Escort Service in Delhi 11k Hotel Foreigner Russian Call Girls in Delhi
Russian Escort Service in Delhi 11k Hotel Foreigner Russian Call Girls in Delhikauryashika82
 
Interactive Powerpoint_How to Master effective communication
Interactive Powerpoint_How to Master effective communicationInteractive Powerpoint_How to Master effective communication
Interactive Powerpoint_How to Master effective communicationnomboosow
 
Unit-IV- Pharma. Marketing Channels.pptx
Unit-IV- Pharma. Marketing Channels.pptxUnit-IV- Pharma. Marketing Channels.pptx
Unit-IV- Pharma. Marketing Channels.pptxVishalSingh1417
 

Dernier (20)

Software Engineering Methodologies (overview)
Software Engineering Methodologies (overview)Software Engineering Methodologies (overview)
Software Engineering Methodologies (overview)
 
Q4-W6-Restating Informational Text Grade 3
Q4-W6-Restating Informational Text Grade 3Q4-W6-Restating Informational Text Grade 3
Q4-W6-Restating Informational Text Grade 3
 
Activity 01 - Artificial Culture (1).pdf
Activity 01 - Artificial Culture (1).pdfActivity 01 - Artificial Culture (1).pdf
Activity 01 - Artificial Culture (1).pdf
 
APM Welcome, APM North West Network Conference, Synergies Across Sectors
APM Welcome, APM North West Network Conference, Synergies Across SectorsAPM Welcome, APM North West Network Conference, Synergies Across Sectors
APM Welcome, APM North West Network Conference, Synergies Across Sectors
 
fourth grading exam for kindergarten in writing
fourth grading exam for kindergarten in writingfourth grading exam for kindergarten in writing
fourth grading exam for kindergarten in writing
 
Grant Readiness 101 TechSoup and Remy Consulting
Grant Readiness 101 TechSoup and Remy ConsultingGrant Readiness 101 TechSoup and Remy Consulting
Grant Readiness 101 TechSoup and Remy Consulting
 
IGNOU MSCCFT and PGDCFT Exam Question Pattern: MCFT003 Counselling and Family...
IGNOU MSCCFT and PGDCFT Exam Question Pattern: MCFT003 Counselling and Family...IGNOU MSCCFT and PGDCFT Exam Question Pattern: MCFT003 Counselling and Family...
IGNOU MSCCFT and PGDCFT Exam Question Pattern: MCFT003 Counselling and Family...
 
General AI for Medical Educators April 2024
General AI for Medical Educators April 2024General AI for Medical Educators April 2024
General AI for Medical Educators April 2024
 
Call Girls in Dwarka Mor Delhi Contact Us 9654467111
Call Girls in Dwarka Mor Delhi Contact Us 9654467111Call Girls in Dwarka Mor Delhi Contact Us 9654467111
Call Girls in Dwarka Mor Delhi Contact Us 9654467111
 
Disha NEET Physics Guide for classes 11 and 12.pdf
Disha NEET Physics Guide for classes 11 and 12.pdfDisha NEET Physics Guide for classes 11 and 12.pdf
Disha NEET Physics Guide for classes 11 and 12.pdf
 
Sports & Fitness Value Added Course FY..
Sports & Fitness Value Added Course FY..Sports & Fitness Value Added Course FY..
Sports & Fitness Value Added Course FY..
 
Mattingly "AI & Prompt Design: Structured Data, Assistants, & RAG"
Mattingly "AI & Prompt Design: Structured Data, Assistants, & RAG"Mattingly "AI & Prompt Design: Structured Data, Assistants, & RAG"
Mattingly "AI & Prompt Design: Structured Data, Assistants, & RAG"
 
BAG TECHNIQUE Bag technique-a tool making use of public health bag through wh...
BAG TECHNIQUE Bag technique-a tool making use of public health bag through wh...BAG TECHNIQUE Bag technique-a tool making use of public health bag through wh...
BAG TECHNIQUE Bag technique-a tool making use of public health bag through wh...
 
1029 - Danh muc Sach Giao Khoa 10 . pdf
1029 -  Danh muc Sach Giao Khoa 10 . pdf1029 -  Danh muc Sach Giao Khoa 10 . pdf
1029 - Danh muc Sach Giao Khoa 10 . pdf
 
Mattingly "AI & Prompt Design: The Basics of Prompt Design"
Mattingly "AI & Prompt Design: The Basics of Prompt Design"Mattingly "AI & Prompt Design: The Basics of Prompt Design"
Mattingly "AI & Prompt Design: The Basics of Prompt Design"
 
Measures of Central Tendency: Mean, Median and Mode
Measures of Central Tendency: Mean, Median and ModeMeasures of Central Tendency: Mean, Median and Mode
Measures of Central Tendency: Mean, Median and Mode
 
Kisan Call Centre - To harness potential of ICT in Agriculture by answer farm...
Kisan Call Centre - To harness potential of ICT in Agriculture by answer farm...Kisan Call Centre - To harness potential of ICT in Agriculture by answer farm...
Kisan Call Centre - To harness potential of ICT in Agriculture by answer farm...
 
Russian Escort Service in Delhi 11k Hotel Foreigner Russian Call Girls in Delhi
Russian Escort Service in Delhi 11k Hotel Foreigner Russian Call Girls in DelhiRussian Escort Service in Delhi 11k Hotel Foreigner Russian Call Girls in Delhi
Russian Escort Service in Delhi 11k Hotel Foreigner Russian Call Girls in Delhi
 
Interactive Powerpoint_How to Master effective communication
Interactive Powerpoint_How to Master effective communicationInteractive Powerpoint_How to Master effective communication
Interactive Powerpoint_How to Master effective communication
 
Unit-IV- Pharma. Marketing Channels.pptx
Unit-IV- Pharma. Marketing Channels.pptxUnit-IV- Pharma. Marketing Channels.pptx
Unit-IV- Pharma. Marketing Channels.pptx
 

Pulmonology critical care- archer step3 lectures

  • 1. Respiratory diseases Archer online USMLE reviews www.ArcherReview.com All Rights reserved Archer Slides are intended for use with Archer USMLE step 3 video lectures. Hence, most slides are very brief summaries of the concepts which will be addressed in a detailed way with focus on High-yield concepts in the Video lectures.
  • 3. Allergic Rhinitis  Hay fever  Onset under age 30  Peak incidence – childhood & adolescence  Most common chronic disease in the USA and significantly affects quality of life  Pathophysiology : Type I hypersensitivity reaction to allergens  Common allergens : Seasonal Allergens: Tree pollen (early spring), Grass pollen (late spring) and Outdoor Molds (summer and fall) ) , Perennial : Dust mites and Animal dander Irritant: Cigarette Smoke  Associated conditions :  Atopy : Eczematous Dermatitis , Allergic Rhinitis and Asthma  Allergic Triad : Aspirin Allergy, Nasal Polyp and Asthma
  • 4. Allergic Rhinitis Symptoms: Specific : Sneezing, Rhinorrhea, Nasal congestion and Pruritus of the nose, eyes, and throat , Eye Tearing and Conjunctival discharge Symptoms due to Chronic Nasal Obstruction: Mouth Breathing, Snoring, Anosmia, Cough, Headache and Halitosis Signs : * Look for antihistamine induced Hypertension in these guys *Nose exam : pale blue and boggy mucosa, clear discharge *Face exam: “Allergic Shiners”  bluish purple rings arround both eyes due to chronic mid face venos congestion “ Dennie’s Lines:  Skin folds under the eyes “ Allergic Salute:  transverse nasal crease from chronic rubbing *Sinuses: r/o sinusitis  purulent discharge, tenderness and impaired transillumination
  • 5. Allergic Rhinitis  Diagnosis : * Skin testing  Gold Standard * RAST  use this if unable to do a skin test or if its contraindicated * CBC  may show eosinophilia * IgE levels are elevated  D/D : 1) Nasal causes of Rhinitis : Nonallergic rhinitis ( eosinophila synd), Nasal polyps, Vasomotor rhinitis, infectious rhinitis, Rhinitis medicamentosa 2) Medications: Aspirin, Clonidine, Hydralazine, Labetalol, propranolol, tearazosin, OC pills  Management  Do Skin test / RAST and find the responsible Allergen. Advise the pt to avoid the allergen. “Avoid pets in the bed if its found to be animal dander”  Intranasal Steroids ( are the drug of choice for pts with chronic symptoms. Can be used prn but most effective when used as maintainance therapy  fluticasone, beclomethasone) , Antihistamines ( cetrizine, loratidine) , Saline nasal drops , decongestants ( pseudoephedrine), nasal cromolyn
  • 6. Allergic Rhinitis Antihistamines vs. Nasal Corticosteroids. The majority of studies favor the use of intranasal corticosteroids over sedating or nonsedating antihistamines for relief of symptoms of nasal allergy. These results are true for seasonal and perennial allergic rhinitis. ( antihistamines are used for immediate symptom relief) Immunotherapy: Immunotherapy is indicated in patients who present with any of the following characteristics:  Insufficient control by pharmacotherapy;  Insufficient control of symptoms;  A desire not to take medication;  Medication produces undesirable side effects; and  A desire to avoid long-term pharmacotherapy (with intranasal steroids)
  • 7. Case Study  You are treating an 18-year-old white male college freshman for allergic rhinitis. It is September and he tells you that he has severe symptoms every autumn, which impair his academic performance. He has a strongly positive family history of atopic dermatitis. Which one of the following medication is considered optimal treatment for this condition?  Intranasal glucocorticoids  Intranasal cromolym sodium  Intranasal decongestants  Intranasal antihistamine
  • 8. Ans  Topical intranasal glucocorticoids are currently believed to be the most efficacious medications for the treatment of allergic rhinitis. They are far superior to oral preparations in terms of safety.  Cromolyn sodium is also an effective topical agent for allergic rhinitis; however, it is more effective if started prior to the season of peak symptoms.  Because of the high risk of rhinitis medicamentosa with chronic use of topical decongestants, these agents have limited usefulness in the treatment of allergic rhinitis.  Some of the newer oral antihistamines have been found to be comparable in efficacy to intranasal steroids, but their use slightly increases the incidence of adverse effects and drug interactions. They are not as useful for congestion as they are for sneezing, pruritis, and rhinorrhea. Newer agents are relatively free of sedation. Overall, they are not as effective as topical glucocorticoids. Azelastine , an intranasal antihistamine, is effective in controlling symptoms but can cause somnolence and has a very bitter taste.
  • 9. Vasomotor Rhinitis  Diagnosis of exclusion  Symptoms similar as Allergic Rhinitis  has rhinorrhea, congestion, nasal obstruction ( normal nasal exam, normal Ig E and Normal skin test/ RAST )  No specific test is available to diagnose vasomotor rhinitis  First exclude allergic rhinitis as the cause of symptoms by using conventional skin testing or by evaluation for specific IgE antibodies to known allergens.  Rx: Stepwise Approach ( next slide )  Pregnancy Step 1: Nasal Saline Step 2: Intranasal Atrovent (Pregnancy Category B) Traditional oral antihistamines have no established beneficial effect in patients with vasomotor rhinitis and may be associated with sedation. Newer, less-sedating antihistamines also have no proven effectiveness for vasomotor rhinitis, and their administration delays proper treatment while incurring significant cost and burden to the health care system. Topical antihistamines are used as first choice if symps are rhinorrhea, sneezing, post nasal drip
  • 10.
  • 11. Rhinitis Medicamentosa Pathophysiology  Associated with topical agent use >5-7 days  Tachyphylaxis associated with medications  Nasal Decongestants (Afrin, Neo-Synephrine)  Other associated medications  Reserpine  Oral Contraceptive pills  Inderal  Aldomet Symptoms  Rebound nasal Congestion after nasal Decongestant Signs  Fiery red edema at nasal mucosa Management  Intranasal Steroid  Withdrawal of nasal Decongestant
  • 12. Acute & Chronic Sinusitis Criteria for diagnosis:  Maxillary toothache  Purulent nasal secretion  History of colored Nasal discharge  Poor response to nasal Decongestants  Abnormal Sinus Transillumination If 4 or more criteria +  diagnosis is definite If 2 or 3 crieria +  Diagnosis is intermediate  recommended initial study Sinus CT If less than 2 criteria  negative for sinusitis  Most common is maxillary sinusitis. Next common is Frontal. Ethmoids are most commonly affected in children. Spenoids has highest risk of intracranial spread  Symptoms may last as long as 4 weeks in acute sinusitis, Symptoms b/w 4-8 weeks is subacute ans symptoms persisting > 8 weeks is chronic sinusitis. In recurrent sinusitis, there are 3 or more episodes of acute sinusitis per year, and different episodes may be caused by different organisms.  Signs  Diagnostic tests  Step wise Treatment  Complications
  • 13. Acute & Chronic Sinusitis Signs : Nasal Mucosa erythema and boggy due to edema  Contrast with Allergic Rhinitis (pale, boggy mucosa)  Nasal exam to view pus discharge from lateral wall Instruments Nasal speculum (minimal visualization) , Flexible Nasolaryngoscopy  Middle Meatus (hiatus semilunaris)  Drains Maxillary, Frontal, and Anterior Ethmoid  Consider local Topical Decongestant application  Superior Meatus (Rarely discharge is seen)  Drains posterior ethmoid sinus  Turbinates enlarged  Sinus tenderness to percussion  Sinus Transillumination in darkened room  Frontal and maxillary sinus
  • 14. Acute & Chronic Sinusitis  Symptoms suggesting bacterial etiology  Symptoms persist beyond 10 to 14 days, Remember that under 10 days, viral sinusitis predominates,  By day 10, 40% of sinusitis resolves spontaneously  0.5% of viral URIs develop into bacterial sinusitis  Symptoms worsen after 5-7 days ( “double” sickening)  purulent nasal discharge  “Unilateral” maxillary sinus tenderness  Maxillary tooth or facial pain (esp. if unilateral)
  • 15. Acute & Chronic Sinusitis  Don’t culture nasal swabs  not cost effective  Diagnosis is clinical in Acute Sinusitis  Indications for Imaging  Complicated sinusitis , Chronic or recurrent sinusitis , Sinusitis refractory to maximal medical therapy  Imaging is not needed in routine cases  Empiric therapy for 1-2 courses is appropriate 1. Sinus X-Ray (Sinus CT preferred)  Plain radiographic signs consistent with sinusitis include greater than 6 mm of mucosal thickening in adults and 4 mm in children, greater than 33% loss of air space volume in the maxillary sinuses, or opacification–air- fluid levels.  Single Waters' View X-Ray is sufficient  Indication (rarely indicated unless CT not available)  Complicated Acute Sinusitis & Suspected Chronic Sinusitis  Sinus CT (gold standard) Indications  Osteomeatal complex occlusion  Complicated acute sinusitis  orbital cellulitis etc  Chronic Sinusitis  Recurrent Sinusitis  Allergic Fungal Sinusitis  Sinus MRI  No advantage over Sinus CT (and more false positives)  Indications : Suspected neoplasm and Fungal Sinusitis
  • 16. Acute Sinusitis - Complications  Unless severe symptoms of acute sinusitis develop, such as fever, facial pain or tenderness, or periorbital swelling, antibiotics should be withheld for 10 to 14 days.  Complications : Orbital Cellulitis, Meningitis, Extradural abscess , Subdural abscess , Brain abscess , Osteomyelitis and Cavernous Sinus Thrombosis  Symptoms: Red Flag (consider immediate ENT referral)  High Fever over 102.2 F (39 C) or peristent fever  Visual complaints (e.g. Diplopia)  Periorbital edema or erythema ( check for EOMs  ?pain)  Mental status changes  Severe facial or dental pain  Infraorbital hypesthesia  consider referral in immunodeficiency or if persistent symptoms despite treatment
  • 17. Sinusitis - Treatment General Measures  Symptomatic relief : Warm, moist compresses over sinuses , Tylenol  Nasal Saline spray (2% buffered saline)  Effective Decongestant  Also use as pretreatment prior to Intranasal Steroid  Effective in recurrent Sinusitis when used daily  Systemic Decongestant: Pseudoephedrine  Mucolytic : Guaifenesin (e.g. Mucinex) 600 to 1200 mg PO bid  there is no evidence that mucolytics are useful adjuncts  Topical Decongestant (Maximum of 3 days of use)  Oxymetazoline or Phenylephrine (Neo-Synephrine)  Intranasal Steroid (treat for 3-6 weeks minimum)  Chronic Sinusitis  Nasal Polyp  Avoid Antihistamines!!  Dries secretions and Impedes osteomeatal complex drainage
  • 18. Sinusitis - Treatment ANTIBIOTICS:  Indicated only in acute bacterial Sinusitis  Protocol Antibiotic course  Minimum course: 10-14 days Longer course for persistent symptoms: 28 days  Change antibiotic if no improvement in 3 days REMEMBER THAT Beta-lactamase resistance in acute cases: <30% , Beta- lactamase resistance in chronic cases: 40-50%  First-Line  Indications to start on first-line agents Mild to moderate symptoms , No daycare exposure & No recent antibiotic use  Amoxiicillin  Disadvantages: Misses Beta-lactamase producers : Haemophilus Influenzae , Moraxella catarrhalis & Penicillin Resistant Pneumococcus (increasing)  Trimethoprim Sulfamethoxazole (Bactrim) No longer recommended as first-line agent , Higher resistance rate than other agents  Disadvantages : Misses Staphylococcus , Risk of Toxic Epidermal Necrolysis& Risk of Steven's Johnson Syndrome
  • 19. Sinusitis - Treatment  Second-Line  Indications to start on second-line agents : Severe symptoms Daycare exposure , Recent antibiotic use  Amoxicillin-Clavulanate (Augmentin ) or Cefuroxime (Zinacef) , Cefpodoxime  Avoid Cefixime ( poor Gram + coverage )  Third Line recommendation If no improvement with above a) Consider adding Flagyl to second-line agents b) Consider second-line agent for longer course (4 week) c) Switch to Fluoroquinolone (avoid under 16 yrs of age ), Moxifloxacin or Gatifloxacin (Tequin)
  • 20. Sinusitis - Treatment Management : Penicillin or Cephalosporin Allergy  Macrolide antibiotics (High bacterial resistance rate)  Erythromycin , Azithromycin (Zithromax) or Clarithromycin (Biaxin)  Trimethoprim-Sulfamethoxazole (Bactrim)  Increasing bacterial resistance, So other agents are preferred for Sinusitis  Clindamycin  Consider in combination with Rifampin if severe , Poor efficacy against Gram Negative Bacteria  Fluoroquinolones ( avoid under age 16 years )
  • 21. Sinusitis - Treatment  “Unless severe symptoms of acute sinusitis develop, such as fever, facial pain or tenderness, or periorbital swelling, antibiotics should be withheld for 10 to 14 days. Although the primary therapy for acute bacterial sinusitis is antibiotics, increasing resistance to penicillin may necessitate the use of alternative antibiotics. The choice of antibiotics is based on predicted efficacy, cost, and adverse effects. A 10- to 14- day course is generally adequate for acute disease, but shorter courses may be indicated for newer antibiotics. If there is no improvement in 3 to 5 days, an alternative antibiotic should be considered” ( guidelines, journal of clinical immunology, 2006)  Primary therapy for acute bacterial sinusitis is antibiotics with a 10- to 14-day course considered adequate. Amoxicillin is a drug of choice with trimethoprim-sulfamethoxazole an alternative.  If no response occurs within 3 to 5 days, a change to high-dose amoxicillin-clavulanate, cephalosporins, or macrolides may be indicated.  In areas of high antibiotic resistance or with failure to improve after 21 to 28 days, broad spectrum single agents should be considered, such as amoxicillin-clavulanate, cefuroxime, or cefpodoxime, or use of anaerobic coverage, such as clindamycin or metronidazole.  Nasal corticosteroids are indicated in acute and chronic sinusitis and short-term adjunct oral steroids may be used after failure of response or when nasal polyps are present.  Saline nasal sprays may help to reduce crusting!!
  • 22. Acute Pharyngitis  Symptoms: Sore throat , Dysphagia & Odynophagia (pain with swallowing)  Generalized symptoms : Fever, Chills, Malaise, Headache , Abdominal Pain , Nausea or Vomiting  Symptoms suggestive of viral illness: Coryza, Conjunctivitis & Hoarseness  Signs  Viral  Non-exudative pharyngeal erythema  Exception: Tonsillar exudate in Mononucleosis (EBV)  Vesicular OR ulcerative oral lesions  Conjunctivits in Adenovirus and Kawasaki Disease  Group A Streptococcus and other bacteria  clues are Enlarged tonsils with or without exudate , Petechiae on Soft Palate (pathognomonic) , Erythema , Tender cervical Lymphadenopathy  Strawberry Tongue (in Scarlet Fever)  Peritonsillar Cellulitis or Peritonsillar Abscess  Suspect Unilateral erythema of Soft Palate , Uvula deviated , Dysphagia, Odynophagia & Fever  Diphtheria  Suspect when Gray membranous exudate covers tonsils and pharynx or Exudate bleeds easily on removal  Kawasaki Disease  Suspect when Pharyngitis with strawberry Tongue in age <5 years , Non-purulent Conjunctivitis (also in Adenovirus) & Palmar erythema and cracked red lips after 3 days
  • 23. STREP THROAT  Acute Pharyngitis caused by Group A beta hemolytic streptococci.  Most common in children 5-12 yr old  Infectivity  Decreases 1-3 days after antibiotic started  Return to School and day care recommendations  Child should receive Antibiotics for minimum of 24 hours and Afebrile
  • 24. Strep Throat Complications  Non-suppurative  Rheumatic Fever  we Rx Strep Throat to prevent this. ABX Rx does not prevent PSGN  Acute Post-Streptococcal Glomerulonephritis ( PSGN)  Suppurative  Peritonsillar Abscess  Suppurative Otitis Media  Cervical lymphadenitis  Acute Sinusitis  Mastoiditis  Meningitis  Bacteremia  Endocarditis  Pneumonia
  • 25. Strep Throat – Strep Score Original Criteria (interpretation below based on these)  1 point for each  Tonsillar exudate  Tender, anterior cervical adenopathy  Cough absent  Fever present  Modifiers : Age younger than 15 years: +1 point, Age 15 to 45 years: 0 points & Age over 45 years: -1 points ER and OP probability:  Score 0: Streptococcus probability 1% (3% in ER)  Score 1: Streptococcus probability 4% (8% in ER)  Score 2: Streptococcus probability 9% (18% in ER)  Score 3: Streptococcus probability 21% (38% in ER)  Score 4: Streptococcus probability 43% (63% in ER)
  • 26. Strep throat (?) - Approach  Strep Score 4 (or Strep Score 2 if patient unreliable)  Treat with antibiotics  Strep Score 2 to 3: Perform rapid antigen test  Antigen test positive: Treat with antibiotics  Antigen test negative: Throat Culture (Requires 24 hour minimum for adequate growth ) most specific (99%). Sensitivity 90%. Not recommended as primary test due to 24 hour delay . Remember that –ve Rapid strep does not rule out Strep throat  Strep Score 0 to 1  Provide Pharyngitis Symptomatic Treatment  salt water gargles, sucking candies, ibuprofen
  • 27. Strep throat - Antibiotics  Penicillin is the first choice ( coz its strep)  penicillin VK 500 mg  If using this standard course duration is 10 days. Alternatively use Amoxicillin 500 bid in adults/ 10 days  Alternative antibiotics : Five days of alternative antibiotics effective Amoxicillin Clavulanate (Augmentin) , Ceftibuten, Cefuroxime, Clarithromycin or Erythromycin estolate ( for pen allergic pts)  Non-Compliant pts  single dose benzathine penicllin IM  Recurrent Strep Throat  Cephalosporins are choice ( Keflex  cephalexin 500 bid) or can use Augmentin
  • 28. Etiologies for recurrent Streptococcal Pharyngitis  Poor Compliance with oral medications (most common)  Day 3: 50% stopped antibiotics  Day 6: 70% stopped antibiotics  Day 9: 80% stopped antibiotics  Families reporting taking all the medication: 80%  Repeat exposure in crowded conditions  School , Daycare & Home or workplace  Eradicated protective throat flora by prior antibiotic  a-hemolytic Streptococcus is protective normal flora  Cephalosporins apparently do less harm  Selected beta-lactam resistance by prior antibiotic  Consider Augmentin for 10 day course  Suppressed Immune response from prior antibiotics  Antibiotic Resistance  Penicillin resistance is infrequent in strep throat  Macrolide (Erythromycin, Biaxin, Zithromax)  Resistance 2-8% in U.S.  Chronic Pharyngeal Carriage of Streptococcus pyogenes  Consider Pharyngitis due to another cause
  • 29. School Attendance ADVISE TO PARENTS!! High Yield!
  • 30. Contraindications to school attendance  Infectious  Fever  Vomiting or dehydration Indications for school return in viral infection  Viral infection examples : Influenza, Rhinovirus (Common Cold) , Fifth Disease, Hand Foot and Mouth Disease  Indications to return to school  No fever and Child must practice good hygiene (i.e. hand washing) Indications for school return in bacterial infection  Bacterial infection examples: Impetigo, Bacterial Conjunctivitis, Streptococcal Pharyngitis (Strep Throat)  Indications to return to school  after Antibiotics for 24 hours Indications for school return in specific conditions  Chicken Pox  All lesions have crusted over  Head Lice  After anti-lice shampoo and manual nit removal  Pinworm  Day after Pyrantel, Mebendazole, or Albendazole  Vomiting  24 hours after last Emesis Conditions allowing immediate school return  Viral Conjunctivitis (Pink Eye)  Otitis Media (ear infection)
  • 31. LRTIs
  • 32. Acute Bronchitis  Usually viral  Treat with antibiotics if second sickening or if associated with COPD exacerbation
  • 33. Pneumonia  Community Acquired Pneumonia  Typical etiology : S.pneumoniae. Others: H.influezae, M.catarrhalis  Atypical pneumonia : Legionella, mycoplasma, chlamydia  Health care associated  - Nursing home acquired, - Hospl acquired , Ventilator associated
  • 34. Severe Pneumonia  CURB 65 predicts highly severe pneumonia  RR>30  DBP<60mmhg  BUN>20  CONFUSION  Age>65 yrs
  • 35. Whether to admit?  Most Pneumonias are treated as Outpatient  Admission is required if:  Those with underlying immunosuppression ( chemotherapy, HIV)  Elderly patients > 65 yrs  Pts with altered mental status  Those with hemodynamic ( shock) or respiratory compromise ( tachypnea, respiratory failure)  Pts with poor social support ( homeless) or without ability to self supervise
  • 36. Where to Admit?  Admission to ICU is needed if: LOOK AT VITALS!  Hypotension (SBP<90)  Hemodynamic Instability/ Shock (map<60)  Hypoxemia<60  Organ failure ( ARF etc)  Impending respiratory failure that may require mechanical ventilation ( persistent tachypnea, desaturation etc)  Deteriorating comorbid illness ( CHF, renal failure etc)  Heart failure, severe copd exacerbation, Diabetic complications (?DKA)
  • 37. Community acqd  Outpatient  Rx with Macrolide ( azithromycin) or newer Quinolones  Inpatient  Rx with Ceftriaxone + macrolide or Fluoroquniolone alone
  • 38. Health Care Associated Pneumonia  Either NH associated or hospital acquired  NH associated pneumonia may have MRSA and Gram –ve bacteria as etiologies ( E.coli, proteus, klebsiella)  so emperically Rx with Vanco +Zosyn (pip/tazo) before sputum culture results are available. Once Cx and sensitivity are obtained d/c the antibiotic that’s not needed  Hospital acquired pneumonia is the one that develops 48 hrs after hospitalization  has a different spectrum of bacteria ( MRSA + resistant gram –ves)  initially can start VANCO + Zosyn before cx results are available. If severe, use imipenem instead of Zosyn (pip/tazo)
  • 39. VAP  Ventilator Acquired Pneumonia  Pneumonia that develops 48 hrs after intubation  diagnosed by c/f like fever, leucocytosis, newly developed CXR infiltrates and purulent ET tube secretions  the spectrum of bacteria here is more resistant i.e; MRSA+ Resistant gram –ves including P.aeruginosa  start emperical VANCO+Imipenem ( do not take chance with resistance here)  Culture ET secretions, Get a CXR  Bronchoscopy may be required in pts showing no response and also to differentiate b/w colonization vs. Infection  Recovery of bacteria in high concentrations from bronchoalveolar lavage (BAL) >10,000 col/ml helps in differentiation of non infectious from infectious causes of pulmonary infiltrates ( i.e; if the colonies are this high think of infection other wise think of non infectious cause like ARDS, CHF etc for explaining these pulmonary infiltrates in vent patients)
  • 40. PCP  Pneumocystis Carinii pneumonia  Seen in immunocompromised pts Pts who are HIV,{CD4< 200} Immunocompromised and pts on high dose steroids ( prednisone>20mg/d),  Symps: dry cough, fever, chills, sob, chestpain  Needs high suspicion for diagnosis  LDH will help when in doubt, Gallium scan will help too  CXR  Interstitial infiltrates, LDH high, Ground glass appearance on CT scan, Sputum for silver staining,  if sputum –ve, bronchoscopy needed for diagnosis where you do Bronchoalveolar lavage – silver staining  Get an ABG  Rx  Simple pcp  oral bactrim • Severe pcp  iv bactrim + steroids ( make sure u give enough i.e; prednisone 40mg bid or solumedrol 30mg iv bid  Po2 < 70mm hg/ increased A-a > 35are indication for steroid Rx) • Sulfa allergy  aerosolized pentamidine
  • 41. Case Study  A 36-year-old woman is admitted to the medical intensive care unit because of respiratory depression resulting from a barbiturate overdose. She is intubated and mechanical ventilation is begun. Physical examination, except for her comatose condition, is unremarkable. Chest radiography and arterial blood gases are within normal limits. Which of the following will minimize her risk of developing a nosocomial infection?  ( A ) Ventilator tubing changes every 12 hours  ( B ) Elevation of the head of the bed to 45 degrees  ( C ) Ceftriaxone, intravenously  ( D ) Oropharynx polymyxin B spray every 8 hours  ( E ) Enteral feedings by nasogastric tube
  • 42. Ans.B  Patients who are mechanically ventilated in the supine position have an approximately six fold increased risk of developing pneumonia compared with patients maintained in a semirecumbent position. Elevation of the patient's head to 45 degrees may reduce aspiration and nosocomial pneumonia.  Nosocomial pneumonia is a major cause of morbidity and mortality in mechanically ventilated patients.
  • 43. Case Study  A 21-year-old woman with cystic fibrosis diagnosed at 6 months of age is evaluated because of increased dyspnea, blood-streaked purulent sputum, decreased energy, and a 1.8-kg (4-lb) weight loss of 4 weeks’ duration. She was last treated with intravenous antibiotics 12 months ago. Her sputum cultures repeatedly grow a mucoid strain of Pseudomonas aeruginosa. Her forced expiratory volume in 1 second (FEV1) has decreased by 400 mL in 6 months and is now 47% of predicted. Chest radiography shows diffuse bronchiectatic changes but no consolidation. She takes replacement pancreatic enzymes, albuterol nebulization three times daily, inhaled recombinant human Dnase once daily,and uses a flutter device to aid expectoration. Which of the following is the best management option at this time?  ( A ) Tobramycin, inhaled, twice daily  ( B ) Increase Dnase, albuterol nebulizations, and chest physiotherapy  ( C ) Piperacillin and tobramycin, intravenously  ( D ) Ciprofloxacin, orally, and tobramycin, inhaled, twice daily  ( E ) Bronchoscopy
  • 44. Ans.C  Patients with cystic fibrosis and a bronchitic exacerbation of chronic bronchiectasis with Pseudomonas aeruginosa require intravenous antibiotics with two antipseudomonal agents for 2 to3 weeks.  The use of aerosolized tobramycin is indicated for patients with chronic Pseudomonas colonization and is associated with long-term improvement in forced expiratory volume in 1 sec (FEV1) of about 10%, as well as decreased need for hospitalization and intravenous antibiotics, but it is not sufficient for an exacerbation.
  • 45. Pulmonary Embolism  Causes  Clinical features  chestpain, sob, cough, leg swelling  EKG – Sinus tachy, S1Q3T3  ABGs – resp alkalosis  Diagnosis  v/q, d-dimer, high resolution CT (Spiral CT scan) ( Serum D-dimer < 500ng/ml  Treatment – if shock or if no shock , if anticoagulation is contraindicated
  • 46. PE on EKG  Pulmonary embolism (acute cor pulmonale): Look for new signs of new signs of tachycardia; complete or incomplete RBBB; the S1Q3T3 pattern; and/or right axis shift. There may be inferior or RV injury patterns. The most common cause of an S1Q3T3 pattern is a completed inferior MI.  Get a Right sided EKG.
  • 47. PE on CXR  Initial CxR may be NORMAL. ( PIOPED study showed that only 12% of CXRs in pts with angiographically proven PE were interpreted as normal)  May show – Collapse, atelectasis, consolidation, small pleural effusion, elevated diaphragm.  Pleural based opacities with convex medial margins are also known as a Hampton's Hump
  • 48. Hampton's Hump  Pleural based opacities with convex medial margins are also known as a Hampton's Hump. This may be an indication of lung infarction. However, that rate of resolution of these densities is the best way to judge if lung tissue has been infarcted. Areas of pulmonary hemorrhage and edema resolve in a few days to one week. The density caused by an area of infarcted lung will decrease slowly over a few weeks to months and may leave a linear scar
  • 49. PE on CXR  Westermark sign – Dilatation of pulmonary vessels proximal to embolism along with collapse of distal vessels, often with a sharp cut off.
  • 50. Pulmonary Embolism with Infarction  Consolidation  Cavitation  Pleural effusion (bloody in 65%)  No air bronchograms  “Melting” sign of healing  Heals with linear scar
  • 51. Case Study  A 56-year-old man is evaluated in the emergency department because of progressive swelling of the right lower extremity during the previous 5 days and right-sided pleuritic chest pain and dyspnea beginning 1 to 2 hours ago.On physical examination, his temperature is 38.2 °C (100.8 °F), pulse rate is 105/min, respiration rate is 28/min, and blood pressure is 160/80 mm Hg. Cardiac and pulmonary examinations are unremarkable. Arterial blood gases with the patient breathing room air are PO2, 78 mm Hg; PCO2, 30 mm Hg; and pH, 7.48.Electrocardiography shows sinus tachycardia and nonspecific ST-T wave changes, and chest radiography is normal.Ventilation-perfusion scanning shows two unmatched segmental defects. The D-dimer value is three times the upper limit of normal.  Which of the following is the most appropriate course of action?  ( A ) Heparin  ( B ) Helical computed tomography with contrast  ( C ) Noninvasive studies of the lower extremities  ( D ) Pulmonary angiography
  • 52. Key Point In patients with a high pretest probability of pulmonary embolism and high-probability ventilation-perfusion scanning, additional diagnostic testing is not necessary before initiating therapy.
  • 53. Pneumothorax  Causes – Trauma, bulla rupture, necrotizing pneumonia  Clinical features  chest pain, dyspnea, shock  Ventilator associated Pneumothorax ? sudden hypotension while on vent  look at peak and plateau pressures  Treatment  needle thoracentesis, needle thoracostomy, tube thoracostomy
  • 54. ARDS  Diffuse pulmonary capillary damage leading to increased permeability of alveolar capillaries  pulm edema  Criteria  1) There should be a cause 2) PO2/Fio2 ( in liter) Ratio, Po2/Fio2 < 300  ALI, <200 ARDS 3) B/L CXR infiltrates 4) Should not be due to CHF; Clues: 2D ECHO EF Good/ no diastolic dysfunction. If in doubt whether CXR infiltrates are due to CHF or ARDS  measure PCWP ( Swan Ganz insertion )  Ventilation strategies  Low Vt ( 6cc/kg) (  prevent overdistension injury) and High PEEP strategy ( reduce derecruitment injury)  Causes  TTP, Sepsis, Shock, Aspiration pneumonia, chemical pneumonitis, Drugs like Heroin, Pancreatitis, Burns, Drowning
  • 55. Case Study  A 58-year-old man is admitted to the intensive care unit with increasing dyspnea after developing influenza symptoms 3 days previously. On physical examination, his temperature is 39.1 °C (102.3 °F), pulse rate is 110/min, and bloodpressure is 135/83 mm Hg. He weighs 73 kg (161 lb). He is using accessory muscles of respiration, and he has finecrackles throughout all lung fields. Cardiac examination is unremarkable, and no edema is noted. Chest radiographyshows diffuse infiltrates throughout both lungs with patchy areas of consolidation. The patient has a history of moderate obstructive lung disease secondary to smoking. Several months before hospitalization his forced expiratory volume in 1sec (FEV1) was 53% of predicted, and he had normal oxygen saturation and no hypercapnia.Shortly after hospitalization, he is intubated because of increasing hypoxemia and hypercapnia. Subsequent arterial blood gases with the patient breathing 100% oxygen and 10 cm H2O of positive end-expiratory pressure are PO2, 68mm Hg; PCO2, 65 mm Hg; pH, 7.23; and bicarbonate, 26 meq/L. Tidal volume is 450 mL, respiration rate is 25/min,inspiratory flow rate is 100 L/min, and inspiratory/expiratory ratio is 1:5. Peak airway and plateau ventilatory pressures are 48 cm H2O and 32 cm H2O.  Which of the following is the best option?  ( A ) Increase the tidal volume  ( B ) Increase the respiration rate  ( C ) Increase the positive end-expiratory pressure  ( D ) Decrease the positive end-expiratory pressure  ( E ) Administer sodium bicarbonate, intravenously
  • 56. Key Points  Ans. E  In patients with acute respiratory distress syndrome, mortality was significantly improved by ventilating patients with tidalvolumes of 6 mL/kg of ideal body weight and keeping plateau ventilatory pressure at =30 cm H2O.  If changes in respirator settings required to prevent hypercapnia have associated untoward effects, it is reasonable to allow arterial PCO2 to rise and, if necessary, prevent acidemia by administration of buffer  as in this case!! ( don’t increase tidal volume here  low Vt is good for this  remember Permissive Hypercapnia)  Increasing PEEP is not good here. Raising PEEP is undesirable because this will narrow the pressure difference between the plateau ventilatory pressure and the PEEP, decreasing the pressure available to deliver the tidal volume. This will reduce the tidal volume and exacerbate hypercapnia. PEEP should remain unchanged because the patient has acceptable oxygenation with the present setting. The level of PEEP cannot be reduced since reduction likely will lead to unacceptable hypoxemia. The patient is barely at an acceptable level without any reduction.  Increasing the respiration rate likely will increase auto-positive end-expiratory pressure (PEEP) in this patient with chronic obstructive pulmonary disease ( they have proloned expiration!) by “breath stacking,” that is, delivering the next breath before the previous breath is completely expired.This will also raise the plateau ventilatory pressure above a desirable range.
  • 57. Case Study  A 57-year-old man with severe chronic obstructive pulmonary disease is hospitalized with respiratory distress of 12 hours’ duration. Arterial blood gases with the patient breathing 35% oxygen through a face mask are PaO2, 50 mm Hg; PaCO2, 70 mm Hg; and pH, 7.24. When seen as an outpatient 1 month previously, his arterial blood gases while breathing room air were PaO2, 58 mm Hg; PaCO2, 50 mm Hg; and pH, 7.37. Despite maximal therapy, mechanical ventilation is required. During controlled breaths, his peak airway pressure is 25 cm H2O, and plateau ventilatory pressure is 12 cm H2O. The arterial blood gases are checked after 1 hour. Which of the following is the most desirable set of arterial blood gas values?  ( A ) Pa O2, 50 mm Hg; PaCO2, 45 mm Hg; pH, 7.44; FIO2, 0.3  ( B ) Pa O2, 65 mm Hg; PaCO2, 52 mm Hg; pH, 7.38; FIO2, 0.4  ( C ) Pa O2, 65 mm Hg; PaCO2, 40 mm Hg; pH, 7.48; FIO2, 0.4  ( D ) Pa O2, 90 mm Hg; PaCO2, 60 mm Hg; pH, 7.32; FIO2, 0.5  ( E ) Pa O2, 133 mm Hg; PaCO2, 55 mm Hg; pH, 7.41; FIO2, 0.6
  • 58. Ans.B  When instituting mechanical ventilation in a patient with chronic hypercapnia, it is critical to avoid the development of respiratory alkalemia secondary to overventilation, and ventilator settings should have pH as a target, rather than PaCO2.  When seen 1 month before hospitalization, the patient had chronic carbon dioxide retention. When instituting mechanical ventilation in a patient with hypercapnia, it is critical to avoid the development of respiratory alkalemia secondary to overventilation. Severe alkalosis in this setting may result in cardiovascular instability, arrhythmias, andseizures. Ventilator settings should have pH as a target, rather than PaCO2.
  • 59. Acute Pulmonary Edema  Treatment  morphine, loop diuretics in LVF, Ventilation strategies in ARDS and Hemodialysis when indicated  Causes  ARDS, Acute LVF, Fluid Overload, Missing Hemodialysis
  • 60.  A 58-year-old man is admitted to the intensive care unit with increasing dyspnea after developing influenza symptoms 3 days previously. On physical examination, his temperature is 39.1 °C (102.3 °F), pulse rate is 110/min, and blood pressure is 135/83 mm Hg. He weighs 73 kg (161 lb). He is using accessory muscles of respiration, and he has fine crackles throughout all lung fields. Cardiac examination is unremarkable, and no edema is noted. Chest radiography shows diffuse infiltrates throughout both lungs with patchy areas of consolidation. The patient has a history of moderate obstructive lung disease secondary to smoking. Several months before hospitalization his forced expiratory volume in 1 sec (FEV1) was 53% of predicted, and he had normal oxygen saturation and no hypercapnia. Shortly after hospitalization, he is intubated because of increasing hypoxemia and hypercapnia. Subsequent arterial blood gases with the patient breathing 100% oxygen and 10 cm H2O of positive end-expiratory pressure are PO2, 68 mm Hg; PCO2, 65 mm Hg; pH, 7.23; and bicarbonate, 26 meq/L. Tidal volume is 450 mL, respiration rate is 25/min, inspiratory flow rate is 100 L/min, and inspiratory/expiratory ratio is 1:5. Peak airway and plateau ventilatory pressures are  48 cm H2O and 32 cm H2O.  Which of the following is the best option for improving this patient’s acid–base disorder?  ( A ) Increase the tidal volume  ( B ) Increase the respiration rate  ( C ) Increase the positive end-expiratory pressure  ( D ) Decrease the positive end-expiratory pressure  ( E ) Administer sodium bicarbonate, intravenously
  • 61. Ans.  In patients with acute respiratory distress syndrome, mortality was significantly improved by ventilating patients with tidalvolumes of 6 mL/kg of ideal body weight and keeping plateau ventilatory pressure at =30 cm H2O.  If changes in respirator settings required to prevent hypercapnia have associated untoward effects, it is reasonable to allowarterial PCO2 to rise and, if necessary, prevent acidemia byadministration of buffer.
  • 62. COPD
  • 63. COPD – Screening with Spirometry  Consider screening smokers or former smokers with certain clinical characteristics for COPD with pulmonary function testing.  In patients who smoke or have smoked, consider obtaining screening spirometry readings to document obstruction if they give a history of cough or sputum production or have findings compatible with emphysema on chest x-ray.  Obtain spirometry readings if the patient has limiting symptoms such as dyspnea inappropriate to the level of activity, frequent episodes of acute bronchitis related to upper respiratory tract infections (i.e., a possible acute exacerbation), difficulty sleeping due to cough and dyspnea, and general diminished activity levels and energy from difficulty in breathing.  If the patient has no other clinical characteristics for COPD, but has a significant history of smoking, consider obtaining spirometry readings because significant pulmonary function impairment may still be present.
  • 64. COPD Exacerbations  COPD – Chr.bronchitis, Emphysema – blue bloaters, Pink puffers  COPD exacerbations  History, Clinical exam, get pulse ox,  Mild, Moderate, Severe  classify depending on 3 criteria (Increase in amount of sputum, Increased sputum purulence, worsening dyspnea)  Mild exacerbation ( 1 of above criteria)  use simple antibiotics like Bactrim or Doxycycline  Moderate exacerbation ( 2 of above criteria) use 2nd line Antibiotics like quinolones, b-lactam/clavulanate ( Augmentin)  Severe Exacerbation ( 3 of above criteria)  Look at the ABGs, o2 inhalation, nebulizer with ipratropium + albuterol  caution with o2, o2 inhalation only as much as to maintain sao2>90%.  If no response , non invasive ventilation ( positive pressure ventilation, BIPAP)  Pt must be cooperative for this  if altered mental status, no response with non invasive ventilation  Intubate and ventilate.  Remember to get ABGs after u place a COPD guy on oxygen  Beware of posthypercapnic alkalosis  if develops, acetazolomide  COPD exacerbation  ? Ask urself secondary to what  Acute bronchitis, pneumonia  use of antibiotics in COPD exacerbations  Steroids is a MUST  methylprednisolone high doses 125mg q6hrs, then tapering steroids
  • 65. When To Admit? Indications for hospitalization of patients with COPD:  Patient has acute exacerbation plus one or more of the following:  Inadequate response of symptoms to outpatient management  Inability to walk between rooms (patient previously mobile)  Inability to eat or sleep due to dyspnea  Conclusion by family, physician, or both that patient cannot manage at home and supplementary home care resources are not immediately available  Presence of a high-risk comorbid condition, pulmonary (e.g., pneumonia) or nonpulmonary  Prolonged, progressive symptoms before emergency department visit  Altered mentation  Worsening hypoxemia  New or worsening hypercarbia  Patient has new or worsening cor pulmonale unresponsive to outpatient management  A planned invasive surgical or diagnostic procedure requires analgesics or sedatives that may worsen pulmonary function  Comorbid conditions (e.g., steroid myopathy or vertebral compression fractures) have worsened pulmonary function
  • 66. Where To Admit?  Admit patients with COPD to an intensive care unit if they meet specific criteria.  Confusion, lethargy, or respiratory muscle fatigue  Persistent or worsening hypoxemia despite supplemental O2 or severe or worsening of respiratory acidosis (pH 7.30); use of supplemental oxygen should be at the lowest flow rate to raise PaO 2 >60 or SaO 2 >90% to avoid hyperoxic hypercapnia  Need for assisted mechanical ventilation, whether through means of tracheal intubation or noninvasive techniques  Severe dyspnea that responds inadequately to initial emergency room therapy
  • 67. COPD – Home Oxygen Therapy  At discharge, evaluate pt for home 02 therapy. Especially at nights when pts may desaturate ( acidosis at nights shifts curve to right). Goal maintain sao2 90 or po2 60  Indications :  Po2<55 or sao2 <85%  Po2 b/w 56 to 59 if corpulmonale or polycythemia ( erythrocytosis) (  these suggest evidence of hypoxia)
  • 68. Lung Volume Reduction Surgery  Consider LVRS for patients whose initial clinical criteria include:  CT scan evidence of bilateral emphysema  Prerehabilitation postbronchodilator TLC and residual volume >/= to 100% and 150% predicted, respectively  Maximum FEV1 </= 45% predicted  PaCO2 </= 60 mm Hg  PaO2 >/= 45 mm Hg  Completion of a pulmonary rehabilitation program  Do not consider LVRS for patients whose clinical criteria include:  FEV1 less than or equal to 20% predicted ( very low for surgery) and either homogenous emphysema or carbon monoxide diffusing capacity less than or equal to 20% predicted (DLCO)  Non-upper-lobe emphysema and high baseline exercise capacity
  • 69. Interpretation of PFT’S  Restrictive vs. Obstructive  FEV1 to FVC Ratio (Normally over 75%)  Not useful if both FEV1 and FVC are normal  Obstructive lung: Moderately to severely decreased  Restrictive lung: Normal or increased Reversibility:  Bronchodilator response (Significant values)  Response suggests reversible component if  FVC or FEV1 improves by 12 to 15% over baseline  FVC or FEV1 increases by at least 200 ml  FEF25-75 improves by 15 to 25% over baseline
  • 70. COPD Outpatient Rx  By MDIs  Ipratropium all the time ( q6hrs) + albuterol as needed. Can use tiotropium because its long acting  Evaluate for home o2 therapy  Steroids/ antibiotics in acute exacerbations only.  ( unlike in Asthma, steroids are not a part of chronic therapy in COPD)  MDIs deliver only fixed dose of drug. Nebulizers deliver larger dose of drug so in exacerbation u start with nebulizer if MDIs don’t work
  • 71. COPD with Asthma  Asthma may be present in about 10% of cases of COPD; however, reversibility of FEV1 alone should never be used to make a diagnosis of asthma in the absence of other supporting evidence such as a childhood history of asthma, atopic symptoms, blood or sputum eosinophilia, or onset of symptoms before substantial history of cigarette smoking
  • 72. COPD in the Young
  • 73.  A 38-year-old man is evaluated because of a morning cough productive of clear sputum, chest tightness, and shortness of breath when walking. He has smoked two packs of cigarettes per day since his teenage years and says that previous chest radiography showed "early emphysema." He is a baker but notes no improvement in symptoms when on vacation. His wife has three indoor cats, and he has an outdoor dog. The patient has normal vital signs. The chest is hyperresonant to percussion, breath sounds are decreased in intensity, and expiration is prolonged. Pulmonary function tests show forced expiratory volume in 1 sec (FEV1) is 45% of predicted, forced vital capacity (FVC) is 65% of predicted, total lung capacity (TLC) is slightly increased (120% of predicted), and diffusing lung capacity for carbon monoxide (DLCO) is moderately reduced (60% of predicted). Chest radiography shows hyperinflation with a suggestion of several small bullae in the lower lung fields.  Which of the following tests is indicated?  ( A ) Sputum Gram stain and culture  ( B ) Methacholine inhalation challenge test  ( C ) Skin tests for allergens and serum precipitins to wheat extract  ( D ) Measurement of serum a 1-antitrypsin level  ( E ) Esophageal pH monitoring for 24 hours
  • 74.  Severe chronic obstructive pulmonary disease in young persons is suggestive of a1- antitrypsin deficiency, and an a1-antitrypsin level should be measured.  Smoking is an important precipitating factor and also increases progression
  • 75. Case Study  A 67-year-old man with longstanding chronic obstructive pulmonary disease (COPD) is hospitalized with a 1-week history of increasing cough productive of large amounts of purulent sputum, low-grade fever, lethargy, and shortness ofbreath.On physical examination, his vital signs are normal except for a temperature of 38.2 °C (100.7 °F) and a pulse rate of 108/min. The neck veins are not distended. The anterior–posterior chest dimension is increased and is hyperresonant to percussion, breath sounds are reduced, and expiration is prolonged.Arterial blood gases are normal except for a PO2 of 62 mm Hg with the patient breathing 28% oxygen through a venturi mask. Chest radiography shows changes compatible with COPD but no acute process.In the emergency department, treatment with inhaled bronchodilators and antibiotics was begun.  Which of the following options is the best choice?  ( A ) Add inhaled fluticasone, every 12 hours  ( B ) Add methylprednisolone, 500 mg intravenously once  ( C ) Add methylprednisolone, 125 mg intravenously every 6 hours for 3 days, then taper over 2 weeks  (D) No need to add steroids in this patient  E) Intubate the patient
  • 76. Key Point Patients with exacerbations of chronic obstructive pulmonary disease (COPD) who receive intravenous corticosteroids and a tapering dose of prednisone over 2 weeks experience shorter hospitalization and less treatment failures. Two weeks of tapering prednisone is just as effective as 8 weeks in treating exacerbations of COPD.
  • 77.  A 57-year-old man with severe chronic obstructive pulmonary disease is hospitalized with respiratory distress of 12 hours’ duration. Arterial blood gases with the patient breathing 35% oxygen through a face mask are PaO2, 50 mm Hg;  PaCO2, 70 mm Hg; and pH, 7.24. When seen as an outpatient 1 month previously, his arterial blood gases while  breathing room air were PaO2, 58 mm Hg; PaCO2, 50 mm Hg; and pH, 7.37. Despite maximal therapy, mechanical  ventilation is required. During controlled breaths, his peak airway pressure is 25 cm H2O, and plateau ventilatory  pressure is 12 cm H2O. The arterial blood gases are checked after 1 hour.  Which of the following is the most desirable set of arterial blood gas values?  ( A ) Pa O2, 50 mm Hg; PaCO2, 45 mm Hg; pH, 7.44; FIO2, 0.3  ( B ) Pa O2, 65 mm Hg; PaCO2, 52 mm Hg; pH, 7.38; FIO2, 0.4  ( C ) Pa O2, 65 mm Hg; PaCO2, 40 mm Hg; pH, 7.48; FIO2, 0.4  ( D ) Pa O2, 90 mm Hg; PaCO2, 60 mm Hg; pH, 7.32; FIO2, 0.5  ( E ) Pa O2, 133 mm Hg; PaCO2, 55 mm Hg; pH, 7.41; FIO2, 0.6
  • 78. Q  65 Y/O comes with cough and exertional sob of several month duration. He has smoked for 35 years. On physical examination, he is sweating, ruddy, and cyanotic. His pulse rate is120/min and regular, respiration rate is 30/min and labored, and blood pressure is 150/90 mm Hg. The neck veins are distended to the angle of the jaw when sitting upright. The chest shows hyperinflation, prolonged expiration, wheezing, and crackles at each posterior base. The pulmonic sound is increased, and there is a summation gallop. An enlargedand tender liver edge is felt 2 cm below the costal margin. He has marked dependent edema up to the knees.The hematocrit is 55%, and leukocyte count is 8000/μL. Arterial blood gases with the patient breathing room air arePaO2, 47 mm Hg; PaCO2, 50 mm Hg; and pH, 7.30. Spirometry performed 2 years earlier showed a forced expiratoryvolume in 1 sec (FEV1) of 0.65 L and a forced vital capacity (FVC) of 3.05 L. Chest radiography shows hyperinflation, clear lung fields, and biventricular enlargement. Ventilation-perfusion lung scanning shows multiple matched fillingdefects that are not segmental. Doppler studies of the legs are negative.After treatment of the patient’s acute condition, which of the following is the best long-term therapy for  this patient?  ( A ) Nifedipine  ( B ) Warfarin  ( C ) Bosentan  ( D ) Oxygen  ( E ) Phlebotomy
  • 79. Case Study  A 65-year old male hospital in-patient has smoked cigarettes since he was 18 years old. He has a chronic cough and marked sputum production. When his doctor starts to give him the usual talk about losing weight, he explains that since he has about fifty pounds to lose, he has tried to exercise, but is unable to because of shortness of breath with any activity. Upon further questioning, he comments that his symptoms have been present for a very long time, but he was hospitalized due to a marked exacerbation of his complaints. On auscultation, rhonchi and wheezes are heard.His laboratory results are as follows:  pCO2 60 mm Hg(35-45 mm Hg)  pH 7.34( 7.35-7.45)  bicarbonate 31 mEq/L( 24 mEq/L)  Na+ 140 mEq/L( 135-145 mEq/L)  K+ 4.0 mEq/L( 3.5-5.5 mEq/L)  Cl-100 mEq/L(98-109 mEq/L)  What is the primary disorder? a) metabolic acidosis with a normal anion gap b) metabolic acidosis with an elevated anion gap c) metabolic alkalosis d) respiratory acidosis e) respiratory alkalosis
  • 80. Ans.D  This patient has symptoms and signs of chronic obstructive pulmonary disease, specifically chronic bronchitis. Symptoms and signs include cough, sputum production and dyspnea with exertions. Patients tend to be stocky or overweight, as the case here. Auscultation will reveal wheezes and rhonchi. This patient is retaining CO2, since his pCO2 is elevated. CO2 is in equilibrium with carbonic acid. An increase in CO2 will shift the Henderson Hasselbalch equation to the left, resulting in acidosis. Since the cause of the primary problem is respiratory, e.g. retention of CO2, this is a respiratory acidosis. This is reflected in the pH being reduced as well.Metabolic acidosis (choice a, choice b) is incorrect because the primary problem is not due to a administration of acid, excess metabolic acid formation, or loss of base. Although the bicarbonate level is abnormal in this patient, that is due to metabolic compensation for the respiratory acidosis. Alkalosis (choice c, choice e) are incorrect because his pH is acidotic. Although compensatory mechanisms can bring the pH towards the normal range, compensatory mechanisms will never overshoot.
  • 81.  A 54-year-old man is hospitalized because of severe shortness of breath, ankle swelling, and confusion of 5 days’ duration. He has smoked for 35 years. On physical examination, he is sweating, ruddy, and cyanotic. His pulse rate is 120/min and regular, respiration rate is 30/min and labored, and blood pressure is 150/90 mm Hg. The neck veins are distended to the angle of the jaw when sitting upright. The chest shows hyperinflation, prolonged expiration, wheezing, and crackles at each posterior base. The pulmonic sound is increased, and there is a summation gallop. An enlarged and tender liver edge is felt 2 cm below the costal margin. He has marked dependent edema up to the knees. The hematocrit is 55%, and leukocyte count is 8000/μL. Arterial blood gases with the patient breathing room air are PaO2, 47 mm Hg; PaCO2, 50 mm Hg; and pH, 7.30. Spirometry performed 2 years earlier showed a forced expiratory volume in 1 sec (FEV1) of 0.65 L and a forced vital capacity (FVC) of 3.05 L. Chest radiography shows hyperinflation, clear lung fields, and biventricular enlargement. Ventilation-perfusion lung scanning shows multiple matched filling defects that are not segmental. Doppler studies of the legs are negative. After treatment of the patient’s acute condition, which of the following is the best long-term therapy for this patient?  ( A ) Nifedipine  ( B ) Warfarin  ( C ) Bosentan  ( D ) Oxygen  ( E ) Phlebotomy
  • 82. Ans. D  In patients with cor pulmonale caused by chronic hypoxemia, oxygen therapy is the treatment of choice; it may decrease the heart failure and polycythemia seen in this condition.
  • 83. Asthma  Classification: Management Grouping  Mild Intermittent Asthma  Occasional exacerbations (Less than twice per week)  Mild Persistent Asthma  Frequent exacerbations (>twice weekly, but not daily)  Moderate Persistent Asthma  Daily symptoms with daily Beta Agonist use  Severe Persistent Asthma  Continuous Symptoms and frequent exacerbations  Treatment  short acting MDIs as needed, long acting bronchodilators ( once asthma becomes moderate to severe add these as adjuncts to inhaled steroids), inhaled steroids ( first line agent in all persistent asthmas) , systemic steroids, monteleukast ( add this as adjunct in moderate to severe asthma)  Status asthmaticus
  • 84. Asthma Examples of different therapeutic approaches:  Mild Intermittent: use only prn albuterol; if related to exercise, use albuterol one-half hour prior to exercise; also used: cromolyn one half-hour prior to exercise.  Mild Persistent: daily: low dose inhaled steroids; and use albuterol intermittently as needed. May use inhaled cromolyn.  Moderate Persistent: use peak flow meter daily; use med dose inhaled steroid or low dose steroids plus serevent or singulair. Others switch to Advair. PO steroids prn.  Severe Persistent: use peak flow meter daily; po steroids as needed. Daily meds to include high dose inhaled steroids, singulair, serevent or possibly Advair.
  • 85. Classification: Mild Intermitt ent Mild Persist ent Mod Persist ent Severe Persistent Sym ptom s < 2 / week > 2 / week Daily Continual Night sx < 2 / month > 2 / month > 1 / week Frequent FEV 1 > 80% predicte d > 80%pr edicted 60- 80% < 60% Peak flow Varia bility < 20% 20- 30% > 30 % > 30%
  • 86. Management of different categories of Asthma:Mild Intermitte nt Mild Persistent Mod Persistent Severe Persistent Peak Flow Meter X X Preventive Meds (daily): --Inhaled Cromolyn X (X) --Inhaled Steroids X (low dose) X (med dose) X (high dose) --Po Singulair X X X --Advair Diskus ( Salmetrol/ Fluticasone) X X --Serevent ( Salmetrol) X X Rescue Medications --Albuterol X X X X --Po Prednisone prn Prn Prn prn Skin Testing: Allergy Evaluation Consider Consider Consider
  • 87. Case Study  A 68-year-old man with asthma is evaluated because he needs to use his albuterol inhaler at night once or twice a week after waking up with chest tightness. His forced expiratory volume (FEV) is 2.18 L (65% of predicted) before and 2.62 L(82% of predicted) after inhaled albuterol. Current medications include inhaled fluticasone, 440 μg twice daily, and an albuterol metered-dose inhaler as needed. Which of the following should be done next to better control his symptoms?  ( A ) Increase fluticasone to 880 μg twice daily  ( B ) Add salmeterol  ( C ) Add prednisone  ( D ) Add allergen immunotherapy  ( E ) Add a long-acting theophylline at bedtime
  • 88. Key Point In patients with moderate-to-severe asthma not responding to adequate doses of a short-acting ß-agonist and inhaled corticosteroids, the next step is addition of a long-acting ß-agonist.
  • 89. Case Study  A 25-year-old woman is evaluated because of a 3-year history of a nonproductive cough. The cough is aggravated by bicycle riding and occasionally awakens her from sleep. During the past year, she experienced two episodes of bronchitis followed by a dry cough persisting for 2 months. The cough worsened when she visited her sister in Alaska. She has seasonal symptoms of watery, runny nose and sneezing. There is no postnasal discharge, nasal congestion, heartburn, weight loss, or night sweats. She does not smoke. Her physical examination and chest radiography are normal. Spirometry shows forced expiratory volume in 1 sec (FEV1) 3.29 L; forced vital capacity (FVC), 4.13 L; and FEV1/FVC ratio of 79%. Which of the following is the best next management step?  ( A ) Chest computed tomography  ( B ) Bronchoscopy  ( C ) Methacholine inhalation challenge testing  ( D ) Observation and reassurance  ( E ) Therapeutic trial of a proton pump inhibitor
  • 90. Cough Variant Asthma Cough-variant asthma is nonproductive, provoked by exercise and cold air, disturbs sleep, and worsens after a lower respiratory tract infection. The inhalation of methacholine produces airway obstruction in most patients with asthma; less than 10% of normal persons have positive responses ( false +ves) .
  • 91. Case Study  A 45-year-old woman is evaluated because of dyspnea during exercise that began when she started an aerobics class. She has dyspnea, chest tightness, and a nonproductive cough after 15 minutes of vigorous step exercises. The symptoms worsen slightly when she stops, then gradually abate. She has a 5-pack-year smoking history but quit 10 years ago. The physical examination, chest radiography, and electrocardiography are all normal. Spirometry shows forced expiratory volume in 1 sec (FEV1), 2.72 L (83% of predicted); forced vital capacity (FVC), 3.2 L(86% of predicted); and FEV1/FVC ratio of 85%. Postexercise spirometry shows FEV1, 2.04 L (25% drop from baseline),and FVC, 3.00 L (2% drop from baseline). Which of the following management options should be done next?  ( A ) Reassure the patient  ( B ) Prescribe an albuterol inhaler 15 minutes before exercise  ( C ) Perform an exercise stress test  ( D ) Measure lung volumes and diffusing capacity  ( E ) Perform high-resolution computed tomography of the chest
  • 92. Ans.b  For patients with exercise-induced asthma, an inhaled ß-agonist should be prescribed before exercise.
  • 93. Case Study  WHEN CONSIDERING THE DRUG TREATMENT OF ASTHMA WITH INHALER DEVICES A. The incidence of oral candidiasis is increased by the use of spacer devices. B. Salmeterol is indicated for p.r.n. usage. C. Intermittent terbutalin has been shown to lead to long term worsening of asthma. D. Steroid dosage of 600mg daily has been shown to be associated with adrenal suppression in adults. E. Sodium cromoglycate is of no proven value in treating acute asthmatic attacks
  • 94. Ans.E  Spacer devices decrease the incidence of oral candidiasis by preventing the deposition in the mouth. Salmeterol is a long acting beta antagonist, its action is slow in onset and therefore it should be given regularly rather than p.r.n. The Committee on Safety of Medicines has reported that salbutamol and terbutaline have not been shown to lead to a worsening of mild asthma. In adults an inhaled dosage of steroid of 1,500 micrograms daily is associated with adrenal suppression . Sodium cromoglycate is of no value in and acute attack and is only indicated for prophylaxsis.
  • 95. OSA  Check for symptoms of excessive daytime sleepiness  Diagnose by sleep study.  Obesity – neck circumference > 17cm important predictor.  Check local anatomy, Throat crowding, secondary factors (thyroid, cushings) causing obesity.  Obesity Hypoventilation syndrome  Rx – c-pap at nights
  • 96. Q  A 43-year-old man is evaluated because of uncontrolled hypertension, documented in and outside of the office, despite moderate doses of hydrochlorothiazide and enalapril. For the past 6 months he has noted increased fatigue and irritability that he attributes to personal problems at work. He admits to difficulty concentrating at work. He has two beers before bed to fall asleep. He is a salesman, smokes one pack of cigarettes per day, and lives alone. His neck circumference is 17.5 in. On physical examination, he has a ruddy complexion, body mass index is 32, and blood pressure is 158/88 mm Hg. Jugular venous distention cannot be evaluated because of obesity. An S4 is present. The remainder of the examination is normal. A complete blood count, serum electrolytes, serum creatinine, blood urea nitrogen, electrocardiography, and chest radiography are normal.  Which of the following is most likely to establish a diagnosis?  ( A ) Ambulatory blood pressure monitoring  ( B ) Pulmonary function studies  ( C ) Polysomnography  ( D ) Arterial blood gases and blood volume determination
  • 97. Q2  A 58-year-old man is evaluated because of daytime sleepiness. He is requesting an evaluation at this time because last week he fell asleep while driving and had a minor accident. He is a lifetime nonsmoker and is otherwise healthy. On physical examination, his body mass index is 26. There are no obvious abnormalities of his oropharynx. Chest and cardiac examinations are normal. There is no peripheral edema. Chest radiography and electrocardiography are normal. Overnight polysomnography for 6 hours of sleep shows 60 episodes of apnea (cessation of airflow for more than 10 seconds) per hour accompanied by frequent oxygen desaturation below 85%. There is evidence of rib cage and abdominal motion during the apneic periods. Which of the following is the most appropriate form of therapy for this patient?  ( A ) Nasal continuous positive-airway pressure  ( B ) Uvulopalatopharyngoplasty  ( C ) Progesterone  ( D ) Mandibular repositioning device  ( E ) Nocturnal supplemental oxygen by nasal cannula
  • 98. Ans. A  Nasal continuous positive-airway pressure (CPAP) is the standard initial treatment for patients with symptomatic moderate-to-severe obstructive sleep apnea syndrome. It works by splinting the upper airway in an open position.  Surgical procedures such as uvulopalatopharyngoplasty (UPPP) and tracheostomy are best considered in severely ill patients for whom more conservative measures such as nasal CPAP are ineffective.  Although tracheostomy cures obstructive sleep apnea, it carries associated complications and is poorly accepted by patients. UPPP, whether performed with conventional or laser surgery, has variable long-term results and also has associated complications.
  • 99. Restrictive Diseases - Sarcoidosis Pathophysiology  Noncaseating granuloma formation  Idiopathic  underlying genetic predisposition  Precipitated by trigger  Infection (e.g. Mycobacteria, Borrelia Burgdorferi), Environmental exposure (e.g. Beryllium, Aluminum)  Common involvement sites (affects all organ systems)  Lungs (>90%): Interstitial Lung Disease  Lymphadenopathy: Hilar adnenopathy (>95%)  Liver (50-80%) : Hepatic Granulomas (86%), Increased Alkaline Phosphatase  Spleen  Skin lesions (25%)  Eyes : Anterior Uveitis ( also in Spondyloarthropathy) , Posterior Uveitis,  Heart (5%)  Tachyarrhythmias  Cardiomyopathy  ERYTHEMA NODOSUM SUGGESTS BETTER PROGNOSIS  Associated with acute arthritis (Lofgren's Syndrome) - Not associated with chronic arthritis  Most common associated nonspecific skin sign
  • 100. Sarcoidosis Diagnosis  Pulmonary Function Testing  Findings consistent with Interstitial Lung Disease  Serum Angiotensin-converting enzyme (Serum ACE)  Increased in 50-80% of Sarcoidosis patients  Biopsy or Cytology (Gold standard)  Finding  Discrete noncaseating epithelioid granuloma  Biopsy sites  Transbronchial lung biopsy (preferred site)  Bronchoalveolar lavage (CD4-CD8 ratio >3.5)  Skin biopsy of lesion  Palpable peripheral lymph node biopsy  Salivary Gland biopsy
  • 101. Sarcoid - lung Radiology: Chest XRay (abnormal in 90% of cases)  Type 0: No abnormality (<10% of cases)  Type I: Lymphadenopathy alone (43% of cases)  Bilateral hilar Lymphadenopathy  Mediastinal Lymphadenopathy  Right paratracheal Lymphadenopathy  Type II: Adenopathy and Infiltrates (24% of cases)  Lymphadenopathy as in Type I Chest XRay findings  Parenchymal infiltrates  Symptomatic respiratory disease presentation  Type III: Infiltrates alone (13% of cases)  Parenchymal infiltrates
  • 102. Sarcoidosis - Treatment  Cutaneous  for Erythema nodosum use NSAIDS, For sarcoid lesions use intralesional Corticosteroids  For uveitis  topical CS, Systemic CS if refractory  For pulmonary sarcoidosis stage 2 or 3  Indications : Dyspnea , Persistent cough and Widespread debilitating disease  systemic CS/ AZA
  • 103. Sarcoidosis - Prognosis Prognosis  Overall mortality (from respiratory failure): 1-5%  Factors suggestive of worse prognosis  Onset after age 40 years  Black race  Chronic Hypercalcemia  Specific higher risk organ involvement  Neurologic involvement  Cardiac involvement  Eye involvement (Chronic Uveitis)  Renal involvement (Nephrocalcinosis)  Cystic bone lesions  Progressive pulmonary fibrosis
  • 104. Idiopathic Pulmonary Fibrosis  CXR – Picture  Physical – chest bibasal creps  HRCT confirmatory  Not responsive to steroids
  • 105. TB - Screening  Screen for LTBI in persons at increased risk of recent infection, including immigrants within the last 5 years from high prevalence countries; pre- and postexposure in travelers visiting countries with a high prevalence of TB; those in recent contact with a case of infectious TB; health care workers with potential exposure to mycobacteria; and residents and employees of high-risk congregate settings where local epidemiology indicates a high rate of TB.  Screen for LTBI in persons with conditions associated with an increased risk of developing active TB, including HIV infection, diabetes, silicosis or exposure to silica dust, low body weight, chronic renal failure or hemodialysis, gastrectomy, jejunoileal bypass, cirrhosis of the liver, organ transplantation, anticancer chemotherapy and other immunosuppression (e.g., TNF-α antagonists), malignant head or neck carcinoma, or fibrotic changes on CXR film compatible with previous TB.  Screen children and adolescents for LTBI who have risk factors for development of active disease (e.g., HIV), have been exposed to adults at high risk for TB, or have been adopted from abroad, especially if they were born in countries with endemic TB.
  • 106. TB - Screening  PPD test is used for screening and its sensitivity approaches 100% in pts with normal immunity.  As an alternative to the PPD, consider using a whole- blood IFN-γ assay such as QuantiFERON-TB Gold, recognizing its limitations in children and immunocompromised patients and that all currently available studies on the sensitivity and specificity of this test are limited by the lack of an eternal “gold standard” for the diagnosis of tuberculous infection, but that there is good reason to believe that the QuantiFERON-TB Gold test is superior to the TST in BCG-vaccinated individuals because it employs the ESAT-6 antigen that is lacking in BCG
  • 107. TB and PPD Interpretation: PPD under 5 mm  Negative  Observe Patient Interpretation: PPD 5 mm or greater  Positive if  HIV Infection  Tuberculosis contact  Immunosuppressed (e.g. HIV, Prednisone >15 mg qd)  Abnormal finding on Chest XRay  Management  Chest XRay and exam for disseminated disease  If cxr –ve INH for 9 mos
  • 108. TB and PPD Interpretation: PPD 10 mm or greater  Positive if  Health care workers  New immigrant within last 5 years  Intravenous Drug Abuse  Homeless  Under 4 years old  Malnutrition  Diabetes Mellitus  Silicosis  Tuberculosis endemic to region  Management  Chest XRay and exam for disseminated disease
  • 109. TB and PPD Interpretation: PPD 15 mm or greater  Positive in all persons  Management  Chest XRay and exam for disseminated disease  INH 9 mos if no active disease ( i.e; you are treating Latent TB)  If active disease  First step Sputum for AFB smear x 3, Sputum for AFB cx and Sensitivity; isolate the patient, isolate organism for susceptibility testing  start emperic multi drug regimens HRZE  For failure/ resistant TB  SHRZE  Sputum –ve pts can be taken off Isolation.
  • 110. Latent TB – Imp Points  In case of patients who are TB contacts:  Do PPD test and if –ve repeat another after 8-12 wks  Begin latent TB therapy in contacts such as children and patients with HIV even if the initial skin test is negative.  If the second test is also negative, stop medication in immunocompetent individuals. If a known high-risk TB exposure has occurred in a patient with HIV infection, continue LTBI treatment for the full period, regardless of TST results.
  • 111. Tuberculosis & PPD – Imp Points  PPD skin test  Next step if ppd +ve ( Latent TB)  INH rx  When screening for LTBI in TB contacts, if the initial PPD result is negative a second PPD should be done 8 to 12 weeks after the last known exposure ( In a review of literature, several studies support a maximum interval of 8 weeks from initial infection to development of a delayed-type hypersensitivity reaction. A Tuberculin Skin Test > 8 weeks from last exposure is recommended. )
  • 112. Tuberculosis & PPD – Imp Points  Do not do a Tuberculin Skin Test on any patient  with a history of severe blistering reactions  with previously documented active TB, With a history of treatment for TB, With a documented previous positive TST result  because the TST remains positive in most of these patients  Do not use TST to see if it turns –ve to monitor the success of your treatment  that wont happen  TST remains +ve even after therapy.
  • 113. Tuberculosis & PPD – Imp Points  Delay the PPD Test for 4 to 6 weeks after a major viral illness such as measles, mumps, rubella, or influenza, because cutaneous anergy can develop, leading to a false-negative TST result.  Do periodic serial PPD screening in persons with ongoing exposure to TB ( Health Care Workers, Residents)  q1year is good enough!
  • 115. Common Causes of Chronic Cough  Smoker’s cough  Chronic bronchitis due to smoking  Post-nasal drip  Post-infectious Viral Bacterial—Bordetella pertussis, Mycoplasma, Chlamydia ( tracheobronchitis)  Gastroesophageal reflux disease  Asthma  Angiotensin converting enzyme inhibitors
  • 116. Less Common Causes of Chronic Cough  Infectious causes Tuberculosis—typical or atypical,  Fungal  Endobronchial lesions  Benign — bronchial adenoma, carcinoid tumor  Malignant — bronchogenic carcinoma, metastatic cancer  Foreign body  Interstitial lung diseases  Hypersensitivity pneumonitis  Bronchiolitis obliterans with organizing pneumonia, ( BOOP)  Sarcoidosis  Chronic interstitial pneumonia  Chronic aspiration  Masses in the neck/thyroid disorders  Hair impinging on the tympanic membrane  Bronchiectasis  Occult congestive heart failure  Disorders of the pleura, pericardium, diaphragm  Psychogenic/habitual cough  Occupational bronchitis  Enlarged tonsils or uvula
  • 117. History  The cause(s) of chronic cough may become apparent after taking a careful history.  Is the symptom a cough or “hawking” or clearing the throat?  It helps to have the patient act out the cough to distinguish true cough from throat clearing.  Is the cough dry or productive? If so, what is produced?  Are systemic symptoms such as fever, night sweats or weight loss present?  A detailed history of the work and home environment should be taken with emphasis on possible exposure to noxious inhalants or allergens. The history should include the time and circumstances of onset, frequency, and aggravating and relieving factors.  Patients with asthma may note worsening of cough on exposure to cold air, irritants or allergens. Is there an allergic history? Does the patient wheeze with cough?
  • 118. HISTORY  Is the cough accompanied by dyspnea? If so, congestive heart failure or interstitial lung disease may be suspected.  Is the cough related to time of day, eating or position?  A nocturnal cough may be associated with asthma, post-nasal drip, congestive heart failure or gastroesophageal reflux disease (GERD). Half of the patients with GERD have none of the classic symptoms.  Does the patient cough while eating? Chronic aspiration is common in the elderly patient, especially following stroke.  Is the patient on angiotensin converting enzyme inhibitors or other drugs that may predispose to cough or asthma?  Do not overlook ophthalmic preparations. Beta blocker eye drops may precipitate asthma.
  • 119. Physical  The physical examination may provide clues to the causes of cough.  Examination of the upper airways may show nasal mucous membrane swelling, post-nasal drip or nasal polyps.  The finding of wheezes, rhonchi or crackles may indicate asthma, bronchitis, COPD, interstitial lung disease or congestive heart failure.  The finding of unilateral wheezing may be due to an endobronchial lesion or foreign body.  Masses in the neck, including thyroid enlargement, can compress the trachea and cause cough.
  • 120. Diagnostic Tests  The work-up for chronic cough should begin with standard posterior- anterior and lateral chest x-rays  these often reveal the presence of underlying infectious or neoplastic causes of chronic cough.  Spirometric studies before and after bronchodilator administration may reveal reversible airways obstruction (asthma).  In patients with normal base-line spirometry  methacholine inhalation challenge (MIC) is indicated to rule out asthma that presents primarily with cough. ( COUGH VARIANT ASTHMA)  Computerized tomograms (CT) of the sinuses are superior to plain x-rays in identifying sinusitis.  High-resolution or spiral CT scans of the thorax may reveal subtle changes consistent with cough due to chronic interstitial pneumonia or bronchiectasis.  The finding of a reduced single breath diffusing capacity (DLCO) may suggest interstitial lung disease.  Barium esophagograms and upper gastrointestinal endoscopy have a low sensitivity (48%) and specificity (76% ) for identifying GERD as the culprit in chronic cough  monitoring the esophageal pH for 24 hours is the gold standard. ( If cough is the only symptom of GERD it gets difficult to diagnose  so, 24hr Ph monitoring)  In patients suspected of having chronic aspiration, a video swallowing study with a speech therapist in attendance should be performed. ( SWALLOW EVALUATION)  A systematic approach to the work-up of a patient with nondrug-related chronic cough is presented in THE NEXT SLIDE. If you suspect Drug related cough  stop the drug and observe
  • 121.
  • 122. Chronic Cough Post-nasal Drip Syndrome  Post-nasal drip syndrome is said to be one of the most common causes of chronic cough and is caused by a variety of conditions including vasomotor rhinitis, allergic rhinitis, nasal polyps and chronic sinusitis.  The diagnosis is made on clinical grounds.  Patients may complain of a tickle or drainage of liquid in the back of the throat.  On examination, cobblestoning of the nasal or oropharygeal mucosa may be observed. In many patients cough may be the only symptom of post-nasal drip syndrome.  Confirmation of the diagnosis may depend on the resolution of symptoms after treatment with antihistamines and intranasal or systemic corticosteroids.
  • 123. Chronic Cough Asthma & Cough-variant Asthma  Typically, asthma patients complain of episodic wheezing, cough, chest tightness and dyspnea and demonstrate reversible obstructive air flow.  In so called cough-variant asthma  a dry cough, particularly at night, is the only symptom and routine spirometry is normal.  Diagnosis : Spirometry is normal in cough variant type  The diagnosis is often made on the basis of a favorable clinical response to empirically administered beta2-agonist bronchodilators and inhaled corticosteroids, and a positive bronchoprovocation test using methacholine inhalation challenge (MIC)  A positive MIC test, defined as a 20% or greater decrease in the FEV1 after MIC, indicates bronchial hyperreactivity but not necessarily asthma. For example, bronchial hyperreactivity may follow viral respiratory tract infections and persist for as long as 6 weeks. Because MIC has a positive predictive value of from only 60% to 80%, Irwin and colleagues advise that a positive test must be correlated with favorable response to therapy before concluding that a patient has cough-variant asthma.
  • 124. Chronic Cough Gastroesophageal Reflux-related Chronic Cough  GERD is a very common problem. Surveys of the general population have led to estimates that 10% of the adult population of the United States have daily heartburn and a third have intermittent symptoms; moreover, GERD has been shown to cause 10% to 40% of cases of chronic cough  Cough in GERD is triggered by reflux of acid into the distal esophagus and stimulation of an esophageal-tracheobronchial reflex. Cough is not dependent on aspiration into the larynx or tracheobronchial tree.  Proving the relationship of chronic cough to GERD can be difficult. The lack of typical symptoms of reflux and negative endoscopic and radiographic studies do not rule it out.  The 24-hour esophageal pH monitoring test has become the gold standard for diagnosis and has both a sensitivity and specificity approaching 90%.  Correlation of the results of pH monitoring with response to therapy adds to the reliability of the test.  If GERD is the sole cause of chronic cough, aggressive anti-reflux therapy should eliminate the cough in nearly all cases. One study reported 100% success. Treatment involves the use of dietary, mechanical and drug therapy. Drug therapy should be initiated with proton pump inhibitors for GERD.
  • 125. Chronic Cough Post-infectious Cough  Patients who have had recent viral respiratory tract infections may have prolonged cough that is refractory to treatment. Airway hyperresponsiveness can be demonstrated by MIC testing in some cases. Treatment with bronchodilators and inhaled or systemic corticosteroids in moderate to high doses may help relieve symptoms. The cough can be self- perpetuating and cause continuing trauma to the airways, and in these cases, prolonged suppression with narcotics may eventually allow resolution.  Bordetella pertussis (the cause of whooping cough) infection in adults should be included in the differential diagnosis of chronic cough. In one series of 75 patients with chronic cough lasting longer than 2 weeks, 21% had pertussis.
  • 126. Chronic Cough Angiotensin Converting Enzyme Inhibitor Cough  Angiotensin converting enzyme inhibitor (ACEI) drugs are frequently used in the treatment of hypertension, congestive heart failure and myocardial infarction. Ten to 20% of patents taking ACEI drugs develop cough. There is no evidence at this time that any one ACEI drug is less likely to cause cough than another. In spite of this well-documented side effect, referrals to a specialist for evaluation of chronic cough still occur frequently. Many of these patients have had extensive and costly work-ups and treatment with a variety of medications, including antihistamines, antibiotics, cough suppressants and corticosteroids, without relief.  Clinically, the cough may begin from as early as 3 weeks to as long as a year after starting treatment. The severity of the cough can vary from a mild tickle in the throat to a severe hacking, debilitating cough that interferes with sleep, work and social function. It is frequently worse at night and in the supine position.  When the ACEI drug is discontinued, the cough usually abates in 2 weeks but may persist for months.  Angiotensin ll receptor antagonists have not been associated with an increased incidence of cough.
  • 127. Chronic Cough Less Common Causes of Cough  Chronic cough may be the presenting complaint in patients who ultimately prove to have tumors, both benign and malignant, sarcoidosis or other infiltrating lung diseases; all these conditions require special investigations to make the diagnosis.  Psychogenic or habitual cough does exist but patients should not be put in this category without an exhaustive work-up, failure of empirical therapy and prolonged follow up.
  • 128. Chronic Cough Symptomatic Treatment  The treatment of cough is effective only if directed at the cause, but patients should be offered symptomatic relief while awaiting the results of specific therapy. Expectorants such as iodides and guaifenesin, hydration, inhaled steam, cough lozenges and hard candies are helpful. Dextromethorphan and codeine are effective cough suppressants. When to Refer  When the patient with chronic cough remains symptomatic despite evaluation and treatment for 6 to 8 weeks, the primary care physician should consider referral to a specialist. In difficult cases referral to a pulmonologist for evaluation, therapy and for specific testing such as fiberoptic bronchoscopy and MIC is recommended. Referral for upper gastrointestinal endoscopy and 24-hour pH monitoring may be indicated to rule out cough due to GERD. Referral to an allergist may be indicated for allergy testing and subsequently for immunotherapy if the patient is sensitive to an unavoidable antigen. Medicolegal Issues  One of the most common reasons patients file suit is for failure to diagnose cancer.  Even though bronchogenic carcinoma is an uncommon cause of chronic cough in the context of a normal chest x-ray, it must not be overlooked .  Failure to diagnose tuberculosis is another cause of litigation but again would be an unlikely cause of chronic cough with normal chest roentgenograms .
  • 130. Characteristics of Solitary Pulmonary Nodules Variable Benign Malignant Age < 30 years > 50 years Calcificati on Popcorn, dense, concentric None or minimal Nodule edge Smooth, round Irregular, spiculated Smoking history Never smoked > 20 pack-years Size of diameter < 1.5 cm > 1.5 cm
  • 131. X-ray Characteristics of SPN  Benign nodule charecterestics : the presence of calcification, which can be a diffuse speckled, or “popcorn,” pattern, typical of a hamartoma, or a large central nidus or concentric calcification typical of a granuloma.  The second important factor distinguishing a malignant from a benign nodule is the growth rate. Since the “doubling time” of a lung cancer ranges from 15 to 450 days, the nodule that does not increase in diameter over a two year period can be considered benign. Any lesion that increases in size over a two year period of observation, or less, must be considered malignant until proven otherwise. One exception is a nodule doubling in less than 20 days, which usually suggests an acute inflammatory process.  The third important characteristic is the appearance of the nodule’s edge. Benign lesions have smooth rounded edges, whereas the incidence of neoplasm increases dramatically in lesions with irregular, spiculated borders. An increasing incidence of malignancy occurs, ranging from 20- 93%, depending on the degree of border irregularity
  • 132. Diagnosis  The first step in evaluating a SPN is to try to obtain old chest x-rays for comparison  If this is not possible, and the nodule does not have a classic, calcified appearance typical of a granuloma or hamartoma, then further testing or a period of careful observation must be undertaken.  A CT scan can help distinguish the pattern of calcification, and classify lesions as “indeterminate” based on the presence of stippled or eccentric calcification and medium density, or “benign” based on the presence of fat density typical of a hamartoma.  The most common CT finding in early stage adenocarcinoma and squamous cell carcinoma of the lung is that of a solitary pulmonary nodule which enhances after administration of IV contrast. In small cell carcinoma, however, hilar and mediastinal adenopathy secondary to metastases is the most common CT presentation. The presence of irregular margins, associated air bronchogram, convergence of the surrounding structure, or the involvement of three or more blood vessels is more likely in malignant lesions.  If a period of observation is chosen, chest x-rays, and possibly serial CT scans, should be done at 3-month intervals over at least a two year period to determine if any change in the size of the nodule has occurred. An increase in the diameter of the nodule by 25% indicates a doubling of the mass volume a sign of malignancy.
  • 133. Diagnosis - PET  Because of the difficulty with noninvasive diagnosis of the SPN, new radiologic techniques are being studied, including positron emission tomography imaging (PET), which is able to distinguish benign from malignant pulmonary nodules by measuring 18-fluorodeoxyglucose (FDG), and by showing increased FDG uptake and retention in malignant cells. PET scanning is a valuable, noninvasive tool with a 95% sensitivity for identifying malignancy and a specificity of 85% or greater. However, false positive results may be obtained in lesions containing an active inflammatory process (for example a reactive lymphadenopathy), and this diagnostic modality is not generally available.
  • 134. SPN When to Refer  Once the decision has been made that the patient’s SPN may represent a malignancy, a histologic diagnosis is needed. If the patient’s SPN has characteristics strongly suggesting malignancy, and there are no contraindications to surgery, refer to a thoracic surgeon.  In most other circumstances refer to a pulmonologist for further workup. Diagnostic procedures may include: fiberoptic bronchoscopy aided by fluoroscopy, or CT-guided transthoracic fine needle aspiration. The yield of these procedures in the diagnosis of the small solitary pulmonary nodule (< 1.5 cm in diameter) is about 40% for fiberoptic bronchoscopy, and 50% for fine needle aspiration. The incidence of pneumothorax requiring chest tube insertion from bronchoscopic transbronchial biopsy is about 5% and from needle aspiration about 25%, depending on patient characteristics and variation of local physician technique.  Thoracoscopic resection or thoracotomy is needed for diagnosis in about 20% of patients, in whom the less invasive techniques were not successful.
  • 135.  A 43-year-old woman is evaluated because of an abnormal chest radiograph taken before an elective hysterectomy for fibroids. She has no previous history of pulmonary disease. Her cardiac and pulmonary review of systems is unremarkable. The patient smoked one pack of cigarettes per day from age 16 to 33 years, but has not smoked since then. On physical examination, her blood pressure is 120/60 mm Hg and the lung fields are clear. The remainder of the examination is unremarkable. Her laboratory evaluation, including a complete blood count and chemistry profile, is normal. Spirometry shows forced expiratory volume in 1 sec (FEV1), 2.72 L (84% of predicted); forced vital capacity (FVC), 3.68 L (98% of predicted); and FEV1/FVC ratio of 74%. Chest radiography shows an approximately 1-cm nodule in the left lower lobe periphery. There are no previous radiographs. High-resolution helical computed tomography (CT) of the 1.2-cm lesion in the left lower lobe is shown. The full chest CT shows no evidence of mediastinal adenopathy. Which of the following is the best management option?  ( A ) Bronchoscopy with transbronchial biopsies  ( B ) No further studies are needed  ( C ) Repeat high-resolution CT in 3 months  ( D ) Resection of the lesion with video-assisted thoracoscopic surgery  ( E ) Percutaneous fine needle aspiration of the lesion
  • 136.  The likelihood of a solitary pulmonary nodule being malignant substantially decreases if the lesion is small (approximately 1cm), has smooth borders, is located in a lower lobe, and, most importantly, has central calcification.  Observation at 3 month intervals for 2 years to ensure stability of the finding is sufficient!!
  • 138. EFFUSIONS  EXUDATES: pleural fluid [protein] / plasma [protein] > 0.5 and/or pleural fluid [LDH] / plasma [LDH] > 0.6 and/or pleural fluid LDH >200 or >2/3 of serum LDH  TRANSUDATES – Nephrotic syndrome, CHF, Atelectasis, Cirrhosis (Hydrothorax)  EMPYEMA  Diagnosis  By thoracentesis. Except in known CHF, must be done in all pleural effusions.
  • 139.  A 56-year-old man with chronic alcoholism is evaluated because of fatigue, decreased appetite, and episodes of sweating. He has a cough productive of purulent, foul-smelling grayish-green sputum and describes heaviness in the right lower chest. The patient has reduced his activity, but only missed 2 days of work as a painter because of his illness. On physical examination, his temperature is 37.4 °C (99.3 °F), pulse rate is 84/min, respiration rate is 14/min, and blood pressure is 132/85 mm Hg. There is dullness to percussion at the right lateral and posterior lung base associated with decreased breath sounds. Chest radiography shows a large right-sided pleural effusion and a small parenchymal infiltrate. The effusion did not layer along the chest wall in a lateral decubitus film. A thoracentesis is performed, and the leukocyte count is 32,500/μL with83% neutrophils, the pH is 7.12, and glucose is 25 mg/dL. The fluid is sent for culture, and antibiotics are begun.  Which of the following should be done next to manage the pleural fluid?  ( A ) Insertion of a pigtail catheter into the pleural space  ( B ) Insertion of a large-bore chest tube  ( C ) Repeat thoracentesis if the pleural effusion does not improve in 48 hours  ( D ) Open surgical decortication  ( E ) Reevaluation of the pleural effusion in 7 days
  • 140. Ans.  Frankly purulent pleural fluid, pleural fluid pH <7.20, and,possibly, loculated collections of fluid are indications for chest tube drainage
  • 141. Critical Care Aspects 1. Shock 2. Respiratory Failure 3. Acute coronary Syndrome 4. Acid-Base problems 5. Neurological Emergencies 6. GI Bleeding
  • 142. Shock - Objectives  Recognize the clinical manifestations of shock - the major classifications of shock  Identify the IV solution type that is best suited for water and electrolyte loss  Describe conditions that may indicate successful volume resuscitation
  • 143. Shock  Hypovolemic  Distributive – Septic, Drug overdose ( opiod, BZDs, Ethanol), Hepatic failure  Obstructive – PE, Cardiac tamponade, Pneumothorax, Air Embolism  Cardiogenic – Acute LHF, RHF  Miscellaneous – think about Adrenal insufficiency, hypocalcemia, Autonomic dysfunction in a shock that is refractory to pressor support and in a shock that tends to persist even after correction of the initial etiology
  • 144. Distributive Shock  1. Distributive (maldistribution of blood flow) - Warm Shock – Warm extremities  a. Septic  b. Neurogenic – Drug overdose, vasovagal  c. Anaphylactic  d. Toxic shock syndrome  e. Metabolic
  • 145. Hypovolemic Shock  Hypovolemic (decreased cardiac output) Secondary to loss of intravascular volume  a. Hemorrhagic  b. Nonhemorrhagic 1. GI fluid loss (e.g., diarrhea, vomiting) 2. Renal loss (e.g., diabetes insipidus, osmotic diuresis) 3. Evaporative loss (e.g., hyperthermia, severe burns) 4. Third space loss (e.g., peritonitis, pancreatitis)
  • 146. Cardiogenic Shock  Cardiogenic (decreased cardiac output due to cardiac factors) a. Impaired left ventricular contractility (e.g., ischemia, infarct, congestive myopathy) b. Impaired right ventricular contractility (e.g., right ventricular infarction) c. Acute regurgitant lesions (e.g., mitral or aortic regurgitation, intraventricular septal rupture) d. Obstructive cardiac lesions (e.g., aortic stenosis, subaortic stenosis) e. Bradyarrhythmias or tachyarrhythmias
  • 147. Obstructive Shock  Obstructive (decreased cardiac output due to factors extrinsic to the heart)  a. Pericardial tamponade  b. Tension pneumothorax  c. Pulmonary embolism  d. Severe pulmonary hypertension
  • 148. Shock  Pulmonary Embolism Etiology Clinical Features Investigations – BEDSIDE ECHO THE BEST INITIAL SCREENING TEST IF U SUSPECT SHOCK DUE TO PE IN A CRITICALLY ILL PATIENT – Can give you information on other causes – Acute MI, Cardiac tamponade, Acute corpulmonale ( ARDS, PE) Treatment
  • 149. Shock  Cardiac Tamponade Etiology – Trauma,Rapid Effusions - tumor Clinical features Investigations – X-Ray, CT chest, 2D echo Treatment
  • 150. Shock  Pneumothorax ( obstructive phenomena) Etiology – usual scenario- an icu patient on ventilator – sudden hypotension Others – trauma, rupture emphysemal bullae Clinical features – auscultate – SILENT!! - look at the ventilator parameters ----- high peak pressures as well as plateau pressures Investigations – CXR, Needle thoracentesis Treatment – tube thoracostomy, high fio2
  • 151. Shock  Acute Corpulmonale - ARDS - Pulmonary Embolism Rx – IV fluids, correct the cause
  • 152. Shock  Cardiogenic - Right Heart Failure  Symptoms, EKG, Signs, Enzymes  Treatment – fluids, fluids, fluids cardiac cath, fix the lesion
  • 153. Shock  Cardiogenic – Left Heart Failure  Causes - Acute MI  Rx – IABC – To buy time - Dopamine, Dobutamine - then cardiac cath, fix the lesion - ptca
  • 154. Shock  Distributive SEPSIS ---- distributes to periphery  WARM shock, systemic vasodilation - reduced SVR ( Swan Ganz ) Investigations – pan cultures, cxr, --- consider random cortisol…if requiring pressor support for a long time  rule out relative adrenal insufficiency. Rx – Fluids,fluids, fluids  antibiotics  hydrocortisone 100 q8hrs if relative/ absolute adrenal insufficiency. Remove the source - ? IV Catheters, hemodialysis access, foley Pressor support – norepinephrine, dopamine, vasopressin  Distributive/ Anaphylactic – Epinephrine, corticosteroids, beta agonists, Antihistamines
  • 155. Shock  Hypovolemic – Hypovolemia Vs. Dehydration - GI blood loss, vomiting, excessive diarrhea,Other trauma/ blood loss, Diabetes Insipidus, Diuretics ( overdiuresis), Diuretic abuse, Laxative abuse, Bulimia nervosa, Surreptious vomiting - Rx: Fluids, blood if required, Correct hypernatremia slowly < 0.5 meq/hr if present - Rx the underlying cause – fix GI bleeds, any ? coagulopathy, consults if required - ? Renal failure – prerenal – indicators Urine Na+, FeNa, urine sp gravity,
  • 156. Shock – Case studies 1) A 38-year old female on birth control pills, has suddenly become extremely short of breath. Someone has seen her collapse and called 911. She was diaphoretic and complained of severe chest pain before she collapsed. She is now in the ER/ED and you have been asked to evaluate her. Her old records show that she is a cocaine abuser and was admitted for subarachnoid hemorrhage 6 weeks ago from which she completely recovered. Clinical findings. Vitals : B.P 65/ palpable, R.R 45. Pulse 140, Tm: 99.2 F. Chest showed decreased breath sounds in right lower lobe and distant heart sounds. Pulse oximetry revealed 88%. EKG showed sinus tachycardia with s wave in lead I, q wave in lead III and T wave inversion in lead III. 2D echo showed hypokinetic right ventricle with pulmonary hypertension. You started her on Intravenos fluids and her blood pressure has slightly improved to 66/30. Your next step in management ? A) Transfer to cath lab and notify the interventional cardiologist stat B) Intra aortic balloon counterpulsation C) Thrombolytic therapy D) Surgical Embolectomy and Inferior vena cava filter E) Obtain cardiothoracic surgery consult for subxiphoid window